You are on page 1of 186

See discussions, stats, and author profiles for this publication at: https://www.researchgate.

net/publication/330042427

Single choice questions in ORTHOPAEDICS Mahmood A Sulaiman

Article · January 2019

CITATIONS READS
0 2,599

1 author:

Mahmood Aljumaily
University of Mosul
24 PUBLICATIONS   7 CITATIONS   

SEE PROFILE

Some of the authors of this publication are also working on these related projects:

MCQ FOR ORHOPAEDIC View project

ethics and medical practice View project

All content following this page was uploaded by Mahmood Aljumaily on 01 January 2019.

The user has requested enhancement of the downloaded file.


1001
Single choice questions in

ORTHOPAEDICS

Mahmood A Sulaiman
Thousand and one
Single choice questions in

ORTHOPAEDICS

Mahmood A Sulaiman

Professor of orthopedic
College of the medicine
Mosul University
IRAQ

Dedication

To memory of my parents,
To memory my teachers,
To our colleagues, students, trainees and patients, all of
whom have helped to make our lives interesting,
stimulating and worthwhile.

Mahmood-2017
Preface
From early eighties when we start our training in orthopedic in Iraqi
commission of medical specialization, Apley's system of orthopedic and fracture was
one the most important reference in our training program. We follow and read many
edition of this nice valuable book and we start to teach our student according to its
system.

This work is simple trail to improve our reading to this book in self-
assessments and to provide a local pool of question to our orthopedic examination.
We select 1001 questions of single choice from Apley's system of orthopedic and
fracture 9th edition, the first 501 questions there is one correct choice, while in
reminder 500 questions there is one wrong choice ( all are correct except). The
answers listed in the end of this book. I am sure that there is many defects in this
work, and we are happy to receive any useful comment to adjust the next edition.

Professor Mahmood A Sulaiman

Mosul – Iraq

2017
PART- ONE
Select single most appropriate

1- The most common symptom in orthopaedics is


a- Loss of function.
b- Deformity.
c- Pain.
d- Swelling.
e- Stiffness
2- The Common sites of referred pain from shoulder is
a- Neck.
b- Scapular region.
c- Chest.
d- Arm.
e- Forearm and hand.
3- The Common sites of referred pain from hip is
a- Leg and foot.
b- Knee.
c- Gluteal region.
d- Lateral side of the thigh.
e- Sacroiliac region.
4- Grade 4 muscle power is
a- Normal power.
b- Movement against resistance.
c- Movement against gravity.
d- Movement with gravity eliminated.
e- Only a flicker of movement.
5- Deltoid muscle supplied by
a- C3,4.
b- C4,5.
c- C5,6.
d- C6,7.
e- C7,8.
6- Wrist flexors supplied by
a- C3,4.
b- C4,5.
c- C5,6.
d- C6,7.
e- C7,8.
7- Wrist extensor muscles supplied by
a- C3,4.
b- C4,5.
c- C5,6.
d- C6,7.
e- C7,8.
8- Ankle dorsiflexion muscles supplied by
a- L 2,3.
b- L 2,3.
c- L 3,4.
d- L 4,5.
e- L 5, S 1.
9- Ankle plantarflexion muscles supplied by
a- L 2,3.
b- L 2,3.
c- L 3,4.
d- L 4,5.
e- L 5, S 1.
10- Big toe flexion muscles supplied by
a- L 3.
b- L 3.
c- L 4.
d- L 5.
e- L S 1.
11- Big toe extension muscles supplied by
a- L 3.
b- L 3.
c- L 4.
d- L 5.
e- L S 1.
12- The most common cause of osteomyelitis in adults is
a- Acute haematogenous osteomyelitis.
b- Postoperative osteomyelitis.
c- Subacute osteomyelitis.
d- Diabetes mellitus.
e- Posttraumatic osteomyelitis.
13- Malignancy after use of metal implant
a- The risk related.
b- The risk probably discounted.
c- Large number of cases.
d- Occur commonly in site of implant.
e- Commonly giant cell tumors.
14- The disadvantage of ultra-high molecular weight polyethylene
a- Not susceptible to deformity.
b- Susceptible to deformity.
c- Crack development not occur.
d- Good hardness.
e- Had high coefficient of friction.
15- The metal implant
a- Cause infection.
b- Enhance drainage.
c- Titanium alloys more susceptible to infection in comparison with stainless
steel.
d- Stainless steel more susceptible to infection in comparison with titanium
alloys.
e- Impedes the formation of biofilm.
16- Emergency expenditure in amputee
a- Is 10- 30% percent for transtibial.
b- Is 5-10% percent for transtibial.
c- Is 10- 30% percent for transfemoral.
d- Is 30-40% percent for transfemoral.
e- Is 30-50% percent for transtibial.
17- Bone marrow edema
a- Gradual and progressive.
b- Acute and self-limiting.
c- MRI shows focal changes.
d- Scintigraphy shows reduced activity.
e- Histological examination shows marrow osteonecrosis.
18- Rapidly destructive osteoarthritis
a- Occurs mainly in elderly woman.
b- Associated with deposit of urate crystal.
c- Associated with deposit of pyrophosphate crystal.
d- There is no bone destruction.
e- It results from analgesic therapy.

19- The most common cause of Charcot's joint is


a- Myelomeningocele.
b- Tabes dorsalis.
c- Leprosy.
d- Syringomylia.
e- Diabetic neuropathy.
20- In surgical treatment of hemophilic arthropathy, the clotting factors
concentration should be raised postoperatively to above
a- 5%.
b- 15%.
c- 25%.
d- 50%.
e- 75%.
21- Gas in the joint indicate
a- Staphylococcus aureus (MRSA) infection.
b- Pseudomonas aeruginosa infection.
c- Proteus mirabilis infection.
d- Kingella kingae infection.
e- Escherichia coli infection.
22- The surgical drainage in suppurative arthritis indicated in
a- If the hip is involved.
b- If the knee is involved.
c- If the ankle involved in adult.
d- In shoulder of young children.
e- In elbow of young children.
23- The reliable investigation for diagnosis of tuberculosis is
a- Mantoux test.
b- ESR & CRP.
c- Synovial fluid culture.
d- Synovial fluid aspirate examination.
e- Synovial biopsy.
24- Synovial fluid examination shows rhomboid shape crystals in
a- Rheumatoid arthritis.
b- Osteoarthritis.
c- Gout.
d- Pseudo- gout.
e- Reiter's disease.
25- Uric acid lowering drugs indicated
a- Acute gout attack.
b- Chronic gout.
c- Hyperuricaemia.
d- Pseudo gout.
e- Recurrent acute attack.
26- Pseudo gout characterized by
a- Affect large joints.
b- Cause severe pain.
c- Affect small joint.
d- There is no joint swelling.
e- Osteophyte formation.
27- Polyarticular osteoarthritis
a- The patients is usually old man.
b- The patients is usually old woman.
c- The patients is usually middle age man.
d- The patients is usually middle age woman.
e- The patients is usually young woman.
28- Type I collagen make up to
a- 10% of unmineralized matrix.
b- 20% of unmineralized matrix.
c- 40% of unmineralized matrix.
d- 60% of unmineralized matrix.
e- 80% of unmineralized matrix.
29- In bone PTH act to promote osteoclastic resorption
a- It dose by direct action.
b- It dose by indirect action.
c- Through decrease expression of RANKL.
d- Through increase production of OPG.
e- Through decrease in 1, 25 ( OH)2 D.
30- In renal tubular rickets there is
a- Myopathy.
b- No growth defect.
c- Serum phosphate decreased.
d- Serum alkaline phosphatase decreased.
e- Urine calcium increased.
31- In renal glomerular rickets , there is
a- Positive family history.
b- Myopathy.
c- No growth defect.
d- Serum calcium increased.
e- Serum phosphate decreased.
32- The x-rays features of scurvy is
a- Localized bone rarefaction.
b- Lytic transverse band at the juxta-epiphyseal zone.
c- Epiphyseal ossification.
d- The ossific centers shows ring sign.
e- Increased density in the metaphyseal region
33- In Paget's disease
a- Serum calcium is high.
b- Serum phosphate is low.
c- Serum alkaline phosphatase is normal.
d- 24-hour urinary hydroxyproline decreased.
e- 24-hour urinary hydroxyproline increased.
34- Multiple epiphyseal dysplasia
a- Children are average height.
b- Walk with a waddling gait.
c- Head and face are normal.
d- Head and face are abnormal.
e- The lower limb had normal height.
35- The commonest form of abnormally short stature is
a- Osteogenesis imperfacta.
b- Metaphyseal dysplasia.
c- Achondroplasia.
d- Dyschondroplasia.
e- Hypochondroplasia.
36- Nail patella syndrome
a- Inherited as an autosomal recessive trait.
b- Inherited as sex linked dominant.
c- The radial head sublaxated medially.
d- There is bony protuberance on the lateral aspects of iliac blades.
e- There is bipartite patella.
37- Type I osteogenesis imperfacta
a- Usually appears at birth.
b- There is marked deformity.
c- The sclera is white.
d- Inherited autosomal dominant.
e- Teeth usually is abnormal.
38- Sprengel's shoulder deformity
a- The patient has short neck.
b- There is a failure of vertebral segmentation.
c- Associated vertebral anomalies is rare.
d- Inherited autosomal dominant.
e- The scapula is small and abnormally high.
39- Radioulnar synostosis is
a- Associated with anterior dislocation of the radial head.
b- Associated with medial dislocation of the radial head.
c- There is complete loss of pronation and supination.
d- There is some degree of pronation.
e- There is some degree of supination.
40- Wide excision of tumors
a- Dissection carried out through normal tissue.
b- The entire compartment in which the tumor removed.
c- Dissection goes beyond the tumor but only just.
d- It is appropriate for high-grade intra-compartmental lesion.
e- It is appropriate for low-grade extra-compartmental lesion.
41- Fibrous cortical defect
a- The commonest site is the diaphysis.
b- The commonest site is the epiphysis.
c- Recurrence is common.
d- The commonest benign lesion of bone.
e- Encountered in young adults.
42- Fibrous dysplasia
a- The common site is distal radius.
b- The cortical bone replaced by cellular fibrous tissue contain woven bone.
c- May affect one bone.
d- Small, single lesion cause local pain.
e- It is self-limiting after maturity.
43- Regarding malignant transformation in chondroma of adult
a- There is spot of calcification.
b- Foot bone affection.
c- There is lytic lesion.
d- The biopsy is very helpful.
e- The biopsy is not helpful.
44- In eosinophilic granuloma of bone
a- The patients is usually young adult.
b- Cause local pain and tenderness.
c- Usually heals spontaneously.
d- Usually treated by complete excision or curettage.
e- X-rays shows ill-define diffuse osteolytic lesions in long bone.
45- Simple bone cyst
a- Is benign tumor.
b- Diagnosis depends on biopsy.
c- Cause local ache.
d- Commonly affect metaphysis of proximal tibia.
e- Appears during childhood.
46- Aneurysmal bone cyst
a- Appears during childhood.
b- Almost any bone may be affected.
c- Usually discovered incidentally or after pathological fractures.
d- Is a subarticular in end of long bones.
e- The lesion is central in metaphysis.
47- Chondromyxoid fibroma
a- Affect adult.
b- Is more common in upper limb.
c- Presenting symptom is ache.
d- Malignant changes is not rare.
e- Treatment of choice is excision.
48- In enchondroma there is
a- Well-define eccentric osteolytic lesion.
b- Pain in site of lesion.
c- Flicks of calcification within lucent area is common features.
d- Solitary lesion.
e- A high risk of malignant changes.
49- Mesenchymal chondrosarcoma
a- Tend to occur in in older individual.
b- In about 10% of cases, the tumor lies in soft tissue.
c- In about 20% of cases, the tumor lies in soft tissue.
d- In about 50% of cases, the tumor lies in soft tissue.
e- Behavior is usually less aggressive.
50- Central chondrosarcoma
a- Develops either in tubular or flat bone.
b- X-rays shows osteolytic lesion without expansion.
c- X-rays shows no flicks of calcification.
d- X-rays shows no cortical destruction.
e- Sometime appears on surface of flat bone.
51- Osteosarcoma
a- Presented by pain increased by activity.
b- Affect most commonly long bone diaphysis.
c- Serum alkaline phosphatase is normal.
d- ESR is usually normal.
e- Characterized by malignant stromal cell showing osteoid formation.
52- Osteosarcoma
a- Usually graded as IA or IB.
b- Usually graded as IIA or IIB.
c- Usually graded as III.
d- Multi-agent neoadjuvent chemotherapy given for 8-12 weeks before biopsy.
e- Centrally, large pulmonary metastases may be completely resected.
53- Adamentinoma
a- Is low-grade tumor.
b- Has predilection to posterior cortex of tibia.
c- X-rays shows atypical bubble like defect in the posterior cortex of tibia.
d- X-rays shows bone rarefaction and punched-out defect in the posterior cortex
of tibia.
e- The patients is usually old female.
54- Periosteal osteosarcoma
a- May changed to more aggressive dedifferentiated Parosteal osteosarcoma.
b- Situated on the surface of the bone.
c- Occurs in the children.
d- X-rays shows defect of medullary canal.
e- X-rays shows thick periosteal reaction.
55- Paget's sarcoma
a- It is the commonest complication of Paget's disease.
b- Presented as a painless mass.
c- It is the commonest osteosarcoma in patients older than 50 years.
d- Metastasis is late.
e- Graded as IIA.
56- Hypercalcemia may treated by
a- Ensuring adequate hydration,
b- Reducing the phosphate intake,
c- Vit D supplement.
d- Increasing the phosphate intake.
e- Avoid administering bisphosphonates.
57- Spastic cerebral palsy
a- Associated with damage to the extra- pyramidal system.
b- Associated with damage to the pyramidal system.
c- Due to cerebellar damage.
d- Catheterized by increased muscle tone and hyporeflexia.
e- Appears in the form of muscular incoordination during voluntary movement.
58- Giant cell tumor of the tendon sheath identical to
a- Ganglion.
b- Giant cell tumor of bone.
c- Non-specific synovitis.
d- Pigmented villi- nodular synovitis.
e- Synovial sarcoma.
59- Synovial sarcoma involve the joint in
a- 10 %.
b- 20 %.
c- 40 %.
d- 60 %.
e- 80 %.
60- Operative correction is indicated if the hip flexion deformity in cerebral palsy
a- Is more than 10 degrees.
b- Is more than 20 degrees.
c- Is more than 30 degrees.
d- Is more than 40 degrees.
e- Is more than 50 degrees.
61- Preganglionic lesion of brachial plexus injuries
a- Is surgically repairable.
b- Potentially capable of recovery.
c- Have good prognosis.
d- Recovered spontaneously but mild residual symptoms may persist.
e- Cannot recover and it is surgically irreparable.
62- Spastic flexion deformity of knee in cerebral palsy may be revealed only when
a- The hip is flexed to 20 degrees.
b- The hip is flexed to 40 degrees.
c- The hip is flexed to 50 degrees.
d- The hip is flexed to 70 degrees.
e- The hip is flexed to 90 degrees.
63- In Erb’s palsy
a- A reliable indicator of recovery is return of biceps activity by the third month.
b- Absence of biceps activity by third month completely rule out later recovery.
c- Is due to injury of C8 and T1.
d- The baby lies with the arm supinated and the elbow flexed.
e- Reflexes are absent and there may be a unilateral Horner’s syndrome.
64- Winging of the scapula
a- Occur if the latissimus dorsi paralyzed.
b- Demonstrated by the patient pushing forwards against the wall.
c- Results from the injury of the long thoracic nerve (C8, T1).
d- It usually recovers spontaneously, though this may take a week or longer.
e- It usually requires operative stabilization by transferring pectoralis minor or
major to the lower part of the scapula.
65- Very high lesions radial nerve injury
a- May caused by fractures of the humerus or after prolonged tourniquet pressure.
b- Are usually due to fractures or dislocations at the elbow.
c- Cannot extend the metacarpophalangeal joints of the hand. There is an obvious
d- There is wrist drop, as well as inability to extend the metacarpophalangeal
joints or elevate the thumb.
e- There is wrist drop, the triceps paralyzed and the triceps reflex is absent.
66- Wrist drop following closed fracture
a- Is usually third degree lesions.
b- Can afford to wait for 4 weeks to see if it starts to recover.
c- If it does not recover by 4 weeks , then EMG should be performed
d- The nerve should explored, if the EMG at 12 weeks shows denervation
potentials and no active potentials.
e- Should be explored and the nerve repaired or grafted as soon as possible if
there is good surgical facilities.
67- Isolated anterior interosseous nerve lesions
a- Are extremely common.
b- The signs are similar to those of a high median nerve injury.
c- The usual cause is brachial neuritis.
d- There is no sensory loss.
e- The thenar eminence is wasted.
68- The femoral nerve injury
a- May be injured by a gunshot, shell, by pressure or traction during an operation.
b- The patient is able to extend the knee actively.
c- There is numbness of the anterior thigh and anterior aspect of the leg.
d- The knee reflex is normal.
e- Severe neurogenic pain is uncommon.
69- The superficial peroneal nerve
a- Innervating the tibialis anterior muscle.
b- Innervating the extensor digitorum longus.
c- Innervating the extensor hallux longus.
d- Descends along the fibula.
e- Injury resulting in paraesthesia and numbness on the dorsum around the first
web space.
70- Tourniquet pressure as cause of nerve injury
a- Is an uncommon cause of nerve injury in orthopedic operations.
b- Damage is due prolonged ischemia.
c- Damage is due to direct pressure.
d- Injury is therefore more likely with a pneumatic tourniquet.
e- Injury is therefore more likely with a wide cuff.
71- Chronic compartment syndrome
a- Long-distance runners sometimes develop pain along the postero-lateral aspect
of the calf.
b- Pain brought on night after muscular exertion.
c- Swelling of the postero-lateral calf muscles.
d- The condition diagnosed from the history and confirmed by measuring the
compartment pressure before exercise.
e- Release of the fascia is curative.
72- The thromboprophylaxis
a- DVT can reduced by one-thirds by prolonging thromboprophylaxis.
b- The ideal duration of thromboprophylaxis is not known.
c- Current evidence supports 30 days for knee replacement.
d- Current evidence supports 14 days for hip replacement and hip fracture.
e- Should not be prolonged after discharge from hospital.
73- The angle between the anatomical axis of the femur and the axis of the
femoral neck is
a- Approximately 128 degrees (±3 degrees).
b- Approximately 128 degrees (±5 degrees).
c- Approximately 125 degrees (±5 degrees).
d- Approximately 125 degrees (±3 degrees).
e- Approximately 122 degrees (±3 degrees).
74- The angle between the anatomical axis of the femur and a tangent to the joint
line of the knee is, On the lateral aspect
a- Approximately 75 degrees (±5 degrees).
b- Approximately 80 degrees (±2 degrees).
c- Approximately 85 degrees (±5 degrees).
d- Approximately 90 degrees (±2 degrees).
e- Approximately 90 degrees (±5 degrees).
75- General complication of osteotomy and deformity correction is
a- Under- and over - correction of the deformity.
b- Tension on a nearby nerve.
c- Compartment syndrome.
d- Infection.
e- Non-union.
76- Bone allografts
a- Cannot be stored.
b- There is no potential for transfer of infection.
c- Sterilization done by ethylene oxide without alteration in the physical
properties.
d- Sterilization done by ionizing radiation with alteration in the physical
properties.
e- Antigenicity cannot reduced by freezing, freeze-drying or by ionizing
radiation.
77- Hair removal
a- Shaving before surgery is useful.
b- Shaving before surgery is safe.
c- Shaving day before surgery reduced wound infection.
d- Depilatory creams used the day before surgery increased wound problems.
e- Depilatory creams used the day before surgery without an increase in wound
problems.
78- Risk of asymptomatic venous thromboembolism in hip fracture
a- 10%.
b- 20%.
c- 40%.
d- 60%.
e- 80%.
79- Low molecular weight heparin
a- Its safety similar to unfractionated heparin.
b- Need constant monitoring.
c- Effectively reduces the prevalence of venographic DVT in hip replacement
surgery.
d- Not reduces the prevalence of venographic DVT in knee replacement surgery.
e- It is effective as the unfractionated heparin.
80- Unlocked elastic intramedullary nails
a- Are rigid rods.
b- Increasingly used in the treatment of long-bone shaft fractures in children.
c- Inserted through the physes at either end of the long bone.
d- Function as rigid internal fixation.
e- Insufficient reaming potentially risks the bone splitting.
81- Cancellous autografts
a- Incorporated by a process analogous to fracture healing.
b- Carried risk for transfer of infection.
c- Induce an inflammatory response in the host
d- Incorporated more rapidly into host bone.
e- Are particularly useful when large defects to be filled.
82- Referred shoulder pain syndromes results from
a- Tendinitis.
b- Glenohumeral arthritis.
c- Suprascapular nerve entrapment.
d- Subluxation.
e- Cardiac ischaemia
83- Active shoulder movements are best examined
a- From left side the patient.
b- From right side the patient.
c- From both sides the patient.
d- From behind the patient.
e- From front the patient.
84- The commonest cause of pain around the shoulder is
a- A disorder of the rotator cuff.
b- Glenohumeral arthritis.
c- Nerves lesions.
d- Subluxation.
e- Cardiac ischaemia
85- Chronic shoulder tendinitis
a- Pain and slight stiffness would not restrict simple activities.
b- Pain persist and not affected by activities.
c- The patient usually aged between 20 and 30.
d- Characteristically pain is sever with activities.
e- Characteristically pain is worse at night.

86- A full thickness tear of rotator cuff of shoulder


a- Always follow a long period of chronic tendinitis.
b- Always follow a jerking injury of the shoulder.
c- There is sudden pain and the patient is unable to abduct the arm
d- There is sudden pain and the patient is able to abduct the arm.
e- Injecting a local anaesthetic into the sub-acromial space restore abduction.
87- Ultrasonography of shoulder
a- Is not accurate like MRI for identifying and measuring the size of rotator cuff
tears.
b- It has the advantage that it can identify the quality of the muscles.
c- Cannot always be accurate in predicting the reparability of the tendons.
d- Are usually normal in the early stages of the cuff dysfunction.
e- Is not save imaging.
88- Arthroscopic acromioplasty
a- Cannot achieve the same basic objectives as open acromioplasty.
b- This procedure has now become the gold standard.
c- The outer side of the acromion trimmed.
d- If a complete cuff tear encountered, then open repair indicated.
e- Delayed the postoperative rehabilitation.
89- Acute calcific tendinitis of shoulder
a- Acute pain always follow deposition of calcium hydroxyapatite crystals.
b- Affects 20–30 year-olds.
c- Is thought that vascular reaction leads to fibrocartilaginous metaplasia and
deposition of crystal.
d- Pain due to the calcification.
e- Affects 30–50 year-olds.
90- Asymptomatic calcification of the shoulder rotator cuff
a- Is uncommon.
b- It is painful after exercises.
c- Appears as an incidental finding in shoulder x-rays.
d- The tendon is thick and hypertrophies.
e- Treatment should directed to the calcification rather than the impingement.
91- Frozen shoulder
a- Is a well-defined disorder characterized by progressive painless stiffness of the
shoulder.
b- Stiffness become complete followed by pain.
c- Is usually resolves spontaneously after about 18 months.
d- The condition not associated with diabetes.
e- The condition not appears after recovery from neurosurgery.
92- Condensing osteitis of the clavicle
a- May be no more than a reaction to the mechanical stress.
b- Is usually seen in men of 40–60.
c- Present with pain at the lateral end of the clavicle.
d- Pain aggravated by adducting the arm.
e- X-rays reveal sclerosis and lytic lesion in the lateral end of the clavicle.
93- Sterno-costo-clavicular hyperostosis
a- Is seen in younger people.
b- Is usually unilateral.
c- Patients develops painless swelling.
d- The histological changes are non-specific.
e- The Microorganisms can be identified.
94- Indications for shoulder arthroplasty is
a- Osteoarthritis of acromioclavicular joint.
b- Early rheumatoid arthritis
c- Fracture- dislocation of the proximal humerus.
d- Early avascular necrosis of the humeral head.
e- Severe arthritis with cuff arthropathy.
95- The commonest complication for shoulder arthroplasty is
a- Infection
b- Loosening of the components.
c- Implant failure.
d- Peri-prosthetic fracture.
e- Rotator cuff failure.
96- Arthrodesis of the gleno-humeral joint
a- Is commonly performed.
b- Is still a useful operation for severe shoulder dysfunction.
c- Postoperative function is limited.
d- Caused painful restriction of gleno-humeral movement.
e- The optimal position is 10 degrees of flexion, 10 degrees of abduction and 10
degrees of internal rotation.
97- Medial epicondyle epiphysis appears at
a- 2 years.
b- 4 years.
c- 6 years.
d- 8 years.
e- 10 years.
98- Proximal radio-ulnar synostosis
a- Is acquired deformity.
b- Is uncommon.
c- Function is usually good.
d- Surgical separation improved forearm rotation.
e- A rotation osteotomy are more suitable.
99- Posttraumatic unreduced dislocation of the head of radius
a- Surgical treatment would not improve function.
b- Is usually associated with cubitus varus.
c- May follow unreduced old Monteggia fracture.
d- Is usually bilateral.
e- Is commonly associated with old supracondylar fracture.
100- Severe rheumatoid arthritis of the elbow
a- Treated by arthrodesis.
b- Joint replacement is usually successful.
c- Treated by arthroscopic debridement.
d- Synovectomy is worthwhile.
e- Treated by excision of the radial head.
101- Gout of elbow region
a- Affect ulno-humeral joint.
b- Affect radio-humeral joint.
c- The olecranon bursa is a favourite site.
d- Affect the common extensor origin.
e- Affect the common flexor origin.
102- The best non operative treatment for posttraumatic elbow stiffness is
a- Passive exercise.
b- Early active movement through a functional range.
c- Manipulation under anesthesia.
d- Manipulation under regional anesthesia.
e- Aggressive passive manipulation.
103- Recurrent elbow instability commonly associated with
a- Muscles weakness.
b- Posterior capsular injury.
c- Lateral collateral ligament injuries.
d- Fracture olecranon.
e- Fracture coronoid.
104- Tennis elbow characterized by
a- Localized tenderness at or just below the lateral epicondyle.
b- Pain radiate widely.
c- Damage to the bones.
d- Damage to soft-tissue attachments around the elbow.
e- The elbow flexion and extension are full and painless.
105- Semi-constrained elbow arthroplasty
a- Associated with instability.
b- Associated with dislocation.
c- Good results achieved in 90% of carefully selected patients.
d- Allow some of the forces to absorb by the soft tissues whilst maintaining some
intrinsic stability.
e- Had a high failure rate due to loosening.
106- Stability of the scapho-lunate joint is tested by
a- Gripping or pinching the lunate with one hand, the triquetral-pisiform with the
other, and then applying a sheer stress.
b- Pushing the pisiform radial wards against the triquetrum.
c- Pressing hard on the palmar aspect of the scaphoid tubercle while moving the
wrist alternately in abduction and adduction.
d- Pushing the wrist medially then flexing and extending it.
e- Holding the radius and then balloting the ulnar head up and down.
107- The normal radial deviation is about
a- 5°.
b- 15°.
c- 25°.
d- 35°.
e- 50°.
108- The embryonic arm buds appear about
a- Fourth week.
b- Sixth week.
c- Eighth week.
d- 10th week.
e- 12th week.
109- Digital rays begin to appear
a- By 6th week.
b- By 8th week.
c- By 10th week.
d- By 12th week.
e- By 14th week.
110- Secondary ulna dysplasia occur in children who had
a- Madelung's deformity.
b- Achondroplasia.
c- Hereditary multiple exostosis.
d- Ulnar club hand.
e- Symbrachydactyly.
111- Comptodactyly is
a- Conjoint digit.
b- Failure of embryological separation.
c- True cleft hand.
d- A bent finger.
e- Phocomelia.
112- The most unstable of the carpal bones is
a- Pisiform.
b- Lunate.
c- Hamate.
d- Trapezoid.
e- Scaphoid.
113- Clinodactyly is
a- A bent finger.
b- A digit bent sideways.
c- Complex polydactyly.
d- Multiple digits syndactyly.
e- Atypical cleft hand.
114- When there is severe pain and restriction of wrist movement in Kienböck’s
disease , the best treatment is
a- Vascular bone graft.
b- Radial shortening.
c- Radial dome osteotomy.
d- Radio-carpal arthrodesis.
e- Scapho- capitate fusion.
115- The cardinal feature of the ‘rheumatoid hand’ is
a- A reciprocal ulnar deviation of the fingers.
b- Combination of instability and erosive tenosynovitis eventually leads to
tendon rupture.
c- The erosion of distal radio-ulnar joint.
d- The erosion of radiocarpal joint and intercarpal joints.
e- Synovitis around the ulnar head with rupture of extensor digiti minimi.
116- The first x-rays changes in rheumatoid arthritis is
a- Peri-articular osteoporosis.
b- Diminution of the joint space.
c- Soft-tissue swelling.
d- Bony erosions.
e- Marked joint destruction.
117- Flexor tenosynovitis in rheumatoid hand
a- Is obvious as extensor tendon involvement.
b- Cause rupture of the flexor pollicis longus tendon.
c- Cause rupture of the flexor digitorum profundus tendon.
d- Cause rupture of the flexor digitorum superficialis tendon.
e- May presented as carpal tunnel syndrome.

118- Relative shortness of the ulna appears in association with


a- Carpal tunnel syndrome.
b- Kienböck’s disease.
c- Ulna-carpal impaction syndrome.
d- Central triangular fibrocartilage complex perforations.
e- Late ulno-carpal arthritis.
119- Early post traumatic boutonniere deformity treated by
a- Division of the extensor tendon distally.
b- Surgical repair and splintage for 6 weeks.
c- Surgical repair and fixation by K-wire for 6 weeks.
d- Splinting the PIP joint in full extension for 6 weeks.
e- Tendon transfer and splintage for 6 weeks.
120- The normal angle of distal radial tilt is
a- 6°.
b- 11°.
c- 22°.
d- 30°.
e- 65°.
121- The normal angle of palmar tilt in distal radius
a- 3°.
b- 5°.
c- 8°.
d- 11°.
e- 22°.
122- The jointed strut in the wrist formed by
a- The scaphoid, trapezoid and thumb.
b- The scaphoid, trapezium and thumb.
c- The scaphoid, trapezoid and second metacarpal.
d- The scaphoid, capitate and central metacarpal.
e- The scaphoid, trapezoid and central metacarpal.
123- The distal radial epiphysis appear at age of
a- First year.
b- Second year.
c- Fourth year.
d- Sixth year.
e- Eighth year.
124- With the wrist flexed, the thumb fall normally in
a- Flexion.
b- Supination.
c- Pronation.
d- Extension.
e- Ulnar deviation.
125- Functionally the thumb is
a- A 20% of the hand.
b- A 25% of the hand.
c- A 30% of the hand.
d- A 35% of the hand.
e- A 40% of the hand.
126- Intrinsic muscle of hand
a- Extend of the MCP and flex IP joints.
b- Extend of the MCP and extend IP joints.
c- Flexed of the MCP and extend IP joints.
d- Flexed of the MCP and flex IP joints.
e- Hyperextend of the MCP and flex IP joints.
127- Osteoarthritis in hand affect mainly the
a- Proximal interphalangeal joints.
b- Distal interphalangeal joints.
c- Metacarpophalangeal joints.
d- Carpometacarpal joints.
e- Intercarpal joints.
128- Abduction of the thumb is
a- Sideways movement in the plane of the palm.
b- Sideways movement across the palm.
c- Upward movement at the right angles to the palm.
d- Pressing against the palm.
e- Lifting the thumb backwards behind the plane of the hand.
129- The cause of hand intrinsic minus is
a- Cerebral palsy.
b- Poliomyelitis.
c- Scarring after trauma.
d- Scarring after infection.
e- Shrinkage due to ischaemia.
130- Agricultural injuries of hand usually treated by
a- Flucloxacillin or cephalosporin.
b- Flucloxacillin and fusidin.
c- A broad-spectrum penicillin and fusidin.
d- A broad-spectrum penicillin and metronidazole.
e- Cephalosporin and fusidin.
131- Mobile Boutonniere deformity in rheumatoid arthritis can be treated with
a- Tendon repair.
b- Tendon transfer.
c- Arthroplasty.
d- Arthrodesis.
e- Splint.
132- In combined median and ulnar nerve injuries
a- The thumb is in palm.
b- The thumb is flexed.
c- There is a clawing of thumb.
d- The thumb lie at the side of the hand.
e- The thumb adducted and rotated.
133- The most common finger affected by trigger finger is
a- Little finger.
b- Ring finger.
c- Middle finger.
d- Index.
e- Thumb.
134- The atlanto-dental interval in adult is
a- 1 or 2 mm.
b- 2 or 3mm.
c- 4 or 5 mm.
d- 6 or 8 mm.
e- 8 or 10 mm.
135- the normal synchondrosis between the dens and the body of C2
a- Fuses at about 4 years.
b- Fuses at about 6 years.
c- Fuses at about 8 years.
d- Fuses at about 10 years.
e- Fuses at about 12 years.
136- The most common site of cervical spondylosis is
a- C2/3 and C3/4.
b- C3/4 and C4/5.
c- C4/5 and C5/6.
d- C5/6 and C6/7.
e- C6/7 and C7/T1.
137- The sagittal diameter of the mid-cervical spinal canal suggestive of spinal
stenosis is less than
a- 9 mm.
b- 11 mm.
c- 13 mm.
d- 15 mm.
e- 17 mm.
138- the most common seronegative spondyloarthropathy to affect the cervical
spine is
a- Rheumatoid arthritis.
b- Ankylosis spondylitis.
c- Juvenile poly arthritis.
d- Reiter's disease.
e- Colitis associated arthropathy.
139- Scoliosis with pain suggests
a- A spinal infection until proved otherwise.
b- A spinal tumor until proved otherwise.
c- A prolapse disc until proved otherwise.
d- A structural scoliosis until proved otherwise.
e- A postural scoliosis until proved otherwise.
140- In structural scoliosis
a- Right thoracic curves are the commonest.
b- Left thoracic curves are the commonest .
c- Right lumbar curves are the commonest .
d- Left lumbar curves are the commonest .
e- Right cervical curves are the commonest .
141- Acute pyogenic infection of the spine
a- Is uncommon.
b- Diagnosis and treatment often early done.
c- The infection start in vertebral body with secondary spread to disc.
d- Children and young adult are at greatest risk.
e- The spinal canal is commonly involved.
142- MRI in acute pyogenic infection of spine
a- May show nonspecific changes in the vertebral end plate. :[Comment [h1

b- May show nonspecific changes in the intervertebral disc.


c- Is highly sensitive and highly specific.
d- Is highly sensitive and not specific.
e- Is highly specific but not sensitive.
143- Pott's paraplegia
a- Is the rarest complication of spinal tuberculosis.
b- Early onset paresis is due to pressure by direct cord compression.
c- Late onset paresis is due to pressure by inflammatory edema.
d- In early cases, the prognosis is good.
e- The prognosis of surgical decompression in late cases is good.
144- The disc space collapse is typical
a- Traumatic compression.
b- Infection.
c- Multiple myeloma.
d- Metastatic disease.
e- Osteoporosis.
145- Hydatid disease in spine
a- Usually picked in adulthood.
b- Take many months before diagnosis made.
c- X-rays may reveal translucent area with sclerotic margin.
d- X-rays may reveal local osteoporosis and periosteal reaction.
e- Surgical eradication prevent morbidity and recurrence.
146- Root canal stenosis result from
a- Degenerative changes of disc.
b- Osteoarthritis of facet joint.
c- Thickening of the ligamentum flavum.
d- Bulging of the disc annulus fibrosus.
e- New bone formation may narrow the lateral recesses of the spinal canal and
the intervertebral foramina.
147- Acute disc prolapse
a- Is uncommon in young adults.
b- Is rare in old age.
c- Presented as sciatica only.
d- The patients usually stand with a slight kyphosis.
e- May cause muscle weakness and wasting.
148- The major postoperative complication of disc surgery
a- Is bleeding from epidural veins.
b- Is recurrent prolapse with sciatica is more common and may require revision
decompression surgery.
c- Is injury to the dura and CSF leakage.
d- Is disc space infection.
e- Is injuries to nerve root and spinal cord.
149- The characteristic feature of ‘segmental instability of lumbar spine’ is
a- Intervertebral disc degeneration.
b- Mainly flattening of the ‘disc space’.
c- Marginal osteophytes.
d- The appearance of a ‘traction spur’.
e- Arthritis of facet joint.
150- Lytic or isthmic spondylolisthesis forms
a- 5%.
b- 10%.
c- 20%.
d- 25%.
e- 50%.
151- Sudden, acute pain and sciatica
a- In young people, it is important to exclude prolapse disc.
b- In patients, aged 20–40 years are more likely to have a spinal instability.
c- In those under the age of 20, it is important to exclude infection.
d- In elderly patients may have spondylolysis.
e- In elderly patients may have spondylolisthesis.
152- Intermittent low back pain after exertion
a- Old Patients only may complain of recurrent backache following exertion.
b- Rest relieves this pain.
c- Features of disc prolapse are always present.
d- In those under 50 years, osteoarthritis of the facet joints is common.
e- In early cases, x-rays usually show signs of lumbar spondylosis.
153- Hip disorders at age between 10-20 years mostly
a- Neglected developmental dysplasia of the hip.
b- Infections
c- Perthes’ disease
d- Slipped epiphysis
e- Adults Arthritis.
154- The reported incidence of neonatal hip instability in northern Europe is
approximately
a- One per 1000 live births.
b- Three per 1000 live birth.
c- Six per 1000 live birth.
d- 10 per 1000 live birth.
e- 20 per 1000 live birth.
155- Acetabular dysplasia
a- Always genetically determined.
b- Always follow incomplete reduction of a congenital dislocation.
c- Always follow damage to the lateral acetabular epiphysis or maldevelopment
of the femoral head.
d- The socket is unusually shallow, the roof is sloping and there is deficient
coverage of the femoral head.
e- Faulty load transmission in the lateral part of the joint may lead to primary
osteoarthritis.
156- People with mild acetabular dysplasia
a- The condition exists only as an ‘x-ray diagnoses.
b- May complain of pain over the lateral side of the hip.
c- Some experience episodes of sharp pain in the groin.
d- Complain of movement – particularly abduction in flexion – is restricted.
e- Complain of leg length asymmetry and the femoral head may be felt as a lump
in groin.
157- The recurrence rate of irritable hip is
a- 5%.
b- 10%.
c- 15%.
d- 20%.
e- 25%.
158- Congenital coxa vara
a- Is uncommon developmental disorder of adolescent.
b- Due to a defect of enchondral ossification in the lateral part of femoral neck.
c- Corrected spontaneously with growth.
d- Associated with anteversion of femoral neck.
e- Is bilateral in about one third of cases.
159- Perthes' disease
a- Is common.
b- Is uncommon.
c- Is rare.
d- Patients are usually 10-15 years.
e- The girls are affected two time as often as boy.
160- Adolescent with slipped capital femoral epiphysis
a- Have femoral neck retroversion.
b- There is femoral head anteversion.
c- The physis has decreased obliquity.
d- Have lessor than average body mass index.
e- Have no hormonal imbalance.
161- The treatment of Perthes' disease in children under 6 years of age is
a- Abduction brace.
b- Abduction spica.
c- Pelvic osteotomy.
d- Femoral osteotomy.
e- Symptomatic treatment.
162- Between 4 to 7 years of age, the femoral head depend for its blood supply
venous drainage
a- On both metaphyseal and lateral epiphyseal vessels.
b- On both metaphyseal vessels and blood vessels in ligamentum teres.
c- Almost entirely on the metaphyseal vessels.
d- Almost entirely on the lateral epiphyseal vessels.
e- Almost entirely on the blood vessels in ligamentum teres.
163- The first x-rays change in Perthes' disease
a- Increased density of the proximal femoral epiphysis.
b- Fragmentation of the proximal femoral epiphysis.
c- Rarefaction and cystic changes in metaphysis.
d- Widening of the joint space.
e- Enlargement of the proximal femoral epiphysis.
164- The most important prognostic factor in Perthes' disease is
a- The degree of femoral head collapse.
b- The degree of femoral head involvement.
c- The calcification lateral proximal femoral epiphyseal plate.
d- The age of child.
e- The sex of child.
165- Pre- slip in slipped capital femoral epiphysis
a- The child complains of gluteal pain particularly on rest.
b- There is limitation of movement.
c- Exertion, and there may be a limp
d- Examination may demonstrate reduced external rotation.
e- The x-ray may show widening or irregularity of the physis.
166- The chronic slip in slipped capital femoral epiphysis
a- The child complains of posterior hip pain lasting more than 3 weeks.
b- The pain is continuous without remission.
c- There is loss of internal rotation, abduction.
d- There is some extension and limb lengthening.
e- There is long prodromal history and a severe exacerbation.
167- Sever slip in slipped capital femoral epiphysis
a- Causes marked deformity which, untreated, will predispose to secondary OA.
b- Closed reduction by manipulation should be attempted.
c- Open reduction by Dunn’s method gives fair results.
d- The alternative treatment is to fix the epiphysis without osteotomy.
e- The patient should be told that this may result in 5–7 cm of shortening.
168- Articular chondrolysis in slipped capital femoral epiphysis
a- Cartilage necrosis probably results from slipping.
b- In these cases, bone changes are marked.
c- There is progressive narrowing of the joint space and the hip becomes stiff.
d- This is a rare complication in SCFE.
e- All cases, the condition improves spontaneously
169- The diagnosis of pyogenic arthritis of the hip is confirmed
a- By the classical clinical picture.
b- By typical radiological features and joint effusion on ultrasonography.
c- By the detailed picture provided by MRI.
d- By aspirating pus or fluid from the joint and submitting it for laboratory
examination and bacteriological culture.
e- By early CT scan of the hip.
170- The most important type of motion in the hip for optimal bipedal function is
a- Extension and abduction.
b- Extension and adduction.
c- Flexion and abduction.
d- Flexion and internal rotation.
e- Flexion and external rotation.
171- The cam type femoro-acetabular impingement
a- Affect young female,
b- Affect acetabulum mainly.
c- The main pathology is non-spherical extension of femoral head.
d- Associated with protrusion- acetabuli and acetabular retroversion.
e- The structure primarily damaged is labrum.
172- The changes of osteoarthritis of the hip are most marked
a- In margin of articular surface.
b- The top of the joint.
c- In the infero-medial part of the joint.
d- In the inferior part of the joint.
e- In the medial part of joint.
173- The common cause of primary OA of the hip is
a- Avascular necrosis.
b- Subluxation of the hip.
c- Dysplasia of the hip.
d- Femoro-acetabular impingement.
e- Coxa magna following Perthes' disease.
174- Rheumatoid arthritis of the hip
a- The hip joint is common site.
b- Characterized by other joints affection.
c- The hallmark is progressive bone destruction on both side of joint.
d- Characterized by reduction of joint space and osteophyte formation.
e- Pain behind hip and limping are earliest symptom.
175- Total hip arthroplasty for rheumatoid arthritis
a- Relieve pain but not restore a useful range of movement.
b- It advocated for old patients only.
c- Fracture during operation is rare.
d- The risk of infection is less.
e- Adolescent with juvenile rheumatoid arthritis may be treated by custom-made
prosthesis.
176- Grade III osteonecrosis of the hip
a- The prognosis is good.
b- Decompression is valuable.
c- For young patients partial hip replacement is the treatment of choice.
d- Older patient treated by total hip replacement.
e- Older patient treated by arthrodesis.
177- Transient osteoporosis of the hip
a- Is common.
b- Characterized by pain and rapidly emerging osteoporosis.
c- Radionuclide scanning show decreased activity.
d- The changes last 1-3 months.
e- The x-ray not returns to normal after pain subside.
178- The best treatment for transient osteoporosis of the hip
a- Symptomatic treatment.
b- Calcitonin.
c- Alendronate.
d- Rest.
e- Osteotomy.
179- The indication to intertrochanteric osteotomy is
a- Wide spread osteoarthritis.
b- Sever collapse in avascular osteonecrosis.
c- Osteoarthritis with sever stiffness.
d- Young patient with osteoarthritis associated with joint dysplasia .
e- Rheumatoid arthritis.
180- The ‘bond’ between bone and the implant surface, or cement,
a- Is never perfect.
b- Optimized in new technique.
c- Improved by embedding the implant in methylmethacrylate cement.
d- Improved in recent technique of bone cementing.
e- Improved by fitting the implant closely to the bone bed without cement.
181- Postoperative dislocation following total hip replacement
a- Is uncommon if the prosthetic components are correctly placed.
b- Reduction is easy and traction in adduction.
c- Usually closed reduction and abduction allows the hip to stabilize.
d- If malposition of the femoral is sever, brace used to prevent recurrence.
e- If malposition of acetabular component is severe, augmentation of the socket
may be needed.
182- Aseptic loosening after total hip arthroplasty
a- Is the third cause of long-term failure.
b- With modern methods of implant fixation, radiographic evidence of loosening
in less than 2 per cent of patients 15 years after operation.
c- Radionuclide scanning shows decreased activity.
d- At microscopic level, symptomatic patients only show cellular reaction and
membrane formation at the bone–cement interface.
e- Revision arthroplasty can be either cemented or uncemented, depending on the
condition of the bone.
183- Highly cross-linked polyethylene (XLPE) acetabular prostheses
a- Gamma irradiation of polyethylene causes cross-linking, which greatly
improves the wear resistance.
b- Gamma irradiation of polyethylene reduce the price of prostheses.
c- Gamma irradiation of polyethylene is directly proportional to the fracture
toughness.
d- Encouraging clinical results with markedly increase wear reported with XLPE.
e- It should be noted that the commercially available XLPEs are the same.
184- Metal-on-metal bearing surfaces
a- Have very high wear rates
b- Are self-polishing, which allows for self-healing of surface scratches.
c- Metal is not brittle, and components therefore to be as thick.
d- Gives a smaller range of motion, and thus lesser mobility and greater stability.
e- Should not be used for patients want to return to vigorous recreational
activities.
185- The post-operative care of total hip arthroplasty
a- The length of inpatient stay reduced to 14–16 days in most hospitals.
b- Patients mobilized independently before discharge.
c- Car driving allowed 14 day.
d- Patients will have negotiated stairs independently 2 months.
e- Full weight bearing without support will usually take 6–8 weeks at the
patient’s own pace.
186- A small, localized swelling on the anterolateral side of the knee joint
a- Makes one think of haemarthrosis.
b- Makes one think of knee effusion.
c- Makes one think of prepatellar bursa.
d- Makes one think of a cyst of the meniscus.
e- Makes one think of semimembranosus bursa.
187- The Q-angle (quadriceps angle) is
a- The angle subtended by a line drawn from the anterior inferior iliac spine to
the tip of the patella and another from the tip of the patella to the tibial
tubercle.
b- The angle subtended by a line drawn from the anterior inferior iliac spine to
the lower pole of the patella and another from the lower pole the patella to the
tibial tubercle.
c- An increased Q-angle regarded as a predisposing factor in the development of
chondromalacia.
d- Normally averages about 4 degrees in men.
e- Normally average about 7 degrees in women.
188- The anterior cruciate ligament stability
a- The ‘sag sign’ is sensitive tests.
b- Anterior drawer test is sensitive.
c- Posterior drawer sign is sensitive.
d- Lachman test is sensitive.
e- Lachman test is sensitive and specific.
189- A unilateral genu varus
a- Mostly physiological.
b- Is rarely to be pathological.
c- Mostly congenital.
d- It is essential in all cases to look for signs of injury.
e- If angulation is severe, early operative correction is necessary.
190- X-ray of Blount's disease
a- The proximal tibial epiphysis flattened laterally and the adjacent metaphysis is
beak-shaped.
b- The lateral cortex of the proximal tibia appears thickened.
c- There is internal rotation of the tibia.
d- The tibial epiphysis always look 'fragmented’; and the femoral epiphysis also
is affected.
e- In the late stages, a bony bar forms across the medial half of the tibial physis.
191- Genu valgus in adult female results
a- May be secondary to rheumatoid arthritis.
b- May be secondary to osteoarthritis.
c- May be secondary to Paget's disease.
d- Corrected by varus high tibial osteotomy.
e- Stress x-rays are not essential in the assessment of these cases.
192- ‘Locking’ of the knee– that is, the sudden inability to extend the knee fully
suggest
a- Anterior horn tear.
b- Posterior horn tear.
c- Horizontal tear.
d- Bucket handle tear.
e- Degeneration of the menisci.
193- Operative treatment of meniscus injuries
a- Indicated if the joint locked.
b- Indicated if symptoms are acute.
c- Tears close to the periphery, treated by meniscectomy.
d- In appropriate cases, the success rate for both open and arthroscopic repair is
almost 60 per cent.
e- Total meniscectomy thought to cause more instability and so predispose to late
secondary osteoarthritis.
194- Discoid lateral meniscus
a- A young patient complains of gives way and ‘thuds’ loudly with history of
injury.
b- A characteristic clunk felt at 60 degrees flexion and at 30 degrees as
straightened.
c- MRI cannot confirms the diagnosis.
d- If there is only a clunk, treatment is essential.
e- If pain is disturbing, arthroscopic partial excision leaving a normally shaped
meniscus'.
195- The treatment of meniscal cyst.
a- Arthroscopic total excision of cyst and total meniscectomy.
b- Arthroscopic removal of damaged part pf meniscus and decompression of cyst
within the joint.
c- Arthroscopic total excision of cyst and partial meniscectomy.
d- Open total excision of cyst and total meniscectomy.
e- Open total excision of cyst and partial meniscectomy.
196- The indication of urgent surgical treatment in recurrent dislocation of the
patella
a- Tear of medial capsule
b- Multiple dislocation in knee flexion.
c- Presence of a large displaced osteochondral fracture.
d- Recurrent dislocation of patella with severe pain.
e- Unstable patella after reduction.
197- Patellofemoral disorders that cause anterior knee pain.
a- Patellar instability
b- Osteochondritis dissecans
c- Loose body in the joint
d- Synovial chondromatosis
e- Plica syndrome
198- The common knee joint disorders that cause anterior knee pain. .
a- Patellar instability.
b- Patello-femoral overload.
c- Patellar ligament strain
d- Synovial chondromatosis
e- Plica syndrome
199- Osteochondritis dissecans of the knee is
a- Females affected more often than males.
b- Bilateral in 50 per cent of cases.
c- Over 80 per cent of lesions occur on the medial part of the lateral femoral
condyle.
d- A large, well-demarcated, vascular fragment of bone and overlying cartilage
separates from the lateral femoral condyles.
e- The lesion better seen in the ‘tunnel view’.
200- Treatment of knee loss body
a- A loose body should be removed.
b- Finding the loose body may be difficult; it may be concealed in a synovial
pouch or sulcus.
c- A loose body should be removed even the joint is severely osteoarthritic.
d- This usually done through the open arthrotomy.
e- A large loose body may even slip under the edge of one of the menisci.
201- Synovial chondromatosis
a- Is a common disorder in which the joint comes to contain multiple loose
bodies.
b- The usual explanation is that synovium undergo cartilage neoplasia and may
ossify.
c- X-rays reveal few loose bodies.
d- Arthrography shows multiple ossified loose bodies.
e- The loose bodies removed arthroscopically; an attempt should be made to
remove all abnormal synovium.
202- The Plica syndrome of knee
a- An adult complains of an ache in the side of the knee (occasionally both
knees).
b- Characteristic feature is tenderness near the upper pole of the patella and over
the femoral condyle.
c- There is no history of trauma or markedly increased activity.
d- Exercise or climbing stairs relieve symptoms.
e- Movement of the knee may cause loud painful click in joint line.
203- Rheumatoid arthritis of knee distinguished from osteoarthritis by
a- Diminution of the joint space,
b- Osteopenia.
c- Marginal erosions.
d- The complete absence of osteophytes.
e- Subchondral cystic changes.
204- If only the patello-femoral joint is affected, suspect
a- Gout.
b- Pyrophosphate arthropathy.
c- Osteoarthritis.
d- Reiter's disease.
e- Rheumatoid arthritis.
205- Charcot’s disease (neuropathic arthritis) of knee treated by
a- Moulded splint.
b- Arthroscopic debridement.
c- Open debridement and synovectomy.
d- Arthrodesis.
e- Arthroplasty.
206- Rupture of extensor mechanism above patella
a- Patient is usually young.
b- Patient had history of long steroid treatment.
c- There is diffuse swelling of the anterior part of thigh.
d- Function of quadriceps muscle usually bad.
e- Patient need early surgical repair.
207- Osgood Schlatter disease
a- Is uncommon disease of adolescent.
b- Always there is history of trauma.
c- Is usually unilateral.
d- Patients is a young adult complain of pain without activity and of a lump.
e- Spontaneous recovery is usual.
208- Pellegrini-Stieda disease
a- X-ray shows osteolytic lesion on lateral condyle.
b- X-ray shows osteolytic lesion on medial condyle.
c- X-rays show a plaque of bone lying next to the femoral condyle under the
lateral collateral ligament.
d- X-rays show a plaque of bone lying next to the femoral condyle under the
medial collateral ligament.
e- Occasionally this is a source of pain on the lateral side of knee.
209- In chronic knee swelling, the most important condition to exclude is
a- Pseudogout.
b- Tuberculosis.
c- Pigmented Villonodular synovitis.
d- Rheumatoid arthritis.
e- Synovial chondromatosis.
210- Prepatellar bursitis
a- There is hard swelling confined to the front of the patella.
b- The knee joint itself is abnormal.
c- This is an infected bursitis due to constant friction between skin and bone.
d- Seen mainly in carpet layers, floor cleaners and miners who use protective
kneepads.
e- In chronic cases, the best treatment is lump excision.
211- The semimembranosus bursa
a- It presents usually as a painful lump behind the knee, slightly to the medial
side of the midline.
b- Is most conspicuous with the knee flexed.
c- The lump is fluctuant and the fluid can pushed into the joint.
d- The knee joint is normal.
e- The best treatment is excision through a transverse incision.
212- The knee varus osteotomy
a- Is required for active patients with isolated medial compartment disease.
b- This performed at the upper proximal part of tibia.
c- The method most commonly employed is a medial opening wedge osteotomy,
d- The fragments should firmly fixed with a blade-plate.
e- In many cases postoperative cast immobilization not needed.
213- The most important early complication of tibial osteotomy is
a- Compartment syndrome in the leg.
b- Peroneal nerve palsy.
c- Failure to correct the deformity.
d- Delayed union and non-union.
e- Mechanical failure of internal fixation.
214- Contraindications to knee arthrodesis
a- Failed knee replacement.
b- Problems in the ipsilateral hip or ankle.
c- Osteoarthritis of contralateral knee.
d- Rheumatoid arthritis.
e- Limited peri-articular bone loss.
215- Unicompartmental knee replacement
a- Has firmly established.
b- Early results for medial compartment osteoarthritis were excellent.
c- Longer-term studies have highlighted the need to avoid low revision rates.
d- Following a successful operation, restoration of function is not impressive.
e- Is reserved for older patients.
216- Patellar resurfacing.
a- A kind of partial replacement performed in osteoarthritis.
b- A kind of partial replacement rheumatoid arthritis.
c- A kind of partial replacement rarely performed alone.
d- A kind of partial replacement, performed alone.
e- Used in treatment of chondromalacia patella.
217- Infection in total knee replacement
a- Prevention is the most important.
b- For established infection treated by antibiotics.
c- The safest salvage procedure is by exchange replacement in one stage.
d- The safest salvage procedure is by exchange replacement in two stage.
e- Intractable infection treated by debridement.
218- Meniscus are prone to injury
a- Particularly during unguarded movements of extension and rotation on the
weight bearing leg.
b- Particularly during unguarded movements of flexion and rotation on the
weight bearing leg.
c- Particularly during unguarded movements of hyperextension and rotation on
the weight bearing leg.
d- In maximum flexion of knee.
e- In maximum internal rotation.
219- Lateral collateral ligaments
a- Attached to the lateral meniscus.
b- Situated more anteriorly.
c- Blend with capsule of knee.
d- Separated from lateral collateral by popliteus tendon.
e- Is fan shape.
220- Forward subluxation of lateral tibial condyles is prevented by
a- Lateral collateral ligament.
b- Posterolateral capsule.
c- Posterior cruciate ligament.
d- Posterior cruciate ligament and arcuate ligament.
e- Anterior cruciate ligament.
221- The tibia sublaxated forward when
a- Anterior cruciate ligament and medial collateral ligament.
b- Anterior cruciate ligament and posterior cruciate ligament.
c- Posterior cruciate ligament
d- Anterior cruciate ligament.
e- Medial collateral ligament.
222- Backward subluxation of the tibia is prevented by
a- The anterior cruciate ligament.
b- The posterior cruciate ligament.
c- The posterior cruciate ligament with the arcuate ligament and the posterior
oblique ligament.
d- Anterior cruciate ligament and medial collateral ligament.
e- The posterior cruciate ligament and lateral collateral ligament.
223- The gait may be disturbed by
a- Pain.
b- Muscles weakness.
c- Deformity.
d- Stiffness.
e- All of above.
224- Pain and tenderness posterior to medial malleolus
a- Fracture of medial malleolus.
b- Tarsal tunnel syndrome.
c- Tibialis posterior tendinitis.
d- Impingement from osteophyte.
e- Achilles Para tendonitis.
225- The last deformity to be corrected in conservative treatment of idiopathic club
foot
a- Fore foot adduction.
b- Forefoot supination.
c- Forefoot varus.
d- Hindfoot equinus.
e- Hindfoot varus.
226- Metatarsus adductus
a- There is varus of hindfoot.
b- There is equinus of hindfoot.
c- Deformity is limited to the forefoot.
d- Had classical pattern of severity.
e- Most of cases need surgical treatment.
227- Congenital vertical talus
a- Passive correction is impossible .
b- The hindfoot is in calcaneus and valgus.
c- The talus points almost horizontally towards the sole.
d- The forefoot is abducted, supinated and dorsiflexed.
e- The tendons and ligaments on the dorsolateral side of the foot are usually
lengthened.
228- Peroneal spastic flatfoot
a- Young children sometimes present with a painful, rigid flatfoot.
b- Flexor tendons are in spasm.
c- X-rays show typical talonavicular coalitions.
d- Pain may be due to abnormal tarsal stress or even fracture of an ossified bar.
e- The picture differs from that of the more common ‘idiopathic’ flatfoot by the
small concave foot.
229- Flexible flatfoot in young children
a- Required no treatment.
b- Treated by stretching and plaster splint.
c- Treated by orthotic splintage.
d- Treated by physiotherapy.
e- Treated by insole and moulded heel-cup.
230- Accessory navicular
a- Associated with rigid flatfoot.
b- Complain of tenderness on medial border of the midfoot.
c- Symptoms are due to bone.
d- Treated by surgical removal.
e- Treated by below knee orthosis and insole.
231- Acquired painful flatfoot in adult commonly due to
a- Ligament laxity.
b- Tarsal coalition.
c- Tibialis posterior dysfunction.
d- Neuropathy.
e- Degenerative arthritis.
232- Surgical treatment of painful flatfoot include
a- Reconstruction of tendon Achilles.
b- Tenosynovectomy of peroneal tendons.
c- Pantalar arthrodesis.
d- Ankle arthrodesis.
e- Triple arthrodesis.
233- Hallux valgus
a- Is one of the common foot deformity.
b- Result from valgus angulation of the first metatarsal bone.
c- Is uncommon in rheumatoid arthritis.
d- There is excessive lateral angulation of big toe.
e- Positive family history obtained in 30% of cases.
234- When hallux valgus exceed 40 degrees
a- The great toe rotate in supination.
b- The great toe rotates in pronation.
c- The sesamoid bone displaced medially.
d- The extensor tendon stretched.
e- The intact adductor halluces prevent progress of deformity.
235- The hallux valgus in elderly is best treated by
a- Shoes modification.
b- Arthrodesis.
c- Arthroplasty.
d- Distal osteotomy.
e- Proximal osteotomy.
236- In adolescent with hallux valgus less than 25 degrees treated by
a- Bunionectomy.
b- A soft tissue rebalancing operation.
c- A distal osteotomy combined with a corrective osteotomy of the base of the
proximal phalanx.
d- Keller’s operation.
e- Arthrodesis.
237- Diagnostic feature of hallux rigidus
a- A pain on walking, especially on slopes.
b- The patient develop altered gait.
c- The great toe is straight with callosity.
d- The MTP joint feel knobby with tender dorsal bunion.
e- The MTP joint dorsiflexion is restricted and painful.
238- Hammer toe characterized by
a- Hyperextension at the MTP joint and flexion of both IP joint
b- Acute flexion deformity of proximal IP joint only and hyperextension of MTP
joint.
c- Flexion of the distal IP joint and extension of proximal IP joint.
d- The MTP joint is dislocated and the little toe sits on the dorsum of the
metatarsal head.
e- An irritating or painful bunionette may form over an abnormally prominent
fifth metatarsal head.
239- A 40 years old patients with rheumatoid arthritis suddenly develop a painful
valgus foot, the most probable cause is
a- Midtarsal subluxation.
b- Subtalar arthritis.
c- Rupture of peroneus longus tendon.
d- Rupture of tibialis posterior tendon.
e- Rupture of planter fascia.
240- The common site for osteochondritis of the ankle is
a- Anterolateral part of the articular part of talus.
b- Anteromedial part of the articular part of talus.
c- Posteromedial part of the articular part of talus.
d- Posterolateral part of the articular part of talus.
e- Central part of the articular part of talus.
241- The atraumatic osteonecrosis of talus involving
a- Posterolateral part of the talar dome.
b- Posteromedial part of the talar dome.
c- Anterolateral part of the talar dome.
d- Anteromedial part of the talar dome.
e- Central part of the talar dome.
242- Insufficiency fracture in diabetic foot should be treated
a- By prolonged cast.
b- Without immobilization.
c- By internal fixation.
d- By internal fixation with bone cement.
e- By external fixation.
243- The foot rotates about an axis running through
a- Fifth metatarsal.
b- Fourth metatarsal.
c- Third metatarsal.
d- Second metatarsal.
e- First metatarsal.
244- The plantar fascia
a- Is a dense fibrous structure that originates from the calcaneum, superficial to
the heel fat pad.
b- Runs distally to the dome of the foot, with slips to each toe distal phalanx.
c- The plantar fascia stiffens and becomes more pliable with age.
d- There may be micro-tears in the fascia, and the fascia thickens.
e- The condition is not associated with gout, ankylosing spondylitis and Reiter’s
disease.
245- Painful fat pad
a- Acute pain and tenderness directly over the fat pad under the heel.
b- Sometimes follows a direct blow to the area, e.g. in a fall from a height.
c- The condition seen in old patients and has been attributed variously to
separation of the fat pad from the bone.
d- Chronic specific ‘inflammation’ has been blamed.
e- Treatment is surgical by debridement of necrotic and inflamed tissue.
246- Heel nerve entrapment
a- Entrapment of the second branch of the lateral plantar nerve has been reported
as a cause of heel pain.
b- The commonest complaint is pain and numbness at rest.
c- Characteristically, tenderness is maximal on the medial aspect of the heel.
d- Diagnosis is easy, because the symptoms and signs differ from plantar
fasciitis.
f- Treatment should be surgical decompression of the nerve.
247- Kohler's disease
a- Is common cause of pain in the midtarsal region in children.
b- The bony nucleus of the medial cuneiform becomes dense and fragmented.
c- The child, over the age of 10 year, has a painful limp.
d- On examination, a tender warm thickening over navicular bone.
e- If symptoms are severe, a surgical decompression of bone helps.
248- Metatarsalgia
a- Is a common expression of foot strain.
b- It result from bone osteopenia.
c- Result from foot neuropathy.
d- Aching felt under the forefoot and the foot arch may have flattened out.
e- There may even be callosities over IP joint of toes.
249- Brailsford’s disease
a- A ridge of bone develops on the dorsal surfaces of the medial cuneiform.
b- The navicular becomes dense, then altered in shape, and later the midtarsal
joint may degenerate.
c- In children, especially if the arch is high, the overbone develop.
d- A lump behind heel, which feels bony and may become bigger and tender if
the shoe presses on it.
e- Surgical removal of heel lump provide relief of the symptom.
250- Sesamoiditis
a- Is part of rheumatoid arthritis manifestation of foot.
b- May be initiated by direct trauma or unaccustomed stress.
c- Acute sesamoid pain and tenderness should signal the possibility of sesamoid
displacement.
d- Sudden pain may result from local infection (particularly in a diabetic patient).
e- Acute pain result from avascular necrosis.
251- Sesamoid chondromalacia
a- Is a term coined by Apley at 1966.
b- Used to explain changes such as fragmentation and cartilage fibrillation of the
lateral sesamoid.
c- X-rays show a sclerosis medial sesamoid.
d- Is often mistaken for a gout.
e- Treated by application of cast for 3 months.
252- Freiberg’s disease
a- Osteochondritis of first metatarsal head in young children.
b- Is probably a type of atraumatic osteonecrosis of the subarticular bone.
c- It usually affects the second metatarsal head (rarely the third) in young adults,
mostly women.
d- The patient complains of pain at the IP joint.
e- A bony lump is palpable and tender at the MTP joint of big toe.
253- Stress fracture
a- Usually of the first metatarsal, occurs in young adults after unaccustomed
activity.
b- Usually of the first metatarsal, occurs in in women with postmenopausal
osteoporosis.
c- The sole of the foot may be edematous and the affected shaft tender.
d- The x-ray appearance is at first normal, but later shows fusiform callus around
a fine transverse fracture.
e- Long before x-ray signs appear, a radioisotope scan will show decreased
activity.
254- The ABC system for resuscitation of sock with catastrophic external bleeding
a- A for airway is the first.
b- B for breathing is second step.
c- The C for circulation is the third
d- Control of the external bleeding takes precedence.
e- Follow the ABC sequence.
255- The majority of patients presenting with shock following a major injury will
be suffering from
a- Hypovolemic shock.
b- Septic shock.
c- Neurogenic shock.
d- Anaphylactic shock.
e- Cardiogenic shock.
256- The systolic blood pressure may not drop significantly
a- Until 10 per cent of the patient’s blood volume has been lost.
b- Until 15 per cent of the patient’s blood volume has been lost.
c- Until 20 per cent of the patient’s blood volume has been lost.
d- Until 25 per cent of the patient’s blood volume has been lost.
e- Until 30 per cent of the patient’s blood volume has been lost.
257- Fracture of the pelvis
a- Can result in devastating retroperitoneal hemorrhage.
b- Bleeding cannot reduced by compressing the pelvis to approximate the
bleeding fracture sites.
c- Compression to reduce hemorrhage cannot achieved manually with a towel or
blanket.
d- Compression by external fixation of the pelvis is useless.
e- MAST trousers are practicable and commonly used.
258- High-energy (velocity) fractures
a- Cause only moderate soft-tissue damage.
b- Cause severe soft-tissue damage, no matter whether the fracture is open or
closed.
c- There is little or no displacement.
d- The displacement does not matter initially.
e- The reduction is unlikely to succeed.
259- Open reduction
a- Is the first step to internal fixation.
b- Used for most fractures in children
c- For fractures that are stable after reduction
d- Can held in some form of splint or cast.
e- Avoids direct manipulation of the fracture site.
260- Contraindications to nonoperative methods of fracture treatment is
a- Fracture of long bones.
b- Inherently unstable fractures.
c- Rotated fractures.
d- Fracture in metaphyseal region.
e- Supracondylar fractures of lower humerus.
261- Soft tissue edema following fracture can be treated by
a- Elevation of limb.
b- Firm support.
c- Elevation and firm support.
d- Exercise.
e- Coban wrap around a limb to control swelling during treatment.
262- The incidence of wound infection in open fractures correlates directly with
a- The type of antibiotics used.
b- Duration of injury.
c- The extent of soft-tissue damage.
d- The site of injury in the limbs.
e- The type and quality of treatment in open fracture.
263- Antibiotics at debridement for open fractures grade II is
a- Gentamicin and vancomycin.
b- Co-amoxiclav.
c- Penicillin and gentamicin.
d- Penicillin and cefuroxime.
e- Gentamicin and clindamycin.
264- In wound debridement viable muscle can be recognized by
a- Its purplish colour.
b- Its mushy consistency.
c- Its failure to contract when stimulated.
d- Its failure to bleed when cut.
e- Its tone preserved.
265- To irrigate open fracture grade II, use
a- 1- 2 liters of normal saline.
b- 2- 4 liters of normal saline.
c- 3- 6 liters of normal saline.
d- 6- 12 liters of normal saline.
e- 12- 24 liters of normal saline.
266- If wound cover is delayed in open fracture
a- The external fixation is safer.
b- The skeletal traction is safer.
c- The back slab splint is safer.
d- The non-reamed intramedullary nail is safer.
e- The minimal contact plate is safer.
267- Early infection of open fracture presented as
a- Discharging wound.
b- Inflamed wound with discharge.
c- Inflamed wound without discharge.
d- Black discoloration of wound surface.
e- Red and swollen tissue with yellowish slough.
268- Gunshot injuries are contaminated by
a- Metallic forging body.
b- Necrotic tissue.
c- Derbies sucked into wound. :[Comment [h2

d- Tract of bullet.
e- Hematoma inside wound.
269- The common nerve injury in Monteggia fracture dislocation
a- Median nerve.
b- Radial nerve.
c- Ulnar nerve.
d- Anterior interosseous.
e- Posterior interosseous.
270- If vascular repair undertaken in close fracture
a- The fracture should reduce and hold by POP. :[Comment [h3

b- The fracture should reduce and hold by traction.


c- The fracture should reduce and hold by internal fixation.
d- The fracture should reduce and hold by external fixation.
e- The fracture should reduce and hold by cast brace.
271- The early symptom of compartment syndrome is
a- Sever pain.
b- Paresthesia.
c- Pallor,
d- Paralysis.
e- Pulslessness.
272- The earliest sings of compartment in upper limb
a- Pallor of finger.
b- Painful dorsiflexion of finger.
c- Anesthesia.
d- Pulslessness.
e- Paralysis.
273- Symptomatic hypertrophic nonunion treated by
a- Bone graft.
b- Bone graft and rigid internal fixation.
c- Rigid internal fixation.
d- Low frequency pulsed ultrasound with cast brace.
e- Pulsed electromagnetic field and cast brace.
274- Hypertrophic non-union –treatment by the Ilizarov technique
a- Treated by compression.
b- Treated by compression and realignment in external fixator.
c- Treated by bone transposition in external fixator.
d- Treated by gradual distraction and realignment in an external fixator.
e- Treated by rigid external fixation and bone grafting.
275- Early treatment of myositis ossificans
a- Muscles stretching exercise.
b- Splintage in position of rest followed by active exercise.
c- Splintage in position of function followed active exercise.
d- Manipulation under anesthesia followed by passive exercise.
e- Manipulation under anesthesia followed by active exercise.
276- In rupture of extensor pollicis longus tendon, all true except
a- May occur 2–4 weeks after a fracture of the lower radius.
b- Cause mallet index.
c- Follow displaced lower radius fracture.
d- Direct suture is possible.
e- Treated by transferring the extensor indicis proprius tendon to the distal stump
of the ruptured tendon.
277- The common cause of joint stiffness are , except
a- Injuries of articular.
b- Injuries of synovial membrane and capsule.
c- Haemarthrosis of joint lead to synovial adhesion.
d- Edema and fibrosis of capsule and muscles.
e- Complex regional pain syndrome.
278- Characteristic x-ray feature of complex regional pain syndrome
a- Generalized reduction in bone density.
b- Localized increase in bone density.
c- Patchy rarefaction in the affected part.
d- Regional osteoporosis of affected part.
e- Patchy osteosclerosis in affected part.
279- Localized disease cause pathological fracture are
a- Osteoporosis.
b- Osteomalacia.
c- Paget's disease.
d- Infection.
e- Myelomatosis.
280- In children, the physeal injuries forms
a- Five% of children fractures.
b- Ten % of children fractures.
c- 15 % of children fractures.
d- 20 % of children fractures.
e- 25 % of children fractures.
281- Middle-aged men with pathological fracture, may result from
a- Severe osteoporosis.
b- Osteomalacia.
c- Skeletal metastases or myeloma.
d- Paget's disease.
e- Hyperparathyroidism.
282- Secondary metastases in femur mostly result from
a- Kidney tumor.
b- Breast carcinoma.
c- Bronchogenic carcinoma.
d- Prostate carcinoma.
e- Thyroid carcinoma.
283- Secondary metastases fracture near a bone end can often be treated by
a- Internal fixation.
b- Internal fixation and bone graft.
c- Prophylactic internal fixation and arthrodesis.
d- Excision and prosthetic replacement; this is especially true of femoral neck
fractures.
e- Internal fixation; if necessary the site packed with acrylic cement.
284- Femoral fracture in Paget's disease treated
a- Systemic medical treatment for Paget's disease.
b- Internal fixation is almost essential.
c- Custom made prosthesis.
d- Bone cement with plate and screws.
e- Bisphosphonate, calcium, Vit D, fluoride and external fixation.
285- Battered baby syndrome
a- The history fit with injuries.
b- The fractures are pathological
c- The fracture caused by accident.
d- There is only fractures.
e- The fractures at different stage of healing.
286- Types 5 and 6 epiphyseal fractures
a- If properly reduced, have an excellent prognosis.
b- Bone growth is not adversely affected.
c- The size and position of the bony bridge across the physis assessed by x-ray.
d- Complications such as malunion or non-union may also occur
e- If the bridge is relatively small, it excised and replaced by a fat graft.
287- sprain is
a- Ligaments tear.
b- Any painful wrenching (twisting or pulling) movement of a joint.
c- Associated with articular cartilage damage.
d- Compression of articular surfaces.
e- Associate with physis fracture separation in children.
288- Displaced lateral third fractures of clavicle
a- Are stable injuries.
b- Have a lower than usual rate of non-union if treated non-operatively.
c- Surgery to stabilize the fracture is rarely recommended.
d- Operations for these fractures have a high complication rate.
e- The best surgical treatment is intramedullary fixation.
289- The incidence of nonunion in clavicle is higher in
a- Displaced middle third fracture.
b- Comminuted middle third fracture.
c- Lateral part fracture.
d- Medial part fracture.
e- Comminuted medial part.
290- Malunion of clavicle with shortening of more than 2 cm
a- Is rare.
b- Do not produce symptoms.
c- Some may go on to develop periscapular pain.
d- Treated by physiotherapy.
e- Operative treatment not indicated.
291- The finding arose suspicion of scapulothoracic dissociation is
a- The scapula exposed to indirect trauma.
b- The limb abducted end externally rotated.
c- The diagnosis depend on CT scan finding.
d- There is swelling below the scapula.
e- A distraction of more than 1 cm of a fractured clavicle in x-ray.
292- Posterior sternoclavicular dislocation
a- Is common and less serious.
b- There is mild discomfort.
c- There may be pressure on the trachea or large vessels.
d- Reduction is not necessary.
e- Open reduction is not justified.
293- After reduction of anterior dislocation of shoulder, the arm is rested in a sling
for
a- About one week in those under 30 years of age.
b- About two weeks in those under 30 years of age.
c- For only three weeks in those over 30.
d- For only two weeks in those over 30.
e- For only one week in those over 30.
294- The axillary nerve injury after anterior dislocation of the shoulder
a- Is uncommon injury.
b- The patient is able to contract the deltoid muscle and there may be a large
patch of anesthesia over the muscle.
c- The inability to abduct must be distinguished from a rotator cuff tear.
d- The nerve lesion is usually a neurotemesis, which recovers after a few months.
e- The results of surgical repair are satisfactory if the delay is less than a few
months.
295- To reduce incidence of recurrence in anterior dislocation of shoulder
a- The use of external rotation splints.
b- The use of immobilization.
c- Continue their sports (particularly contact sports).
d- Arthroscopic anterior stabilization surgery after early detection of Bankart's
e- The value of early surgery had been confirmed.
296- Postural downward displacement of the humerus
a- It is similar to true inferior dislocation.
b- The condition is harmful.
c- Not resolves as muscle tone regained.
d- May results quite commonly from tear of ligaments and following laxity of the
muscles around the shoulder.
e- Occur after trauma and shoulder splintage.
297- The Neer classification of proximal neck fracture based on x-ray appearances
a- Fragment displacement defined as greater than 25 degrees of angulation or 0.5
cm of separation.
b- However many fracture lines there are, if the fragments are undisplaced it is
regarded as a one-part fracture;
c- If one segment is separated from the others, it is a one-part fracture;
d- If two fragments are displaced, that is a two-part fracture;
e- The observers do usually agree with each other on which class a particular
fracture falls into.
298- Four part fracture of proximal humerus
a- The both tuberosities displaced.
b- These are severe injuries with some risk of complications.
c- In older patients, open reduction and fixation is advisable.
d- In young patients, an attempt should be made at closed treatment.
e- The results of hemiarthroplasty are unpredictable.
299- Fractured shaft of humerus
a- Bruising is always extensive.
b- Closed transverse fracture treated by internal fixation.
c- Ready-made braces are usually not adequate in moderate displacement.
d- The conservative methods is suitable for all cases.
e- The complication rate after internal fixation of the humerus is rare.
300- Fractured shaft of humerus is well to remember
a- The complication rate after internal fixation of the humerus is high
b- The great majority of humeral fractures need operative treatment.
c- There is good evidence that the union rate is higher with fixation.
d- The rate of union may be better if there is distraction with nailing
e- The rate of union may be better if there is periosteal stripping with plating.
301- Holstein–Lewis fracture.
a- Fracture of proximal part of humerus.
b- Displaced fracture of proximal humerus in children.
c- Displaced transverse fracture of humeral shaft.
d- Oblique fractures at the junction of the middle and distal thirds of the bone.
e- Transverse fracture in lower third of humerus.
302- Fracture of the distal humerus in adult
a- Are often low-energy injuries.
b- May associated with vascular and nerve damage.
c- Most of injuries can be treated conservatively.
d- Rarely need complex surgical techniques.
e- There is low tendency to stiffness of the elbow.
303- Fracture capitulum
a- Is a rare articular fracture, which occurs in any age.
b- The patient falls on the hand, usually with the elbow semiflex.
c- The anterior part of the capitulum sheared off and displaced proximally.
d- Fullness behind the elbow is the most notable feature.
e- In the lateral view, the capitulum seen in front of the coronoid process.
304- Combined fractures of the radial head and coronoid process plus dislocation of
the elbow
a- Is associated with rupture of the medial collateral ligament.
b- Is associated with rupture of the interosseous membrane
c- Is Essex Lopresti lesion.
d- Is the terrible triad.
e- Excision of the radial head is indicated.
305- Side-swipe injuries
a- These severe fracture-dislocations of elbow
b- Are rarely associated with damage to the nerves.
c- The priorities are skeletal stabilization by cast.
d- The injuries stabilized by K- wires.
e- Surgery should done early in emergency theater.
306- Stiffness after dislocation of the elbow
a- Loss of 20 to 30 degrees of extension is common.
b- The most common cause of undue stiffness is prolonged immobilization.
c- The joint should be moved as soon as possible by passive stretching.
d- Persistent stiffness of severe degree can often be improved by arthroplasty.
e- Sometimes the stiffness is due to osteoarthritis.
307- Isolated dislocation of the radial head
a- Is uncommon.
b- Search carefully for an associated fracture of the capitulum.
c- In adult, the ulnar fracture may be difficult to detect.
d- Green-stick or mild plastic deformation of the radial shaft may be missed.
e- Bended ulnar bone may prevent full reduction of the radial head dislocation.
308- The average ages at which the ossific centres appear
a- Capitulum – 1 years.
b- Radial head – 3 years.
c- Medial epicondyle – 6 years.
d- Trochlea – 10 years.
e- Olecranon – 12 years.
309- The fat pad sign of elbow
a- Is seen most clearly in the anteroposterior view.
b- Seen in displaced supracondylar fracture.
c- Is diagnostic of undisplaced supracondylar fracture.
d- Arose suspicions undisplaced supracondylar fracture.
e- Is a triangular lucency behind the distal humerus, due to the fat pad being
pushed backwards by a hematoma.
310- In the anteriorly displaced supracondylar fracture the
a- The fracture line runs downwards and backwards.
b- Fracture line runs obliquely downwards and forwards.
c- The distal fragment tilted backwards.
d- The distal fragment shifted backwards.
e- The proximal fragment tilted forwards.
311- The incidence of vascular injuries in the displaced supracondylar fractures
a- Is probably less than one per thousand.
b- Is probably less than 1 percent.
c- Is probably less than 5 percent.
d- Is probably less than 8 percent.
e- Is probably less than 10 percent.
312- Ischemia following supracondylar fracture suggested by
a- Pain and reduced capillary return on pressing the finger pulp.
b- Pain and blunted sensation.
c- Undue pain and pain on passive extension of the fingers.
d- Pain and a tense and tender forearm.
e- Pain and an absent pulse.
313- Fractured lateral condyle
a- A small fragment of bone and cartilage avulsed.
b- Even with reasonable reduction, malunion not inevitable.
c- Closed reduction with casting is often wise.
d- If left unreduced non-union is inevitable.
e- A varus deformity of the elbow with delayed ulnar palsy the likely sequel. -
314- Pulled elbow
a- Is a subluxation of the orbicular ligament, which slips up over the head of the
radius.
b- A child aged 5-8 years brought with a painful, dangling arm.
c- The forearm held in supination and extension, and any attempt to flex it is
resisted.
d- The x-ray shows subluxation of the radial head.
e- A dramatic cure is achieved by forcefully flexing the elbow; the ligament slips
back with a snap.
315- Fractures of radius and ulna in adults
a- Displaced fractures treated by closed reduction and cast for 4 weeks.
b- The comminuted type held by intramedullary fixation.
c- Bone grafting is not advisable if there is comminution.
d- If the interosseous membrane is severely damaged, plating prevent cross-
union.
e- The deep fascia left open to prevent a build-up of pressure in the muscle
compartments, and only the skin is sutured.
316- Open fractures of the forearm
a- In late presentation antibiotics and tetanus prophylaxis; the wounds are
washed.
b- The wounds are excised and extended and the bone ends are exposed and
thoroughly cleaned.
c- Are primarily fixed with intramedullary nails.
d- If bone grafting is necessary, it should be done early in treatment.
e- If there is major soft-tissue loss, the bones are better stabilized K- wires.
317- Removal of plates and screws from radius and ulna
a- Regarded as a completely innocuous procedure.
b- Complications are uncommon.
c- The damage to vessels and nerves are not expected
d- Infection is extremely rare.
e- Postoperative fracture through a screw-hole may occur.
318- ‘Nightstick fracture’ is
a- Is fracture of the radius alone.
b- Fracture of radius with wrist subluxation.
c- Direct fracture of the ulna alone.
d- Fracture of ulna and proximal radioulnar subluxation.
e- Fracture of both the radius and ulna with tear of interosseous membrane.
319- Isolated fracture of the radius
a- Are prone to rotary displacement.
b- To achieve reduction in children the forearm needs to be pronated for upper
third fractures.
c- To achieve reduction in adult the forearm needs to be supinated for middle
third fractures
d- To achieve reduction in children the forearm needs to be supinated for lower
third fractures.
e- Internal fixation with an intramedullary nail and screws in adults.
320- Treatment of Monteggia fracture dislocation of ulna in adult
a- By closed reduction and cast splintage for 8 weeks.
b- By open reduction and intramedullary fixation.
c- By open reduction through an anterior approach, the ulnar fracture accurately
reduced, with the bone restored to full length, and then fixed with a plate and
screws; bone grafts may be added for safety.
d- By open reduction through a posterior approach, the ulnar fracture accurately
reduced, with the bone restored to full length, and then fixed with a plate and
screws; bone grafts may be added for safety.
e- The radial head reduced always after open reduction.
321- The Galeazzi fracture
a- Is much less common than the Monteggia.
b- Prominence or tenderness over the lower end of the radius is the striking
feature.
c- It may be possible to demonstrate the instability of the radio-ulnar joint by
‘ballotting’ the distal end of the ulna (the ‘piano-key sign’).
d- It is important also to test for a radial nerve lesion, which may occur.
e- X-ray a transverse or short oblique fracture seen in the lower third of the ulna,
with angulation or overlap.
322- Colles' fracture splinted after reduction in
a- In 5 degrees flexion and 5 degrees ulnar deviation.
b- In 10 degrees flexion and 10 degrees ulnar deviation.
c- In 15 degrees flexion and 15 degrees ulnar deviation.
d- In 20 degrees flexion and 20 degrees ulnar deviation.
e- In 25 degrees flexion and 25 degrees ulnar deviation.
323- ‘Dorsal Barton’s fracture’.
a- The line of fracture runs obliquely across the dorsal lip of the radius and the
carpus carried anteriorly.
b- The fracture is not easy to control than the volar Barton’s fracture is.
c- The fracture can be easily reduced and to hold.
d- Is reduced closed like Colles' fracture and the forearm is immobilized in a cast
for 3 weeks.
e- If it re-displaces closed K-wiring or open reduction and plating is advisable.
324- Comminuted intra-articular fracture of distal radius in young adult
a- Is a low energy injury.
b- A good outcome will result even there is intra-articular congruity.
c- CT scans must be used to show the fragment alignment.
d- The most successful option is a manipulation and cast.
e- Open reduction and a combination of wires, plates, screws and bone grafts
may be used.
325- The commonest wrist injuries is
a- Fracture scaphoid.
b- Lunate dislocation.
c- Sprains of the capsule and ligaments.
d- Injury of the triangular fibrocartilage complex.
e- Injury of the distal radio-ulnar joint.
326- Scaphoid fractures account for
a- Almost 75 per cent of all carpal fractures.
b- Almost 60 per cent of all carpal fractures.
c- Almost 50 per cent of all carpal fractures.
d- Almost 35 per cent of all carpal fractures.
e- Almost 25 per cent of all carpal fractures.
327- Scaphoid non-union or avascular necrosis of the proximal fragment.
a- This accounts for the fact that 5 per cent of distal third fractures.
b- Develop in 10 per cent of middle third fractures
c- Develop in 20 per cent of proximal fractures.
d- Relative translucency of the proximal fragment is pathognomonic of avascular
necrosis.
e- Bone grafting may be successful,
328- Triquetro- lunate dissociation
a- A lateral sprain followed by weakness of grip and tenderness distal to radius.
b- X-rays show overlapped between the triquetrum and the lunate.
c- Acute tears should be repaired with interosseous sutures and a cast for 4–6
weeks.
d- Acute tears should be repaired with interosseous sutures Supported by
temporary K-wires for 3 weeks and a cast for 4–6 weeks.
e- In chronic injuries, a ligament substitution or a limited intercarpal fusion may
be considered.
329- Midcarpal dislocation
a- The extrinsic ligaments, which bind the proximal to the distal row, can
rupture.
b- The diagnosis is easy clinically.
c- The patient complains of a painless, recurrent snap in the wrist.
d- If an acute ligament rupture diagnosed, then treated by reduction and cast for 4
weeks.
e- In a chronic lesion, stabilization by K-wire is the most effective treatment.
330- Splintage in hand injuries
a- Splintage is not a cause of stiffness.
b- It must be appropriate and it must be kept to a minimum length of time.
c- If a finger has to be splinted, a rigid cast used.
d- Internal fixation should be avoided.
e- If the entire hand needs splinting, this must always be in the position of rest.
331- Multiple metacarpal fractures
a- Can adequately held by the surrounding muscles and ligaments.
b- Allows free early mobilization.
c- Should be fixed with rigid plates.
d- Should be held by cast.
e- Treated by multiple longitudinal wires.
332- Transverse fracture of the shaft of phalanges,
a- Often with backward angulation.
b- Often with medial angulation.
c- Often with lateral angulation.
d- Often with forward angulation.
e- Result from a twisting injury.
333- A mallet finger
a- Is best treated with a splint for 8 weeks.
b- Surgery is good alternative.
c- Surgery carries a low rate wound failure.
d- Metalwork problems is also rare.
e- Using a special mallet-finger splint make the outcome worse.
334- Avulsion of the flexor tendon of finger
a- Caused by direct trauma.
b- Caused by sudden hyperextension of the distal joint.
c- The little finger is most commonly affected.
d- The flexor digitorum superficialis tendon is avulsed.
e- Even If the diagnosis delayed, repair is likely to be successful.
335- Carpo-metacarpal dislocation
a- The thumb is less frequently affected and clinically resembles a Bennett’s
fracture dislocation;
b- The displacement of the thumb is easily reduced by traction and supination.
c- The reduction is stable.
d- A K-wire fixation is not recommended to prevent the joint from dislocating
again.
e- Chronic instability can occur.
336- Complex metacarpo-phalangeal dislocation
a- The avulsed palmar plate sits in the joint, blocking reduction.
b- The phalangeal base clasped between the flexor tendon and lumbrical tendon.
c- The finger extended only about 10 degrees and there is usually a telltale
dimple in the palm.
d- Usually the fracture reduced closed by hyperextending the MCP joint and
flexing the IP joints.
e- A volar approach is safest.
337- The complete rupture of ulnar collateral ligament of thumb
a- Is very common.
b- Only the ligament proper is torn.
c- The thumb is unstable in flexion only.
d- The thumb is unstable in all positions.
e- It will heal without surgical repair.
338- The zone II of hand injury is
a- Proximal to the carpal tunnel.
b- Within the carpal tunnel.
c- Between the opening of the flexor sheath (the distal palmar crease) and the
insertion of flexor superficialis.
d- Between the end of the carpal tunnel and the beginning of the flexor sheath.
e- Distal to the insertion of flexor digitorum superficialis.
339- Nail bed injuries
a- Are often seen as isolated injury.
b- If appearance is important, meticulous repair of the nail bed under
magnification.
c- Healing will be quicker with a split-skin graft.
d- Replacing any loss with a split skin graft from one of the toes, will give the
best cosmetic result.
e- In children, these injuries are associated with dislocation of DIJ.
340- The commonest cause of stiffness in hand injuries is
a- The presence of fractures.
b- Tendon injures.
c- Failure to use splintage in safety position.
d- The presence of edema.
e- The prolonged immobilization in volar slab.
341- After primary flexor tendon suture , the hand splinted in
a- The wrist held in about 20 degrees of flexion, the metacarpo-phalangeal joints
are flexed to only about 70 degrees but the interphalangeal joints must remain
straight.
b- The wrist held with a dorsal splint in about 50 degrees of flexion but the
interphalangeal joints must remain in 20 degrees of flexion.
c- The metacarpo-phalangeal joints flexed at least 70 degrees and the
interphalangeal joints almost straight.
d- The metacarpo-phalangeal joints extended and flexion of the interphalangeal
joints
e- The wrist extended to 30 degrees the metacarpo-phalangeal joints are flexed to
only about 30 degrees, and the interphalangeal joints remain straight.
342- After extensor tendon repair, the hand splinted in
a- The wrist held in about 20 degrees of flexion, the metacarpo-phalangeal joints
flexed to only about 70 degrees but the interphalangeal joints must remain
straight.
b- The wrist held with a dorsal splint in about 50 degrees of flexion but the
interphalangeal joints must remain in 20 degrees of flexion.
c- The metacarpo-phalangeal joints flexed at least 70 degrees and the
interphalangeal joints almost straight.
d- The metacarpo-phalangeal joints extended and flexion of the interphalangeal
joints
e- The wrist extended to 30 degrees, the metacarpo-phalangeal joints are flexed
to only about 30 degrees, and the interphalangeal joints remain straight.
343- MRI is the method of choice for
a- Showing structural damage to individual vertebrae.
b- Showing displacement of bone fragments into the vertebral canal.
c- Displaying the intervertebral discs, ligamentum flavum and neural structures.
d- Provides information on the dimensions of the spinal canal.
e- provides information on impingement by fracture fragments or intervertebral
disc
344- Stable injuries of spine treated
a- By supporting the spine in a position that will cause no further strain.
b- By prolonged splintage.
c- By traction for 2 months.
d- By stabilization by internal fixation followed by exercise and physiotherapy.
e- By decompression of spine and inter-spinal fusion. :[Comment [h4

345- Odontoid fractures can be fixed


a- With small plates between the lateral masses.
b- With lag screws.
c- With a halo-vest.
d- Anteriorly with plates between the vertebral bodies.
e- Posteriorly with wires between the spinous processes.
346- The anterior approach to the spine
a- Is suitable for wedge fractures.
b- The vertebral body preserved and a bone graft added.
c- Is suitable for burst fracture with significant canal impingement.
d- Suitable for flexion-compression injuries.
e- Suitable for seat-belt injuries and fracture-dislocations.
347- In the lateral view of cervical spine
a- Not all irregularity suggests a fracture or displacement.
b- Forward shift of the vertebral body by 50 per cent suggests a unilateral facet
dislocation.
c- Forward shift of the vertebral body by 40 per cent suggests a unilateral facet
dislocation.
d- Forward shift of the vertebral body by 25 per cent suggests a unilateral facet
dislocation
e- Forward shift of the vertebral body by 25 per cent suggests a bilateral facet
dislocation.
348- The distance between the odontoid peg and the back of the anterior arch of the
atlas should be
a- No more than 2 mm in adults and 2 mm in children .
b- No more than 3 mm in adults and 3 mm in children .
c- No more than 5 mm in adults and 5 mm in children .
d- No more than 3 mm in adults and 2.5 mm in children .
e- No more than 3 mm in adults and 4.5 mm in children.
349- Hangman's fracture
a- Treatment in a semi-rigid orthosis for 2-3 weeks.
b- Fractures with more than 3mm displacement need treatment in collar for 6
weeks.
c- In the treatment, traction must be avoided.
d- If displaced, reduced and the neck is held in Minerva jacket for 6 weeks.
e- If associated with a C2/3 facet dislocation Minerva jacket applied for 9 weeks.
350- C2 Odontoid process fracture
a- Odontoid fractures are not uncommon.
b- Occur as extension injuries in young adults after high velocity accidents or
severe falls.
c- A displaced fracture is really a fracture-dislocation of the atlanto-axial joint.
d- There is no room for displacement without neurological injury.
e- Cord damage is common.
351- Odontoid fractures Type II
a- Is stable.
b- Unites without difficulty.
c- Is the most uncommon.
d- Is potentially the most dangerous type.
e- The fracture is in tip of odontoid.
352- Posterior ligament injury of cervical spine
a- Sudden extension of the mid-cervical spine can result in damage to the
posterior ligament complex.
b- The upper vertebra tilts backwards on the one below, opening up the
interspinous space posteriorly.
c- The patient complains of pain and there may be localized tenderness
anteriorly.
d- It is always advisable to obtain a lateral view with the neck in the extension
position.
e- Flexion should not be permitted in the early post-injury period.
353- Wedge compression fracture of cervical spine
a- A pure extension injury.
b- The middle and posterior elements injured.
c- Is unstable.
d- Treated by a comfortable collar for 6–12 weeks.
e- Is potentially dangerous.
354- Burst fractures of cervical spine
a- Are due to flexion compression.
b- Vertebral body crushed by axial compression in neutral position of the neck.
c- There is no risk of posterior displacement of the vertebral body fragment and
spinal cord injury.
d- Soft collar applied for 6 weeks.
e- X-ray is sufficient to look for retropulsion of bone fragments into the spinal
canal.
355- Tear-drop fracture of cervical spine
a- Is comminuted vertebral body fracture has produced a large anterior fragment.
b- Obvious anterior displacement of the posterior fragment.
c- The severity of the injury can estimated well.
d- Treated effectively by a collar for 3 weeks is sufficient.
e- Neurological deficits is rare.
356- Bilateral facet joint dislocations are caused by
a- A sever hyperextension.
b- A flexion compression.
c- A vertical compression.
d- A sever flexion distraction.
e- A severe flexion–rotation.
357- Patients with minimal wedging of thoracolumbar spine treated by
a- Cast brace followed by bed rest for 4 weeks.
b- Bed rest for a week or two until pain subsides and are then mobilized.
c- Immobilization and back support needed for 6 weeks.
d- Thoracolumbar brace used for 6 weeks and are then mobilized.
e- Body cast applied with the back in extension and are then mobilized.
358- Wedge fracture with loss of anterior vertebral height is greater than 40 per cent
a- Is stable fracture.
b- It resist further collapse and deformity.
c- Treated by thoracolumbar brace.
d- Surgical correction and internal fixation is the preferred treatment.
e- Body cast applied with the back in extension and are then mobilized.
359- Thoracolumbar fracture-dislocation
a- In fracture-dislocation with paraplegia, surgery will facilitate nursing, and help
the patient’s rehabilitation.
b- In fracture-dislocation with a partial neurological deficit, there is evidence that
surgical stabilization provides a better neurological outcome.
c- If surgical decompression and stabilization are performed, this may require a
combined posterior and anterior approach.
d- In fracture-dislocation without neurological deficit, surgical stabilization will
not prevent future neurological complications.
e- Usually can be managed non-operatively with postural reduction, bed rest and
bracing.
360- Complete and incomplete paralysis in spinal injuries
a- The patient must be transported with great care to prevent further damage.
b- Bladder training begun at 2nd week.
c- The bowel training is more difficult.
d- Heterotopic ossification is a rare complication.
e- If bedsores have allowed developing, usually heal by postural treatment.
361- The morale of a paraplegic patient
a- Not liable to reach a low ebb.
b- The restoration of self-confidence is not an important part of treatment.
c- Constant encouragement by doctors, physiotherapists and nurses is not
essential.
d- The unpleasant smells of bowel accidents, or those associated with skin or
urinary infection cannot prevented.
e- The patient should find a hobby or be trained for a new job as quickly as
possible.
362- Fractures of the pelvis
a- Account for less than 1 per cent of all skeletal injuries.
b- Is important because of the high incidence of associated soft tissue injuries and
the risks of severe blood loss
c- Like other serious injuries, they demand an isolated approach by expert's
surgeon.
d- About one-thirds of all pelvic fractures occur in road accidents involving
pedestrians.
e- Over 1 per cent of these patients will have associated visceral injuries.
363- The patient with pelvic fracture with suspected urethral injury
a- Should not be encouraged to void.
b- If he is able void, there is no damage.
c- No attempt should be made to pass a catheter, as this could convert a partial to
a complete tear of the urethra.
d- The absence of blood at the meatus exclude a urethral injury.
e- Can be diagnosed more accurately by cystography.
364- The ilium and acetabulum is well defined in
a- Anteroposterior view.
b- Lateral view.
c- Oblique view.
d- Inlet view.
e- Outlet view.
365- Avulsion fractures of pelvis.
a- A apiece of bone is broken direct trauma.
b- This is usually seen old patients.
c- All are essentially impact injury.
d- Treated by skin traction and rest for a few weeks.
e- Biopsy of the callus in site of injury, may lead to an erroneous diagnosis.
366- Vertical shear pelvic fracture
a- The innominate bone on both side displaced vertically.
b- Fracturing the pubic rami and disrupting the sacroiliac region on the same
side.
c- This occurs typically when someone falls from a height onto both leg.
d- Are usually stable.
e- Rarely complicated with gross tearing of the soft tissues and retroperitoneal
hemorrhage.
367- High-energy fractures of the pelvis
a- Are stables injuries.
b- Carrying a low risk of associated visceral damage.
c- Carry low risk of intra-abdominal and retroperitoneal hemorrhage.
d- Carrying great risk of shock, sepsis and ARDS.
e- The mortality rate is low.
368- The main cause of death following high-energy pelvic fractures is
a- Airway obstruction.
b- Respiratory system injury.
c- Severe bleeding.
d- Fat embolism.
e- Respiratory distress syndrome.
369- Pelvic fracture with a large retroperitoneal hematoma
a- It should be evacuated by laparotomy.
b- It should be evacuated by laparoscopy.
c- It should be controlled by selective embolization.
d- It should not be evacuated.
e- It should be controlled by vascular repair.
370- Urological injury in pelvic ring fracture
a- Occurs in about 10 per cent of patients.
b- Occurs in about 15 per cent of patients.
c- Occurs in about 20 per cent of patients.
d- Occurs in about 25 per cent of patients.
e- Occurs in about 30 per cent of patients.
371- Treatment of open book fracture with the anterior gap is more than 2 cm
a- By bed rest.
b- By a posterior sling to ‘close the book’.
c- By a pelvic binder to ‘close the book’.
d- By external fixation with pins in both iliac blades connected by an anterior
bar.
e- By plating anteriorly and ilio-sacral screw fixation posteriorly.
372- Types IV Thompson and Epstein classification of hip dislocations is
a- Dislocation with no more than minor chip fractures.
b- Dislocation with single large fragment of posterior acetabular wall.
c- Dislocation with comminuted fragments of posterior acetabular wall.
d- Dislocation with fracture through acetabular floor.
e- Dislocation with fracture through acetabular floor and femoral head.
373- Type I posterior hip dislocation
a- Reduction is usually unstable.
b- Apply traction after reduction and maintain it for a few weeks.
c- Movement and exercises are begun as soon as pain allows.
d- The terminal ranges of hip movements are avoided to allow healing of the
capsule and ligaments.
e- The patient is allowed to walk with crutches about 2 weeks but without taking
weight on the affected side.
374- The period of hip ‘protection’ after posterior hip dislocation
a- Varies according to the age of the patient.
b- If the reduction was performed promptly (within 6 hours), then no more than
16 weeks should suffice.
c- If there was a longer delay then an extended period of 22 weeks may be wiser.
d- Progression of weight bearing should be graduated and the hip joint monitored
by x-ray.
e- The rationale for not bearing weight is to prevent avascular change.
375- Anterior dislocation of the hip
a- Is not rare compared with other types.
b- The usual cause is a fall from height.
c- The femoral head will then lie superiorly (type I - pubic) or inferiorly (type II -
obturator).
d- The leg lies externally rotated, adducted and slightly flexed.
e- The prominent head is easy to feel posteriorly.
376- Central dislocation of the hip
a- Is commonest type of dislocation.
b- The usual cause is dashboard injury.
c- A blow over the greater trochanter, may force the floor of the acetabulum
laterally.
d- It is really a fracture of the acetabulum.
e- Commonly complicated by sciatic nerve injury.
377- The fractures of femoral neck
a- Occasionally results from a simple fall.
b- Usually result from car accident or fall from height.
c- Some patients may have experienced minor symptoms.
d- In younger individuals, the usual cause is a fall on ground.
e- Stress fractures of the femoral neck occur in runners is common.
378- In Garden III fractures of femoral neck
a- The femoral head is in its normal position or tilted into valgus and impacted
on the femoral neck stump.
b- The femoral head trabeculae are normally aligned with those of the innominate
bone.
c- The femoral head trabecular markings are not in line with those of the
innominate bone.
d- The proximal fragment has lost contact with the femoral neck.
e- The anteroposterior x-ray shows that the femoral head is tilted out of position.
379- Displaced femoral neck fracture treatment
a- Displaced fractures will unite without internal fixation.
b- Operative treatment is almost mandatory.
c- Non-operative treatment used in patients with advanced dementia.
d- Non-operative treatment used in patients with little discomfort.
e- The fractures united if traction applied for 4 months.
380- In young patients with fracture neck femur
a- The longer the delay, the lesser is the likelihood of complication.
b- Operation is urgent.
c- Displaced fractures will unite without internal fixation.
d- Non-operative treatment can be used.
e- There is low incidence of complications.
381- Garden’s index for assessing reduction in subcapital fractures
a- On the anteroposterior x-ray, the medial femoral shaft and the axis of
trabecular markings over the medial aspect of the femoral neck lie at an angle
less than 155°.
b- On the anteroposterior x-ray, the medial femoral shaft and the axis of
trabecular markings over the medial aspect of the femoral neck lie between
160° and 180°.
c- On the lateral view, the trabecular markings would be in angle about 90°, if
the fracture perfectly reduced.
d- On the lateral view, the trabecular markings would be in angle about 120°, if
the fracture perfectly reduce
e- On the lateral view, the trabecular markings would be in angle about 150°, if
the fracture perfectly reduced.
382- Hip prostheses used for femoral neck fractures
a- This procedure carries a shorter operating time.
b- This procedure carries a less blood loss.
c- This procedure carries a lower infection rate than internal fixation.
d- Usually of the femoral part only (hemiarthroplasty) and may be inserted with
or without cement.
e- Uncemented prostheses have better mobility and less thigh pain.
383- Total hip replacement for femoral neck fractures may be indicated
a- If treatment not delayed.
b- If acetabular damage is not suspected.
c- In patients with metastatic disease.
d- If there is no Paget’s disease.
e- Old patient with impacted fracture.
384- The mortality rate in elderly patients with femoral neck fracture
a- May be as high as 5 per cent at 4 months after injury.
b- May be as high as 10 per cent at 4 months after injury.
c- May be as high as 15 per cent at 4 months after injury.
d- May be as high as 20 per cent at 4 months after injury.
e- May be as high as 25 per cent at 4 months after injury.
385- Ischemic necrosis of the femoral head after femoral neck fracture
a- Occurs in about 10 per cent of patients with displaced fractures.
b- Occurs in about 20 per cent of patients with displaced fractures.
c- Occurs in about 20 per cent of those with undisplaced fractures.
d- A few weeks later, an isotope bone scan may show increased vascularity.
e- Core decompression has no place in the management.
386- A ‘tip-apex’distance is described to identify a ‘sweet-spot’ for positioning the
sliding screw in intertrochanteric
a- If within 5 mm, there is a lower risk of the screw cutting out of the femoral
head.
b- If within 10 mm, there is a lower risk of the screw cutting out of the femoral
head .
c- If within 15 mm, there is a lower risk of the screw cutting out of the femoral
head .
d- If within 20 mm, there is a lower risk of the screw cutting out of the femoral
head .
e- If within 25 mm, there is a lower risk of the screw cutting out of the femoral
head .
387- Pathological intertrochanteric fractures
a- May be due to metastatic disease or myeloma.
b- These fractures seldom fail to unite.
c- Fracture fixation is essential in order to ensure union.
d- Methylmethacrylate cement is contraindicated.
e- Cementless total hip replacement may be preferable.
388- Hip fractures in children
a- Are commonly occur and potentially very serious.
b- The fracture is usually due to mild trauma.
c- Pathological fractures sometimes occur through a bone cyst or benign tumor.
d- In children over two years, the possibility of child abuse considered.
e- There is a low risk of complications.
389- Type IV fracture neck femur in children
a- Is a fracture-separation of the epiphysis.
b- The epiphyseal fragment is dislocated from the acetabulum.
c- The transcervical fracture; this is the least variety.
d- Is a basal (cervico-trochanteric) fracture.
e- Is an intertrochanteric fracture
390- Avulsion of the lessor trochanter
a- In the elderly, may occur by the pull of the psoas muscle.
b- The injury nearly always occurs swimming.
c- Treatment is rest, followed by return to activity when comfortable.
d- In the young adult, the lessor trochanteric fixed by screw.
e- In the adolescent, separation of the lesser trochanter should arouse suspicions
of metastatic malignant disease.
391- Fracture of the greater trochanter
a- In the elderly, a direct blow can fracture it after a fall.
b- In the elderly, treatment is operative and functional recovery is usually good.
c- The greater trochanter fractured by direct trauma in a young individual.
d- It can be treated conservatively by bed rest and analgesia.
e- Full weight bearing allowed early.
392- Intramedullary nails in subtrochanteric fractures
a- Are generally weaker.
b- Preferable for a pathological fracture.
c- Can tolerate stresses for shorter period if healing is slow.
d- Used for simple stable fracture.
e- Used with wide operative dissection.
393- Femoral shaft fractures in elderly patients should be considered
a- Open fracture.
b- Transverse fracture.
c- Oblique fractures.
d- Pathological fracture.
e- Comminuted fracture.
394- In proximal femoral shaft fractures
a- The proximal fragment is flexed, adducted and externally rotated.
b- The distal fragment is frequently abducted.
c- The distal fragment is frequently adducted.
d- The distal fragment is tilted by gastrocnemius pull.
e- The distal fragment adducted by gastrocnemius pull.
395- An ipsilateral femoral neck fracture associated with femoral shaft fracture is
occur in about
a- Two per cent of cases.
b- Four per cent of cases.
c- Six per cent of cases.
d- Eight per cent of cases.
e- Ten per cent of cases.
396- The risk of systemic complications in femoral shaft fracture can be
significantly reduced by
a- By use of Thomas splint.
b- By skin traction.
c- Early stabilization of the fracture, usually by a locked intramedullary nail.
d- Early stabilization of the fracture, usually by a plate and screws.
e- Early stabilization of the fracture, usually by reamed K nail.
397- In the multiply injured patient, particularly one with severe chest trauma,
prompt stabilization with
a- A cast splintage.
b- A skin traction.
c- A plate and screws.
d- An external fixator.
e- An intramedullary nail.
398- Ipsilateral fractures of the femur and tibia may leave the knee joint ‘floating’.
a- This is a very serious situation.
b- One of fractures will need immediate stabilization.
c- A lateral approach to the knee joint will allow both fractures to be stabilized
by plate.
d- Retrograde for the tibia and antegrade for the femur.
e- It is usual to fix the tibia first.
399- Pathological fractures through femoral shaft should be fixed by intramedullary
nailing
a- Provided the patient is fit enough to tolerate the operation.
b- A short life expectancy is a contraindication.
c- Prophylactic fixation is indicated, if a lytic lesion is greater than quarter the
diameter of the bone.
d- Prophylactic fixation is also indicated if a lytic lesion is longer than one cm on
any view.
e- The femur is likely to be bowed in the case of Paget’s disease, an osteotomy to
straighten the femur is contraindicated.
400- Femoral shaft fractures around a hip implant
a- Are common.
b- Occurs years later.
c- There are no x-ray signs of osteolysis.
d- There are no x-ray signs of implant loosening.
e- They may happen during primary hip surgery.
401- When a comminuted femoral fracture is plated
a- Early weight bearing allowed.
b- The rate of union is high and knee stiffness is less.
c- Custom brace applied and weight bearing delayed.
d- Bone grafts should be added and weight bearing delayed.
e- Bone grafts should be added and weight bearing allowed early.
402- Fractures of the femur in children
a- Are uncommon in older children.
b- Are usually due to indirect violence.
c- Healing is slow and complications are high.
d- Pathological fractures are rare in generalized disorders such as spina bifida and
osteogenesis imperfecta,
e- Pathological fracture may occur in cyst or tumor.
403- Femoral shaft fracture in children under 2 years of age
a- The commonest cause is fall from height.
b- The commonest cause is car accident.
c- The commonest cause is child abuse.
d- Malunion is common and serious in this age.
e- Nonunion is common after cast application.
404- The principles of treatment of femoral shaft fractures in children
a- Are different from in adults.
b- Open treatment is rarely necessary.
c- The choice of closed method depends largely type of fracture.
d- As children get smaller, fractures take longer to heal.
e- As children get smaller, there is a greater risk of malunion.
405- Infants with femoral shaft fractures
a- Treated by a 2 weeks in fixed traction.
b- Treated by a spica cast for another 2 –3 weeks.
c- Angulation of up to 30 degrees can be accepted.
d- Immediate spica casting is bad choice.
e- Surgery had low risk of complications.
406- Operative treatment with internal fixation for femoral supracondylar fracture
a- If the fracture is only slightly displaced.
b- If it reduces easily with the knee in flexion.
c- Enable accurate fracture reduction.
d- If the patient is young.
e- If the facilities and skill are limited.
407- Retrograde locked intramedullary nails for femoral supracondylar fracture are
suitable for
a- The type B fracture.
b- The type A fracture.
c- Comminuted type C fractures.
d- Severe osteoporotic bone.
e- Undisplaced fracture or is only slightly displaced.
408- For severely comminuted type C femoral supracondylar fracture
a- Traditional angled blade-plates used.
b- A 95-degree condylar screw-plate.
c- Retrograde locked intramedullary nails
d- The minimal contact plate with locking screws.
e- Unprotected early weight bearing is advisable.
409- Knee stiffness after operative treatment of femoral supracondylar fracture
a- Due to scarring from the injury and the operation.
b- Can prevented easily by early exercises.
c- A short period of exercise needed in all cases.
d- Full range of movement regained after physiotherapy.
e- Arthroscopic division of adhesions in the joint is contraindicated.
410- Fracture-separation of the distal femoral epiphysis
a- Is nearly as common as physeal fractures at the elbow.
b- In the childhood equivalent of a supracondylar fracture .
c- Is important because of its potential for nonunion.
d- Rarely caused growth and deformity of the knee.
e- Is usually a Salter–Harris type IV.
411- The femur’s length is derived from the distal physis in about
a- 40%.
b- 50%.
c- 60%.
d- 70%.
e- 80%.
412- Fracture-separation of the distal femoral epiphysis treatment
a- Rarely perfectly reduced manually.
b- Salter–Harris types 2 should be accurately reduced and fixed.
c- A flap of periosteum may be trapped in the fracture line.
d- The metaphyseal fragments should not be stabilized with percutaneous
Kirschner wires or lag.
e- The limb immobilized in plaster for 2-3 weeks postoperatively.
413- The primary stabilizer for valgus stress at 30 degrees of flexion is
a- The MCL.
b- The LCL.
c- The PCL.
d- The ACL .
e- Posterior oblique ligaments.
414- The cruciate ligaments provide
a- Anteroposterior stability.
b- Rotary stability.
c- Both anteroposterior and rotary stability.
d- Mainly resist excessive valgus angulation.
e- Mainly resist excessive varus angulation.
415- Triad of O’Donoghue is
a- MCL, LCL and medial meniscal injury.
b- MCL, ACL and medial meniscal injury.
c- MCL, PCL and medial meniscal injury.
d- PCL, ACL and medial meniscal injury.
e- PCL, LCL and medial meniscal injury.
416- Posterior sag of the proximal tibia is a reliable sign of injury in
a- ACL.
b- MCL.
c- LCL.
d- PCL.
e- Both cruciate damage.
417- A positive anterior drawer test is diagnostic of
a- ACL.
b- MCL.
c- LCL.
d- PCL.
e- Both cruciate damage.
418- Avulsed a small piece of bone from the near edge of the lateral tibial condyle
by
a- Lateral collateral ligament.
b- Iliotibial tract.
c- Lateral collateral and ACL.
d- Lateral collateral and posterolateral structures.
e- Posterior oblique ligaments .
419- Arthroscopy should not be attempted
a- Anterior cruciate ligaments.
b- Posterior cruciate ligaments.
c- Meniscus injuries.
d- Collateral ligaments.
e- Osteochondral injuries.
420- Isolated tears of the MCL
a- The knee is unstable in full extension.
b- Usually heal well enough to permit near-normal function.
c- Operative repair is necessary.
d- arthroscopy should be attempted.
e- A long cast-brace worn for 3 weeks.
421- Locking is feature of chronic
a- Anterior cruciate ligament injury.
b- Posterior cruciate ligament injury.
c- Anteromedial instability.
d- Posterolateral instability.
e- Meniscal tear.
422- The most reliable test for anterior cruciate ligament injury is
a- Anterior drawer test.
b- Posterior drawer test.
c- Lachman's test.
d- Pivot shift test.
e- Reverse pivot shift test.
423- The reliable method of diagnosing central meniscus injury
a- Pivot shift test.
b- Revers pivot test.
c- Apley's compression test.
d- MRI.
e- Arthroscopy.
424- Arteriography in dislocation of knee
a- It is essential in all cases.
b- It must be done in all cases with nerve injury.
c- Is not essential if the clinical assessment of the circulation is normal.
d- If the ankle/brachial arterial pressure index more than 0.9.
e- If there is, any associated fracture an arteriogram obtained.
425- Popliteal artery damage in dislocation of knee
a- Occurs in nearly 40 per cent of patients.
b- Occurs in nearly 50 per cent of patients.
c- Needs elevation and closed observation.
d- Needs an immediate repair.
e- Delay and an extended warm ischemic period can reduce risk of amputation.
426- Nerve injury in dislocation of knee
a- The posterior popliteal nerve may be injured.
b- Spontaneous recovery is rare.
c- About 60 per cent of patients can be expected to improve.
d- If there is no sign of recovery, a transfer of tibialis posterior tendon may help
restore ankle dorsiflexion.
e- If there is no sign of recovery, a transfer of tibialis anterior tendon may help
restore ankle plantarflexion.
427- Adolescents suffer disruption of the extensor apparatus in
a- The quadriceps tendon.
b- The attachment of the quadriceps tendon to the proximal surface of the patella.
c- Through the patella and retinacular expansions.
d- The junction of the patella and the patellar ligament.
e- The insertion of the patellar ligament to the tibial tubercle.
428- Fracture of the tibial tubercle
a- Usually occurs in old people.
b- The area over the tubercle is swollen but active extension painless.
c- The anteroposterior x-ray shows the fracture.
d- Sometimes the patella is abnormally high.
e- Complete separation treated by applying a long-leg cast with the knee in
extension for 6 weeks.
429- Osgood–Schlatter disease
a- Single trauma is the cause.
b- Give rise to a painful, tender swelling over the tibial tubercle.
c- The condition is uncommon in adolescents who are keen on sport.
d- X-ray show cystic lesion in tibial tubercle.
e- Treatment consists of a long-leg brace for 5 weeks.
430- Fractured patella
a- The patella is not a sesamoid bone.
b- The patella is the only insertion site of quadriceps muscle.
c- The mechanical function of the patella is to reduce the efficiency of the
quadriceps.
d- The key to the management of patellar fractures is the state of the entire
extensor mechanism.
e- If the patella is fractured, active knee extension is impossible.
431- Direct fracture of patella
a- Is a transverse fracture with a gap between the fragments.
b- Occurs, when contracts the quadriceps muscle forcefully.
c- Patient cannot lift the straight leg.
d- Occur usually when fall onto the knee or a blow against the dashboard of a car .
e- Operative treatment is essential.
432- Bipartite patella
a- Is often unilateral.
b- The line is sharp and irregular.
c- The line is transverse.
d- The lines are longitudinal.
e- The line running obliquely across the superolateral corner of the patella.
433- Type 1 – a vertical split of the lateral condyle
a- This is a fracture through dense bone.
b- Usually occur in older people.
c- May be virtually undisplaced, or the condylar fragment may be pushed
superiorly and tilted.
d- The lateral meniscus protected from damage.
e- The depressed fragments may be wedged firmly into the subchondral bone.
434- The most reliable imaging of tibial plateau fracture
a- Anteroposterior x-ray.
b- Lateral x-ray.
c- Oblique x-rays.
d- CT scan.
e- MRI.
435- Treatment of Type 1 tibial plateau fractures
a- Undisplaced fractures treated operatively.
b- Weight bearing not allowed for 12 weeks.
c- The aim is for an accurate reduction.
d- Displaced fractures treated by bone graft.
e- The femoral condylar surface examined and trapped fragments removed.
436- Type 3 tibial plateau fractures principles of treatment
a- The knee is usually unstable.
b- A satisfactory outcome is less predictable.
c- The depressed fragments may need to be elevated through joint.
d- The elevated fragments supported with bone grafts and the whole segment
fixed in position with ‘raft’ screws.
e- Postoperatively, exercises delayed.
437- Medial tibial condylar split fracture-
a- Usually occur in older people.
b- Caused by low-energy trauma.
c- The fracture itself is simple.
d- Good lateral x-rays or CT are needed to define the fracture pattern.
e- There is often an underlying ligament injury on the medial side.
438- Deformity following tibial plateau fracture
a- Some residual valgus or varus deformity is quite common.
b- Result only from incompletely reduced
c- Result only from re-displaced fracture during treatment.
d- Moderate deformity is not compatible with good function.
e- Predispose to osteoarthritis early after treatment.
439- Fracture of the proximal end of the fibula
a- Caused only by direct injury.
b- An isolated fracture of the proximal fibula is common.
c- It may be part of a more extensive rotational injury of the leg.
d- X-ray of the ankle not indicated.
e- The fracture need open reduction and internal fixation.
440- Dislocation of proximal tibiofibular joint
a- Twisting injury is the only cause dislocation of the distal tibio-fibular joint.
b- Isolated injuries are common.
c- Occasionally the condition is habitual and associated with generalized
ligamentous laxity.
d- The fibular head displaces downwards.
e- Always check for posterior tibial nerve injury.
441- The incidence of infection in tibial Gustilo type I is
a- 1 per cent.
b- 2 per cent.
c- 3 per cent.
d- 4 per cent.
e- 5 per cent.
442- The incidence of infection in tibial Gustilo type III Cis
a- 5 per cent.
b- 10 per cent.
c- 20 per cent.
d- 30 per cent.
e- 40 per cent.
443- The least stable fracture of tibia is
a- Long spiral fracture.
b- A butterfly fracture.
c- Severely comminuted fractures.
d- Long oblique fracture.
e- Transverse fracture .
444- Tscherne’s IC4 fracture of tibia is associated with
a- Necrosis from contusion.
b- Extensive, closed degloving.
c- Circumscribed degloving.
d- Skin contusion.
e- No skin lesion
445- Gustilo’s IIIA open tibial fractures
a- Wound usually <1 cm long.
b- Bone injury is simple low-energy.
c- Moderate comminution.
d- Comminuted but soft-tissue cover possible.
e- Requires soft-tissue reconstruction for cover.
446- Gustilo’s IIIC open tibial fractures
a- Wound Usually >10 cm long.
b- Moderate soft tissue injury, some muscle damage.
c- Moderate comminution.
d- Comminuted fracture, but soft-tissue cover possible.
e- Severe loss of soft-tissue cover with need for vascular repair
447- Gustilo’s II open tibial fractures
a- Wound usually <1 cm long.
b- Minimal soft tissue injury.
c- Bone injury is simple low-energy.
d- Moderate contamination.
e- Requires soft-tissue reconstruction for cover.
448- The method of choice for internal fixation of displaced fracture of tibia in
adults is
a- Plate and screws.
b- Open intramedullary nailing.
c- Closed intramedullary nailing.
d- External fixation.
e- Submuscular plating.
449- Large bone defects in open tibia fractures treated by
a- Plate and bone graft.
b- Closed intramedullary nail and bone graft.
c- Open intramedullary nail and bone graft.
d- External fixation and bone graft.
e- Bone transport or compression-distraction with an external fixator.
450- The safest temporary stabilization for Gustilo grades IIIB and C is
a- A spanning external fixator.
b- Plate and screws.
c- Open intramedullary nailing.
d- Closed intramedullary nailing.
e- Submuscular plating.
451- Postoperative management of open tibial fractures
a- The limb should be elevated and frequent checks made for signs of
compartment syndrome.
b- With locked intramedullary nails, weight bearing delayed.
c- If the fracture comminuted, full weight bearing permitted when callus seen on
x-ray.
d- With plate fixation, early weigh bearing permitted without cast application.
e- Patient with transverse fracture stabilized by external fixation, weight bearing
delayed.
452- Patients with open tibial fractures stabilized with external fixators
a- Cannot weight-bear early.
b- Weight bearing through the fractured tibia increased when callus is visible on
x-ray.
c- The fixator is ‘dynamized’ early to allow greater load transfer through the
bone.
d- When fracture consolidated the external fixator exchanged to a plate.
e- If the pin sites are in poor condition, a change to plate is helpful.
453- The diagnosis of compartment syndrome is usually suspected on
a- Doppler examination.
b- Ultrasonography.
c- Leg radiological examination.
d- Measurement of compartment pressure.
e- Clinical grounds.
454- Warning symptom of compartment syndrome in the leg is
a- Increasing pain.
b- Numbness.
c- Hypoesthesia.
d- Anesthesia.
e- Loss of function.
455- Warning sign of compartment syndrome in the leg is
a- Swelling.
b- Bruises.
c- Pallor of the toes ends.
d- Pain provoked by muscle stretching.
e- Changes of skin colour in dependent position.
456- The diagnosis of compartment syndrome can be confirmed by
a- Angiography.
b- Doppler examination of the leg blood vessels.
c- Measuring the compartment pressures in the leg.
d- Skin oxygen tension.
e- The leg venography.
457- The tibial fracture associated with decompression fasciotomy is treated as
a- A grade I open fracture by closed intramedullary nailing.
b- A grade I open fracture by open intramedullary nailing.
c- A grade II open fracture by closed intramedullary nailing.
d- A grade III open fracture by a spanning external fixator.
e- A grade III open fracture by closed intramedullary nailing.
458- Compartment decompression of leg should be performed within
a- Within 4 hours of the onset of symptoms.
b- Within 6 hours of the onset of symptoms.
c- Within 8 hours of the onset of symptoms.
d- Within 10 hours of the onset of symptoms.
e- Within 12 hours of the onset of symptoms.
459- Hypertrophic non-union of tibia can be treated by
a- Intramedullary nailing (or exchange nailing).
b- Neutral plating.
c- Neutral plating and bone grafting.
d- Compression plating and bone grafting.
e- External fixation and bone grafting
460- Fracture of tibia alone
a- An indirect injury may cause a transverse fracture of the tibia alone at the site
of impact.
b- In children, the fracture is usually caused by a direct injury.
c- Local bruising and swelling are usually evident.
d- A fracture of the tibia alone takes half of the time of both leg bones to unite.
e- In children at least 12 weeks is needed for union
461- Fracture of fibula alone
a- Isolated spiral fractures is safe injury.
b- A long oblique fracture may be due to a direct blow.
c- There is local tenderness, but the patient is able to stand and to move the knee
and ankle.
d- Cast applied for 8 week for undisplaced transverse fracture.
e- In displaced transverse fracture, plating is preferable.
462- Fatigue fractures of tibia and fibula
a- Single stress may cause a fatigue fracture of the tibia or the fibula.
b- This injury seen in army recruits, mountaineers, and runners, who complain of
pain in the leg.
c- There is no local tenderness and swelling.
d- In the first week, periosteal new bone formation seen.
e- Treated usually by casting for 8–10 weeks.
463- During running and jumping, the loads transmitted through the ankle and foot.
a- Two times body weight.
b- Four times body weight
c- Six times body weight
d- Eight times body weight.
e- Ten times body weight.
464- The ankle moves in
a- Flexion / extension plane.
b- A complex axis of rotation.
c- A rolling forward.
d- A sideways movement.
e- Inversion / eversion plane.
465- The most common of all sport related injuries is
a- Meniscus injuries.
b- Anterior cruciate ligament injuries.
c- Anterior shoulder dislocation.
d- Ankle sprain.
e- Wrist sprain.
466- Lateral collateral injuries in ankle sprain forms more than
a- 35%.
b- 45%.
c- 55%.
d- 65%.
e- 75%.
467- In planter flexion of ankle, the vulnerable ligament for injury is
a- The posterior talofibular ligament.
b- The calcaneofibular ligaments.
c- The anterior talofibular ligament.
d- The talocalcaneal ligament.
e- The deep part of medial collateral ligament.
468- The deep portion of medial collateral ligament of ankle principal effect is to
a- Resist eversion of the hindfoot.
b- Prevent external rotation of the talus.
c- Restraining eversion of ankle.
d- Restraining inversion of ankle.
e- Restraining external rotation.
469- The first ligament injured in twisted ankle is
a- The talocalcaneal ligaments.
b- The anterior talofibular ligament.
c- The posterior talofibular ligament.
d- The calcaneofibular ligament.
e- The medial collateral ligament.
470- Sprain of lateral collateral of ankle
a- A history of a twisting injury followed by pain and swelling.
b- Is the third common type of sport injuries.
c- Extensive bruising appear early.
d- The patient unable to put any weight on the foot.
e- The x-ray is essential to confirm the diagnosis.
471- The lateral ligament injuries of ankle may mimic
a- Displaced fractures of the fibula
b- Displaced fractures of the tarsal bones.
c- The injuries of the distal tibiofibular joint.
d- The injuries of the tibialis posterior tendon sheath.
e- The injuries of the tibialis anterior tendon sheath.
472- For patients with ankle sprain, who have had persistent pain, swelling,
instability and impaired function over 6 weeks
a- Repeat x- ray examination.
b- Stress film x-ray examination.
c- Magnetic resonance imaging.
d- Ultrasonography examination of ankle.
e- Arthrography of ankle joint.
473- Recurrent ankle sprains occur after acute lateral collateral ligament tears in
about
a- Five per cent of cases.
b- Ten per cent of cases.
c- Fifteen per cent of cases.
d- Twenty per cent of cases.
e- Twenty five per cent of cases.
474- Recurrent dislocation of peroneal tendons treated by
a- A below knee cast for 3 weeks.
b- A below knee cast for 6 weeks.
c- Leg-ankle splint with lateral bar.
d- Operative treatment.
e- Physiotherapy and electrical muscles stimulation.
475- Complete diastasis of inferior tibiofibular ligaments tear
a- Result from tearing of both the anterior and posterior ligaments.
b- Follows a severe adduction strain.
c- Result from tearing of only the anterior tibiofibular ligament.
d- Rarely associated with fractures of the malleoli or rupture of the collateral
ligaments.
e- X-ray shows narrowing of the ankle mortise.
476- Type A Denis and Weber ankle fracture
a- Is due to severe abduction or a combination of abduction and external rotation.
b- Is a transverse fracture of the fibula below the tibiofibular syndesmosis,
associated with vertical fracture of the medial malleolus.
c- Is an oblique fracture of the fibula at the level of the syndesmosis; often there
is also an avulsion injury on the medial side.
d- Is probably an external rotation injury and it may be associated with a tear of
the anterior tibiofibular ligament.
e- Associated injuries are a posterior malleolar fracture and diastasis of the
tibiofibular joint.
477- Non-union of the medial malleolus due to
a- Sever trauma.
b- Skin blister and skin necrosis.
c- Periosteal flap interposition.
d- Delayed reduction.
e- Use of screw fixation.
478- Joint stiffness following ankle fractures
a- Stiffness of the ankle are usually the result of neglect in treatment of the bone.
b- The patient must walk correctly in plaster.
c- Delaying operative treatment minimized stiffness.
d- Avoid wearing crepe bandage when the plaster removed.
e- Avoid elevation of the leg when the plaster removed.
479- Rüedi type 1 Pilon fracture
a- There is severe disruption of the articular surface but without very marked
comminution.
b- There is an intra-articular fracture with little or no displacement of the
fragments.
c- There is a severely comminuted fracture with displacement of the fragments.
d- There is gross articular irregularity.
e- The assessment is far better with plain x-ray examination.
480- The frequent late complications of Pilon fractures is
a- Late foot edema.
b- Postoperative osteomyelitis.
c- Implant failure.
d- Secondary osteoarthritis.
e- Shortening and malunion.
481- Ankle fractures in children
a- Physeal injuries are rare in children.
b- The tibial (or fibular) physis wrenched apart, usually resulting in a Salter–
Harris type 4 fracture.
c- Type 1 and 2 fractures are uncommon.
d- With severe external rotation, the fibula may also fracture more proximally.
e- With abduction injuries, the tip of the fibula may be avulsed.
482- Tillaux fracture
a- Is a simple triplane fracture.
b- Is an avulsion of a fragment of tibia by the posterior tibiofibular ligament.
c- Occur in the old age group.
d- The fragment is the medial part of the epiphysis.
e- Is a Salter–Harris type 3 fracture.
483- Asymmetrical growth in ankle epiphyseal fractures
a- Fractures through the epiphysis may result in generalized fusion of the physis.
b- The bony bridge is usually in the lateral half of the growth plate.
c- The medial half goes on growing and the distal tibia gradually veers into
varus.
d- MRI and CT are helpful in showing precisely where physeal arrest has
occurred.
e- If the bony bridge is large, it can be excised and replaced by a pad of fat in the
hope that physeal growth may be restored.
484- CT is especially useful for evaluating
a- Fracture of the talus.
b- Fractures of the calcaneum.
c- Fractures of the navicular.
d- Osteochondral fractures of the talus.
e- Stress fractures.
485- MRI is helpful in diagnosing
a- Osteochondral fractures of the talus.
b- Fractures of the calcaneum.
c- Fractures of the navicular.
d- Fracture of the medial the cuneiform.
e- Stress fractures.
486- The superior articular surface carries a greater load per unit area in
a- The femoral head.
b- The Proximal tibia.
c- The Proximal fibula.
d- The talus.
e- The calcaneum.
487- Fractures of the neck of the talus Type IV by Hawkins is
a- Associated with subluxation or dislocation of the subtalar joint.
b- Displaced, with dislocation of the body of the talus from the ankle joint.
c- Displaced vertical talar neck fracture with associated talonavicular joint
disruption.
d- Undisplaced.
e- Little displaced.
488- The incidence of avascular necrosis in type 3 of fracture talus is
a- More than 50 per cent.
b- More than 60 per cent.
c- More than 70 percent.
d- More than 80 per cent.
e- More than 90 per cent.
489- Osteoarthritis of the ankle and/or subtalar joints occurs some years after talar
neck fractures
a- In over 40 per cent of patients.
b- In over 50 per cent of patients.
c- In over 60 per cent of patients.
d- In over 70 per cent of patients.
e- In over 80 per cent of patients.
490- The calcaneum
a- Is the most commonly fractured tarsal bone.
b- Fracture in 28 % of cases both heels injured simultaneously.
c- Crush injuries; always heal with little long-term disability.
d- “The man who breaks his heel-bone is finished”, still applicable.
e- Open reduction and internal fixation of crush fractures not improve the
outcome.
491- Fracture calcaneum suffer associated injuries of the spine, pelvis or hip in
a- Over 5 per cent of these patients.
b- Over 10 per cent of these patients.
c- Over 15 per cent of these patients.
d- Over 20 per cent of these patients.
e- Over 25 per cent of these patients.
492- Extra-articular fractures of calcaneum
a- Account for 50 per cent of calcaneal injuries.
b- They usually follow complex patterns.
c- Associated with crushing of the anterior process, the sustentaculum tali, the
tuberosity or the inferomedial process.
d- Fractures of the posterior (extra-articular) part of the body caused by
compression.
e- Extra-articular fractures are usually difficult to manage and have a bad
prognosis.
493- Compartment syndrome following fracture calcaneum develop in
a- About 2 per cent of patients.
b- About 5 per cent of patients.
c- About 10 per cent of patients.
d- About 15 per cent of patients.
e- About 20 per cent of patients.
494- The undisplaced fractures of navicular bone
a- Percutaneous fixation by K-wire used followed by cast.
b- Need open reduction and screw fixation.
c- Cast brace applied immediately and continue for 8 weeks.
d- The foot is elevated to counteract swelling, after 3 or 4 days a below-knee cast
for 4–6 weeks.
e- The foot is elevated to counteract swelling, after 3 or 4 days a percutaneous
fixation by K-wire used.
495- Even with accurate reduction of midtarsal fracture–dislocations, post-
traumatic osteoarthritis may develop and
a- About 10 per cent of patients fail to regain normal function.
b- About 20 per cent of patients fail to regain normal function.
c- About 30 per cent of patients fail to regain normal function.
d- About 40 per cent of patients fail to regain normal function.
e- About 50 per cent of patients fail to regain normal function.
496- The best treatment for the first metatarsal fracture with significant
displacement in the sagittal plane is
a- Reduction and cast brace.
b- Elevation for few days followed by cast splintage.
c- Open reduction and internal fixation.
d- By removable boot splint, the foot is elevated and partial weight bearing for
about 4–6 weeks.
e- A below-knee cast is applied and weight bearing avoided for 7 weeks.
497- The fracture at metaphyseal/diaphyseal junction of 5th metatarsal
a- Is avulsion fracture of the base of the fifth metatarsal – the pot-hole injury.
b- Has a higher rate of non-union, probably because of the relatively poor blood
supply in that region.
c- Examination will disclose a point of tenderness directly over the prominence
at the base of the fifth metatarsal bone.
d- Treated symptomatically, with initial rest and support, but with early
mobilization and return to function.
e- A normal peroneal ossicle or apophyseal ossification centre in the tuberosity
may be mistaken for a fracture.
498- The proximal avulsion fractures of base of 5th metatarsal treated
a- By closed reduction under anesthesia followed by cast for 6 weeks.
b- Symptomatically, with initial rest and support.
c- Nonoperatively, but there is a greater risk of non-union and slower return to
function.
d- Fixed internally by with an interfragmentary screw.
e- Fixed internally by plate and screws.
499- Fractured toes
a- A twisting force is the commonest cause of fracture phalanges.
b- If the skin is broken it must be covered with a sterile dressing, and antibiotics
are given.
c- An associated contaminated wound will require percutaneous Kirschner wire.
d- The wound is disregarded and the patient encouraged walking in a supportive
boot or shoe.
e- If pain is marked, the toe splinted plaster of Paris.
500- Fractured sesamoids bone of big toe
a- One of the sesamoids (usually the lateral) may fracture from either a direct
injury or sudden traction.
b- The patient complains of pain in tip of toe.
c- There is a tender spot in medial side of first MP joint.
d- The pain exacerbated by passive hyperflexion of the big toe.
e- Treated by walking cast for 6 weeks.
501- The bipartite sesamoid of big toe
a- X-rays will usually show the sharp fracture.
b- X-rays will usually show a smooth- edged two-bone segment.
c- Treated by support in a removable boot/splint for 2–3 weeks.
d- Treated by an insole with differential padding to speed recovery.
e- Occasionally, intractable symptoms call for excision of the offending ossicle.
PART- TWO

The following are correct except

1- Periosteal reaction typically seen in


a- Fractures healing.
b- Osteoid osteoma.
c- Infection.
d- Ewing's sarcoma.
e- Osteosarcoma.
2- The classical criteria of osteoarthritis is
a- Narrowing of the joint space.
b- Subchondral sclerosis.
c- Subchondral cysts.
d- Osteophytes.
e- Periarticular erosions
3- The classical criteria of inflammatory arthritis
a- Narrowing of the joint space.
b- Local osteoporosis.
c- Periarticular erosions.
d- Osteophytes formation.
e- Soft tissue swelling.
4- Radionuclide imaging is useful in many situations
a- The diagnosis of stress fractures or other undisplaced fractures that are not
detectable on the plain x-ray.
b- The detection of a small bone abscess, or an osteoid osteoma.
c- The early detection of multiple myeloma.
d- The diagnosis of femoral head ischaemia in Perthes’ disease or avascular
necrosis in adults.
e- The early detection of bone metastases.
5- The erythrocyte sedimentation rate (ESR) is usually increased in
a- Acute inflammatory disorders.
b- Chronic inflammatory disorders.
c- After tissue injury.
d- Low-grade infection.
e- Myelomatosis.
6- Arthrocentesis and synovial fluid analysis considered in the following
conditions.
a- Acute joint swelling after injury
b- Acute atraumatic synovitis in adults.
c- Suspected infection.
d- Chronic synovitis.
e- Villinodular synovitis.
7- Open bone biopsy, has several drawbacks
a- It requires an operation, with the attendant risks.
b- New tissue planes opened up, predisposing to spread of tumor.
c- The biopsy incision may jeopardize subsequent wide excision.
d- The more inaccessible lesions reached only by dissecting widely through
healthy tissue.
e- In the best hands, it has an accuracy rate of over 95%.
8- Increased activity in the perfusion phase of Radionuclide scanning due to
a- Increased soft-tissue blood flow, suggesting inflammation.
b- A fracture.
c- Local vascular insufficiency.
d- A vascular tumor.
e- Regional sympathetic dystrophy.
9- Arthroscopy of the hip
a- Is widely used.
b- Proving to be useful in the diagnosis of unexplained hip pain.
c- Proving to be useful in the diagnosis of Labral tears.
d- Proving to be useful in the diagnosis of synovial lesions and loose bodies.
e- Proving to be useful in the diagnosis of articular cartilage damage.
10- Host susceptibility to infection is increased by
a- Trauma and scar tissue.
b- Poor circulation and diminished sensibility.
c- The absence of foreign bodies.
d- Malnutrition and general illness.
e- Corticosteroid administration and all forms of immunosuppression,
11- Anaerobic organisms had been found in patients with osteomyelitis,
a- Usually as part of a mixed infection.
b- Are more likely in heroin addicts.
c- As opportunistic pathogens in patients with compromised immune defense
mechanisms.
d- With sickle-cell disease.
e- Usually as part of prolonged chronic infection.

12- In adults, haematogenous bone infection


a- Accounts for only about 20% of cases of osteomyelitis,
b- Mostly affecting the vertebrae.
c- Staphylococcus aureus is the commonest organism
d- Pseudomonas aeruginosa often appears in patients using intravenous drugs.
e- Diabetics are prone to haematogenous bone infections.

13- Infants osteomyelitis ( under a year old)


a- The constitutional disturbance can be misleadingly mild.
b- The baby fails to thrive and is drowsy but irritable.
c- May be complicated by septic arthritis.
d- May cause pseudo-paralysis.
e- Multiple infection is uncommon.
14- All investigation are uncertain in diagnosis of diagnosis acute osteomyelitis
a- Aspirate pus or fluid from the metaphyseal subperiosteal abscess,
b- Blood cultures.
c- Elevated C-reactive protein (CRP).
d- Elevated erythrocyte sedimentation rate (ESR).
e- White blood cell (WBC) count rises.
15- Streptococcal necrotizing myositis
a- Caused by Group B beta hemolytic streptococci.
b- Caused by Group A beta hemolytic streptococci.
c- Is rare.
d- Is a differential diagnosis to acute osteomyelitis.
e- Immediate treatment with intravenous antibiotics is essential.
16- Treatment of acute osteomyelitis in neonates and infants up to 6 months of age
a- Initial antibiotic treatment should be effective against penicillin-resistant
Staphylococcus aureus.
b- Initial antibiotic treatment should be effective against Group B streptococcus.
c- Initial antibiotic treatment should be effective against Gram-negative
organisms.
d- Drugs of choice are flucloxacillin plus a third-generation cephalosporin like
cefotaxime.
e- Drugs of choice intravenous vancomycin with a third-generation
cephalosporin.
17- Suppurative arthritis as complication to acute osteomyelitis
a- May occur in very young infants.
b- May occur after upper tibia osteomyelitis.
c- Where the metaphysis is intracapsular, as in the upper femur.
d- May occur as a form of metastatic infection.
e- Ultrasound will help to demonstrate an effusion, but the definitive
diagnosis given by joint aspiration.
18- Brodie’s abscess is
a- A circumscribed, round or oval radiolucent ‘cavity’ 1–2 cm in diameter.
b- Sometimes the ‘cavity’ surrounded by a halo of sclerosis.
c- Diaphysis lesions may be associated with periosteal new bone formation and
marked cortical thickening.
d- Diaphyseal lesions may be associated with little periosteal new bone
formation.
e- Metaphyseal lesions cause little or no periosteal reaction.
19- Treatment of subacute haematogenous osteomyelitis
a- May be conservative if the diagnosis is doubtful.
b- May be conservative if the diagnosis is not in doubt.
c- Is immobilization and antibiotics (flucloxacillin and fusidic acid)
intravenously for 4 or 5 days and then orally for another 6 weeks.
d- Is curettage if the x-ray shows that there is no healing after conservative
treatment.
e- Is an open biopsy and the lesion may be curetted if the diagnosis in doubt.
20- In chronic osteomyelitis, the local defence mechanism impaired by
a- Scarring.
b- Sequestrum.
c- Sclerosis of cavity wall.
d- Covering of bacteria by glycocalyx.
e- Removal of forging implant.
21- Multifocal non-suppurative osteomyelitis
a- Is associated with chronic diabetes mellitus.
b- Is rare.
c- Comprises several different syndromes.
d- Is associated with chronic skin infection.
e- In children the condition usually takes the form of multifocal (often
symmetrical), recurrent lesions.
22- Infantile cortical hyperostosis
a- Is characteristically show periosteal new-bone formation resulting in
thickening of the affected bone.
b- Scurvy must be excluded.
c- Is not rare disease.
d- May resolved spontaneously.
e- Affect infants.
23- The cardinal features of osteoarthritis
a- Reduction of joint space
b- Periarticular erosion.
c- Subchondral sclerosis.
d- Osteophytes formation.
e- Subchondral cyst formation.
24- Calcium based synthetic substitute include the following
a- Calcium phosphate chips.
b- Calcium sulphate chips.
c- Calcium sulphate granules.
d- Bone morphogenetic protein.
e- Hydroxyapatite.
25- Bone allograft
a- Can be stored in bone bank.
b- Supplies can be plentiful.
c- Can transfer infection.
d- Can be sterilized by irradiation.
e- Can be sterilized by heat.

26- Acrylic cement


a- Acts as a grouting material.
b- Is mixed to set off an exothermic reaction of polymerization.
c- Can withstand large compressive loads.
d- Is hardly broken by tensile stress.
e- Mixing techniques have been shown to influence the tensile strength.
27- Fibrous layer forms at the cement bone interface
a- If the initial cement application is perfect.
b- Its thickness depending on the degree of cement penetration into the bone
crevices.
c- It is flimsy membrane fine granulation tissue and foreign body giant cells.
d- If polyethylene or metallic wear products track down into the cement/bone
interface, an aggressive reaction ensues.
e- This is relatively quiescent tissue remain unchanged under a wide range of
conditions.
28- Silicone rubber ( silastics) is particularly useful , it is
a- Firm.
b- Tough.
c- Flexible.
d- Inert.
e- Hard.
29- Silicon compound
a- Had susceptibility to fracture.
b- May induced a giant cell synovitis.
c- Sometime bone erosion and cyst formation seen at some distance from the
implant.
d- Is used extensively as external fixators.
e- May be used as temporary spacer prior to tendon transplant.
30- Malignancy after use of metal implant
a- The risk related.
b- The risk probably discounted.
c- Large number of cases.
d- Occur commonly in site of implant.
e- Commonly giant cell tumors.
31- Stainless steel implant
a- Had tensile plasticity.
b- Is hard.
c- The common form is 310L.
d- Strong.
e- Cannot bend the plate without serious disturbing their strength.
32- Cobalt chromium- based alloys
a- Used in joint prostheses.
b- Is corrosion sensitive.
c- Is biocompatible.
d- Had high strength.
e- Had through forging and cold-working.
33- Titanium alloys
a- Contain aluminum and vanadium in high concentration.
b- Used internal fixation devices.
c- Is corrosion resistance.
d- Had elastic modulus close to that of bone.
e- Scratch can reduce its fatigue life.
34- Implant failure result from
a- Defect during manufacturing.
b- Incorrect implant selection.
c- Exposure to repeated high stress from incorrect seating.
d- Delayed union.
e- Using heavy duty prostheses.
35- Replantation surgery is
a- Time consuming.
b- Often successful.
c- Need trained team.
d- Expensive.
e- Carried out only in centers specially equipped.
36- In amputation of ischemic limb
a- Energy expenditure in amputee is increased 30- 50% percent for transtibial.
b- Energy expenditure in amputee is increased 10-30% percent for trans-tibial.
c- Energy expenditure in amputee is increased 40-67% percent for trans- femoral.
d- Most of amputation done at site of election below the most distal palpable
pulse.
e- The skill of the modern prosthetist has made it possible to amputate at any
level.
37- Early complications of the amputation include
a- Secondary hemorrhage.
b- Gas gangrene.
c- Breakdown of skin flaps.
d- Wound infection.
e- Painful neuroma.
38- Operation to increase stature
a- Is painful.
b- Is prolonged.
c- May complicated with substantial complications
d- Gain height is same as normality.
e- Successful treatment is rewarding.
39- Distraction osteogenesis.
a- Safe regarding nerve and vascular injuries.
b- Can correct major shortening.
c- Has a lower rate of complications.
d- Has a high rate of complications.
e- Can applied at two level on same time.
40- The commonest cause of drug-induced osteonecrosis are
a- Alchohol.
b- Heparin.
c- Corticosteroid.
d- Immunosuppressive.
e- Cytotoxic.
41- Sickle –cell disease cause secondary changes
a- Periostitis.
b- Osteonecrosis.
c- Hyperuricaemia.
d- Increase susceptibility to bacterial infection.
e- Sever neurological defect.
42- The early treatment of osteoarthritis
a- Maintain movement and muscle strength.
b- Relieve pain.
c- Modify daily activities.
d- Protect joint from overload.
e- Arthroscopic joint debridement.
43- In Chondroblastoma
a- Appears in epiphysis.
b- There is predilection for female.
c- The presenting symptoms is ache.
d- The patients is adolescent or young adult.
e- Do not undergo malignant changes.
44- Surgical treatment of exostosis indicated in
a- If it cause symptoms.
b- If recently become bigger.
c- If recently become painful.
d- Large exostosis in lower femur.
e- Pelvic exostosis.
45- In osteoid osteoma
a- The nidus is dark brown nucleus.
b- There is no risk of malignancy.
c- The effective treatment is complete removal or destruction of nidus.
d- Any bone may be affected.
e- Patients are usually under 30 years of age.
46- The following lesion appear in metaphysis.
a- Simple bone cyst.
b- Aneurysmal bone cyst.
c- Osteosarcoma.
d- Chondromyxoid fibroma.
e- Chondroblastoma.
47- Chondrosarcoma characterized by
a- Slow growing.
b- Metastasize late.
c- The ideal case for radical resection.
d- Not respond to radiotherapy and chemotherapy.
e- The prognosis depending on the cellular grade and the resection margin.
48- Malignant fibrous histiocytoma
a- Spread early to soft tissue.
b- Patients are usually young age.
c- Tends to occurs in old bone infarct.
d- Tends to occurs in Paget's disease
e- X-rays may reveal a destructive lesion.
49- In Ewing's sarcoma
a- Patients presented with pain.
b- Sunray appearance is common in x-rays.
c- Codman's triangle is common in x-rays.
d- Marked periosteal reaction in diaphysis of long bone is common.
e- Histological examination show round cell surrounded by reticulin.
50- Parosteal osteosarcoma
a- Is situated on the surface of bone.
b- The patients is young adults.
c- The cortex is not eroded.
d- It spread into the adjacent muscle early.
e- Is low-grade sarcoma.
51- The commonest sites for bone metastases are the
a- The vertebrae,
b- The pelvis,
c- The tibia,
d- The proximal half of the femur ,
e- The proximal half of humerus.
52- Bauer's positive criteria for survival in metastases are
a- A solitary metastasis.
b- No pathological fracture.
c- No visceral metastases.
d- Prostate or thyroid primary.
e- No lung cancer.
53- Pigmented villonodular synovitis
a- Presented as a longstanding boggy swelling of the large joint.
b- The effective treatment is the synovectomy.
c- May undergo malignant changes.
d- X-rays may show excavations in the juxta- articular bone.
e- When the joint is opened, the synovium is swollen and hyperplastic, often
covered with villi and golden-brown in colour.
54- Diplegia
a- Involve both side of body.
b- Lower limbs always most severely affected.
c- Side to side involvement may be asymmetrical.
d- Intelligence is often subnormal.
e- Many cases are secondary to prematurity.
55- Botulinum toxin
a- Act by blocking acetylcholine release.
b- Injected in spastic muscles.
c- Muscles tone return at 10-12 weeks.
d- Single level injection should be used.
e- Injection are followed by intensive physiotherapy.
56- Multiple neurofibromatosis
a- Transmitted by autosomal dominant inheritance.
b- Patients develop numerous skin nodules.
c- The tumor cannot be completely separated from nerve.
d- Malignant transformation occur in 5-10 %.
e- On cross-section, the lesion consist of red fibrous tissue with nerve element.
57- Glomus tumors
a- Affect the nail bed.
b- Affect the children.
c- Cause pain.
d- Had local tenderness.
e- Is sensitive to cold.
58- Synovial sarcoma
a- Pain is a common feature.
b- X-rays shows extensive calcification.
c- Patients usually presented with swelling.
d- Histological examination shows fibroblastic tissue with foamy histiocyte and
multinucleated giant cells.
e- Small well define lesion can be treated by local excision.
59- Subluxation of the hip in cerebral palsy
a- Occurs in about 50 per cent of children with cerebral palsy.
b- Correction of flexion and adduction deformities before the age of 6 years may
have a role in preventing subluxation.
c- A persistent flexion-adduction deformity leads to femoral neck anteversion.
d- If the abductors are weak and the child is not fully weight bearing, there is a
risk of complete dislocation.
e- Older children may need varus derotation osteotomy of the femur.
60- Neural tube defects are associated with high levels of alpha-fetoprotein
a- In the amniotic fluid .
b- In the serum.
c- In the maternal urine.
d- Offers an effective method of antenatal screening during the 15th to 18th week
of pregnancy.
e- Maternal blood testing is performed routinely at 15–18 weeks and followed by
an amniocentesis if necessary.
61- In Myelomeningocele
a- The best predictor of walking ability and function in myelomeningocele is the
motor level of the paralysis.
b- Children with lesions below L4 will have quadriceps control and active knee
extension and should be encouraged to walk.
c- Children with higher lesions may start walking with the aid of orthotic devices
but they are likely to opt for a wheelchair with time.
d- Immobilization and muscle imbalance both lead to joint deformity and the risk
of pathological fracture.
e- All treatment, including surgery, must be conducted in latex-gloves.
62- In poliomyelitis
a- Sensation is intact.
b- There is sensory loss and paralysis.
c- The paralyzed limb is often cold and blue.
d- Large chilblains sometimes develop.
e- Sympathectomy may be needed.
63- The keys to successful treatment of hip disorders in poliomyelitis are:
a- To reduce any scoliotic pelvic obliquity by correcting or improving the
scoliosis.
b- To overcome or improve the muscle imbalance by suitable tendon transfer.
c- To correct the proximal femoral deformities by intertrochanteric or
subtrochanteric osteotomy.
d- To deepen the acetabular socket, if necessary, by an acetabuloplasty.
e- Fixed adduction of the hip can be treated by Soutter’s muscle slide operation.
64- Diabetic neuropathy
a- Is one of the commonest peripheral neuropathy.
b- Autonomic dysfunction and vascular disturbance also play a part.
c- There is a decreased susceptibility to nerve entrapment syndromes.
d- Late feature is loss of balance.
e- In advanced cases neuropathic ulcers of the feet may developed.
65- Treatment of acute pain is directed
a- Removing or counteracting the painful disorder.
b- Mobilizing the painful area.
c- Making the patient feel comfortable and secure.
d- Administering analgesics, anti-inflammatory drugs or – if necessary – narcotic
preparations.
e- Alleviating anxiety.
66- Acute nerve compression
a- Causes numbness and tingling within 15 minutes.
b- Causes loss of pain sensibility after 30 minutes.
c- Causes muscle weakness after 90 minutes.
d- Results in local intraneural transient anoxia.
e- Leaves no trace of nerve damage after decompression.
67- Neurapraxia
a- Is a reversible physiological nerve conduction block.
b- There is a segmental demyelination.
c- Followed by spontaneous recovery after a few days or weeks.
d- Is due to mechanical pressure.
e- The term means, axonal interruption.
68- For tendon transfer in nerves injuries
a- The patient should be motivated and cooperative.
b- The transferred tendon should route deeply in tissues.
c- The transferred tendon should be capable of firm fixation.
d- The recipient site should be stable and mobile.
e- The donor muscle should be expendable e and powerful.
69- Low ulnar nerve lesions
a- There is numbness of the ulnar one and a half fingers.
b- The hand assumes a typical posture of claw hand deformity.
c- Hypothenar and interosseous wasting may be obvious by comparison with the
normal hand.
d- Are often caused by cuts on shattered glass
e- Caused by compression or entrapment of the nerve in the medial epicondylar
(cubital) tunnel.

70- In ulnar nerve injury


a- Grip strength diminished.
b- Pinch is poor.
c- Fine, coordinated finger movements are also affected.
d- Metacarpophalangeal extension can improved by extensor carpi radialis
longus to intrinsic tendon transfers.
e- Index abduction improved by transferring extensor indicis to the interosseous
insertion on the radial side of the finger.
71- Complete injury of sciatic nerve
a- There is hamstrings paralysis.
b- There is paralysis of all muscles below the knee.
c- There is absent ankle jerk.
d- Sensation is lost below the knee.
e- Is rare.
72- Indications for sciatic nerve exploration are
a- Total sciatic palsy.
b- Partial lesions affecting common peroneal part.
c- A partial lesion associated with severe burning pain.
d- Strong evidence of a local cause such as a bone fragment, acrylic cement.
e- Strong evidence of a reversible cause such as hematoma near the nerve.
73- The risk of exposure to radiation.
a- The dose limit for the public is one mSv per year.
b- Is great for surgeon, because of repeated exposure.
c- The effect is accumulative.
d- For hip procedure, lead aprons will reduce the effective dose received by a
factor of 4- 16.
e- The thyroid shield decrease the risk by five time.
74- Complications of tourniquet usage
a- Nerve injury due to ischemia.
b- Skin burns.
c- Failure to diagnose peripheral vascular disease before surgery.
d- The risk minimized, by always using a wide cuff.
e- The risk minimized, by sealing the cuff against seeping fluids.
75- The drapes should be.
a- Barrier effectiveness throughout the length of the procedure.
b- Configurability to cover different areas of body or limb.
c- Not tear- resistance and does lint.
d- Not cause for skin reactions through allergy or abrasiveness.
e- Reasonable cost.
76- Post-thrombotic syndrome.
a- Presents with leg discomfort.
b- Presented with leg wasting.
c- Presented with skin changes and even ulceration.
d- This debilitating condition directly influences quality of life.
e- Approximately one-third of patients with symptomatic DVT will develop
features of Post-thrombotic syndrome within 2 years.
77- Bone allografts
a- Can be stored in a bone bank.
b- The potential for transfer of infection is either from contamination at the time
of harvesting or from diseases present in the donor.
c- The graft must harvested under sterile conditions and the donor must cleared
for malignancy, syphilis, cytomegalovirus, hepatitis and HIV.
d- Sterilization of the donor material can b done by ionizing radiation without
changes on property of the graft.
e- Antigenicity can be reduced by freezing, freeze-drying or by ionizing
radiation.
78- Demineralization of bone allografts
a- Is a way of reducing antigenicity.
b- Contains collagen and growth factors.
c- It may also enhance the osteoinductive properties of the graft.
d- It is available in a variety of forms and is sometimes combined with other
types of bone substitutes.
e- The osteoinductive capability is uniform.
79- Bone morphogenetic proteins (BMPs)
a- BMPs are osteoconductive.
b- BMPs are osteoinductive.
c- Used with a carrier, which may demineralized bone matrix, or bioactive bone
cement.
d- BMP-2 and BMP-7 are now manufactured using recombinant techniques.
e- BMP-2 and BMP-7 are available commercially.
80- Bone marrow aspirates
a- Contains stem cells and osteoprogenitor cells.
b- The number of osteoprogenitor cells in aspirates from the iliac crest decreases
with age.
c- Centrifugation of the aspirate has provided encouraging results and this also
may be the optimal method for using in humans.
d- The recommended procedure is to take multiple small-volume aspirates (four
1 mL aspirates from separate site punctures).
e- The osteoconductive capability is variable.
81- Thoracic outlet syndrome
a- Neurovascular symptoms and signs in the upper limbs produced by
compression of the upper trunk of the brachial plexus and subclavian vessels
between the clavicle and the first rib.
b- The subclavian artery and lower brachial trunk pass through a triangle based
on the first rib and bordered by scalenus anterior and medius.
c- Are congenital.
d- Symptoms are rare before the age of 30.
e- The neurovascular structures made taut when the shoulders braced back.
82- The differential diagnosis of thoracic outlet syndrome
a- Ulnar nerve compression.
b- Median nerve compression.
c- Cervical spondylosis.
d- Pancoast's syndrome.
e- Rotator cuff lesions.
83- Tarsal tunnel syndrome
a- Pain and sensory disturbance over the plantar surface of the foot.
b- Due to compression of the posterior tibial nerve behind and below the medial
malleolus.
c- Symptoms is often worse at walking.
d- Symptoms precipitated by prolonged weight bearing.
e- Nerve conduction studies may show slowing of motor or sensory conduction.
84- Manipulation under general anesthesia for frozen shoulder
a- There is a risk of fracturing the neck of the humerus in elderly.
b- May relief pain
c- Always improve the range of movement.
d- The shoulder moved gently into external rotation, then abduction and flexion.
e- At the end, the joint injected with methylprednisolone and lignocaine.
85- Shoulder joint instability
a- Is an abnormal symptomatic motion.
b- Is a degree of translation which falls within a physiological range
c- Is complex.
d- Results in pain.
e- Results in subluxation or dislocation of the joint.
86- Recurrent subluxation of shoulder
a- Symptoms and signs of recurrent subluxation are less obvious.
b- The patient may describe a ‘catching’ sensation
c- There is pain with the arm in adduction.
d- On examination, between episodes of dislocation, the shoulder looks normal
and movements are full.
e- Clinical diagnosis rests on provoking subluxation by the apprehension test.
87- The Hill–Sachs lesion is
a- Seen by an anteroposterior x-ray with the shoulder externally rotated.
b- Seen by an anteroposterior, x-ray with the shoulder internally rotated.
c- Seen by an axillary view.
d- Indentation on the posterolateral aspect of the humeral head.
e- A compression fracture due to the humeral head bring forced against the
anterior rim of glenoid, each time it dislocated.
88- The Putti–Platt operation,
a- It is now not commonly used.
b- The subscapularis is overlapped and shortened.
c- Prevents redislocation but at the cost of significant loss of external rotation.
d- Indicated for repeated painful dislocation of shoulder.
e- The coracoid process with its attached muscles transposed to the front of the
neck of the scapula.
89- Atraumatic structural shoulder instability
a- Is a recognized problem in athletes.
b- The labrum and anterior capsule are detached
c- Develop symptoms of instability due to overload and fatigue in the stabilizing
muscles of the shoulder.
d- Dislocation may occur in several different directions.
e- It is important to rule out the presence of any pathological condition,
90- Posterior dislocation of shoulder
a- Is common, and when it occur it is commonly missed.
b- Usually due to a violent jerk in an unusual position.
c- Following an epileptic fit.
d- Following severe electric shock.
e- May be associated with fractures of the proximal humerus.
91- Tuberculosis of the shoulder
a- Is uncommon.
b- It usually starts as an otitis.
c- May proceed to abscess and sinus formation.
d- Some cases the tendency is to fibrosis and ankylosis‘ caries sicca’.
e- Is a cause of frozen shoulder.
92- Rheumatoid arthritis of shoulder
a- Is the most common arthropathy to affect the shoulder complex.
b- Fifty per cent of patients with rheumatoid arthritis have involvement of the
acromioclavicular joint.
c- Cartilage destruction and bone erosion are often severe.
d- May go on to capsular disruption and instability.
e- The various synovial pouches around the shoulder may be affected.
93- Osteoarthritis of the gleno-humeral joint
a- Is more common than is generally recognized.
b- The patient is usually aged 50–60.
c- May give a history of injury.
d- It is usually secondary to local trauma.
e- There is usually little to see and shoulder movements are free.
94- Rapidly destructive arthropathy of shoulder
a-Called a cuff tear arthropathy.
b-Similar condition seen in other joints.
c-Attributed to pyrophosphate crystal shedding from the torn rotator cuff.
d-Synovial reaction involving the release of lysosomal enzymes that lead to
cartilage breakdown.
e- Known as Milwaukee shoulder.

95- Osteonecrosis of humeral head


a- Shoulder is the most common site of steroid-induced osteonecrosis.
b- The condition seen in association with marrow storage disorders, sickle-cell
disease and caisson disease.
c- May follow irradiation of the axilla.
d- Articular collapse occurs more slowly than in weight bearing joints.
e- Operative treatment can delayed for several years.
96- Winging of the scapula is due
a- Direct damage to the long thoracic nerve.
b- Fascioscapulohumeral muscular dystrophy.
c- Congenital lesions.
d- Neuralgic amyotrophy.
e- Injury to the brachial plexus.
97- Congenital dislocation of the radial head
a- May be anterior.
b- May be posterior.
c- Is usually bilateral.
d- Cause painful limitation of elbow movement.
e- X-rays show that the dislocated radial head is dome-shaped.
98- Cubitus valgus
a- If carrying angle more than 15°.
b- It is obvious when the patient stands with arms to the sides and palms facing
forwards.
c- The commonest cause is longstanding non-union of a fractured lateral
condyle.
d- The importance of it, is the liability to delayed ulnar palsy.
e- The deformity itself needs treatment.
99- Pulled elbow
a- Is a common injury in children under the age of 6 years.
b- The limb held immobile with the elbow fully extended and the forearm
pronated.
c- Attempt to supinate the forearm is resisted.
d- The diagnosis is essentially radiological .
e- Reduced by quick supination and then slightly flexion; the radial head is
relocated with a snap.
100- Loose bodies in the elbow may be due to
a- Acute trauma.
b- Rheumatoid arthritis
c- Osteochondritis dissecans.
d- Synovial chondromatosis .
e- Osteoarthritis (separation of osteophytes).
101- Lateral epicondylitis may results in
a- Small tears.
b- Fibrocartilaginous metaplasia.
c- Avulsion of common extensor origin.
d- Microscopic calcification.
e- A painful vascular reaction in the tendon fibers close to the lateral epicondyle.
102- Avulsion of the distal biceps tendon
a- The typical patient is a woman of about 45 years.
b- The typical patient feels sudden pain and weakness at the front of the elbow
after strenuous effort.
c- There is a loss of supination power with the elbow flexed.
d- Can feel the distal biceps tendon while the patient flexes the elbow against
resistance.
e- MRI helps to confirm the diagnosis.
103- Arthroscopy of the elbow indicated for
a- Removal of loose bodies.
b- Irrigation for infection.
c- Trimming of osteophytes.
d- Radial head arthroplasty.
e- Synovectomy.
104- The elbow joint permitting
a- A few degrees of valgus movement.
b- A few degrees of varus movement.
c- Some rotational laxity.
d- 30–130 degrees of flexion.
e- 90 degrees of both pronation and supination.
105- Stability of elbow provided by.
a- The relative conformity of the humeral trochlea with the olecranon.
b- The medial collateral ligament (particularly the anterior band).
c- The lateral collateral ligament) particularly the ulnar part).
d- The radial head is a secondary constraint to valgus instability.
e- The biceps tendon.
106- The wrist MRI is particularly useful for detecting changes associated with
a- Scaphoid fractures.
b- Avascular necrosis of the lunate (Kienböck’s disease).
c- The luno-triquetral ligament injuries.
d- The occult dorsal ganglia.
e- The intra-osseous ganglia.
107- Early surgical reconstruction of syndactyly indicated, if there is
a- Thumb affection.
b- Index affection.
c- Incomplete syndactyly of central finger.
d- Ring finger affection.
e- Little finger affection.
108- Instability of the distal radio-ulnar joint treated by
a- Reattachment of the triangular fibrocartilage complex.
b- Ulnar shortening osteotomy.
c- Reconstruction of ligament by tendons.
d- Excision of the ulnar head.
e- Ulnar head implants.
109- Dorsal intercalated segment instability of wrist
a- Following fracture of the scaphoid.
b- Following rupture of scapho-lunate ligament.
c- Following rupture of luno-triquetral ligament.
d- The scaphoid tend to flex.
e- The lunate tend to extend.
110- Overgrowth of the finger seen in
a- Static macrodactyly.
b- Multiple exostosis.
c- Multiple enchondromatosis.
d- Neurofibromatosis.
e- Vascular malformation.
111- Madelung's deformity
a- The lower radius curved ventrally.
b- Always appear as an isolated entity.
c- The lower end of ulna prominent dorsally.
d- May be congenital.
e- Rarely appear before age of 10 years.
112- Progressive destruction of the distal radio-ulnar joint
a- Is a characteristic feature of severe rheumatoid arthritis.
b- Lesser degenerative changes seen in secondary osteoarthritis.
c- Possibly following marked instability of the joint.
d- Possibly following longstanding instability of the joint.
e- Late stage treated by arthroscopic radial styloidectomy.
113- DeQuervian's disease
a- The patient is usually a woman aged 40–50.
b- The patient complains of pain on the radial side of the wrist.
c- There may be a history of unaccustomed activity.
d- There is a visible swelling over the ulnar styloid and the tendon sheath feels
thick and hard.
e- Tenderness is most acute at the very tip of the radial styloid.
114- Flexor carpi radialis tendinitis
a- Causes pain on the front of the wrist alongside the scaphoid tubercle.
b- Symptoms reproduced by resisted wrist flexion.
c- Tenderness is sharply localized and should be distinguished from that of de
Quervain’s disease or
d- Should be distinguished from osteoarthritis of the basal joint of the thumb.
e- Patients had tenderness and crepitus on the dorsum of the wrist.
115- The ganglion cyst
a- Is the most common swelling in the wrist.
b- Contains a glairy, viscous fluid.
c- Usually develops on the volar surface of the scapho-lunate ligament.
d- Occur in young adults, presents with a painless lump.
e- Presented as a well-defined lump, cystic and not tender.
116- Compound palmar ganglion
a- Chronic inflammation distends the common sheath of the flexor tendons both
above and below the flexor retinaculum.
b- Rheumatoid arthritis and tuberculosis are the commonest causes.
c- The swelling is hourglass in with cross fluctuation.
d- If the condition is tuberculous, the entire flexor sheath dissected out.
e- Complete excision is the best treatment when the cause is rheumatoid disease.
117- The median nerve supplies
a- The abductor pollicis brevis.
b- The adductor pollicis.
c- The opponens pollicis.
d- The lumbrical to the middle finger.
e- The lumbrical to the index.
118- The ulnar nerve supplies
a- The Hypothenar muscles.
b- All the interossei muscles.
c- Lumbricals to the little and ring fingers.
d- Adductor pollicis.
e- The abductor pollicis brevis.
119- Rupture of extensor pollicis longus
a- Occur after a Colles’ fracture, or in rheumatoid arthritis.
b- It is due to interruption or stretching of the central slip of the extensor tendon.
c- The thumb cannot actively elevated backwards above the plane of the hand.
d- Direct repair is unsatisfactory.
e- Treated by a tendon transfer, using the extensor indicis.
120- Septic arthritis of MCP joints
a- May infected directly penetrating injuries.
b- May infected by intra-articular injection.
c- May infected from adjacent structures.
d- May infected directly by haematogenous spread.
e- May infected by a 'fight bite'.
121- Mucous cyst
a- Develop between extensor tendon and collateral ligament of DIP joint.
b- Develop inside DIP joint.
c- Cause nail deformity.
d- Occasionally ulcerated.
e- Treated by excision of cyst with underling osteophyte.
122- Extensor tendon rupture in wrist
a- Is common complication of chronic synovitis.
b- Extensor digiti minimi is usually the last to rupture.
c- May be treated by tendon transfer.
d- May be treated by tendon graft.
e- Synovectomy may be necessary in treatment.
123- The clinical signs of flexor acute infective tenosynovitis in hand
a- Flexed posture of digit.
b- Tenderness along the course of tendon.
c- The finger is red and swollen.
d- Pain on passive finger extension.
e- Pain on active extension.
124- The following conditions must be excluded in finger infection
a- An insect bite or sting.
b- A thorn prick.
c- Acute tendon rupture.
d- Acute gout.
e- Synovitis of rheumatoid arthritis.
125- Swan neck deformity of finger occur
a- After mallet finger.
b- Following volar subluxation of the MCP joint.
c- Following division of flexor digitorum superficialis.
d- Following division of flexor digitorum profundus.
e- After failure of palmar plate of the PID, joint in rheumatoid arthritis.
126- Dupuytren's contracture
a- Inherited as autosomal dominant trait.
b- Is more common in male.
c- Is associated with diabetes.
d- Is associated with alcoholic liver cirrhosis.
e- There is strong association with injury to the hand.
127- Surgical treatment of Dupuytren's contracture
a- Indicated if the deformity is rapidly progressive.
b- Indicated if the PIP joints contractures become irreversible.
c- Surgery dose not cure the disease.
d- Correction of the MCP joints is less predictable than the PIP joints.
e- Recurrence is common.
128- Secondary torticollis may be secondary to
a- Congenital atlanto-axial rotatory displacement.
b- Infection (lymphadenitis, retropharyngeal abscess or tuberculosis).
c- Trauma.
d- Juvenile rheumatoid arthritis.
e- Sternomastoid injured at birth.
129- Klippel–Feil syndrome
a- Is a short neck and vertebral anomalies.
b- About one in three children with this syndrome also has Sprengel’s deformity.
c- Is associated with scoliosis in about 20 per cent.
d- Is associated with rib anomalies in about 30 per cent.
e- X-rays reveal fusion of two or more cervical vertebrae.
130- Neuralgic amyotrophy
a- This condition can closely resemble an acute disc prolapse.
b- There is no definite history of a strain episode.
c- The neck pain is unrelenting with sever local spasm.
d- Pain is sudden and severe, and situated over the shoulder.
e- Careful examination will show that more than one neural level is affected.
131- Cervical spondylosis
a- Cause neck pain.
b- Cause paraesthesia, weakness, and clumsiness in the arm and hand.
c- Neck movements are limited.
d- Pain referred to intrascapular area and upper limb.
e- Pain and paraesthesia are worse at night.
132- The indication for operative treatment in cervical spine tuberculosis are
a- To drain a retropharyngeal abscess.
b- To decompress a threatened spinal cord.
c- To fuse an unstable spine.
d- To correct and prevent spinal deformity.
e- To correct narrowing of the disc space.
133- In postural scoliosis
a- There is a triplanar deformity with lateral, anteroposterior and rotational
components.
b- The deformity is secondary to short leg.
c- The deformity is compensatory to pelvic tilt due to contracture of the hip.
d- When the patient sits, the curve disappears.
e- When patient bends forwards, the deformity disappears.
134- Idiopathic adolescent thoracic scoliosis is
a- Ninety per cent female.
b- Ninety per cent convex to right.
c- Rib rotation exaggerates the deformity.
d- Fifty per cent develop curves of greater than 70°.
e- Not noticed early, but cause backache in adult life.
135- Risser’s sign ( ossification of iliac apophyses)
a- The iliac apophyses start ossifying shortly after puberty.
b- Ossification extends from lateral to medial.
c- Used to estimate skeletal age.
d- Once the iliac crests completely ossified, further progression of the scoliosis is
minimal.
e- Usually coincides with fusion of the vertebral ring apophyses.
136- Structural scoliosis with severe chest deformity
a- Pulmonary function tests performed in all cases.
b- Associated with marked reduction in vital capacity.
c- Associated with diminished life expectancy.
d- Carries obvious risks for surgery.
e- In prepubertal children, slow progression is liable to occur during the growth
spurt.
137- The Milwaukee brace
a- Is a snug-fitting underarm brace that provides lumbar or low thoracolumbar
support.
b- Is principally a thoracic support consisting of a pelvic corset connected by
adjustable steel supports to a cervical ring carrying occipital and chin pad.
c- Its purpose is to reduce the lumbar lordosis and encourage active stretching
and straightening of the thoracic spine.
d- A well- made brace worn 23 hours out of 24 and does not preclude full daily
activities, including sport and exercises.
e- Used for the treatment of progressive scoliotic curves between 20 and30
degrees.
138- Spinal cord function monitored during the operation of scoliosis
a- By measuring somatosensory potentials during spinal correction.
b- By measuring motor evoked potentials during spinal correction.
c- By the ‘wake-up test’.
d- Patients have memory of the wake-up procedure.
e- If there are signs of cord compromise, the instrumentation is relaxed or
removed and reapplied with a lesser degree of correction.
139- The osteopathic ( congenital ) scoliosis
a- May result from hemivertebra.
b- May result from wedged vertebra.
c- May result absent or fused ribs (failure of segmentation).
d- Is often mild, some cases progress to severe deformity.
e- Treatment is easier than that of idiopathic infantile scoliosis.
140- Scoliosis and neurofibromatosis
a- Scoliosis occur in one-third of patients with neurofibromatosis.
b- Accompanied by skin lesions, and multiple neurofibromata.
c- The scoliotic curve is typically ‘short and sharp’.
d- Mild cases treated correction and arthrodesis.
e- Severe deformities need combined anterior and posterior instrumentation and
fusion.
141- Structural kyphosis
a- Is fixed and associated with changes in the shape of the vertebrae.
b- Is voluntarily correctable.
c- May be due to congenital vertebral defects.
d- Seen in skeletal dysplasias such as achondroplasia and in osteogenesis
imperfecta.
e- Older children may develop severe deformity secondary to tuberculous
spondylitis.
142- Congenital kyphosis, Type II (failure of segmentation)
a- This is the commonest (and the worst) type.
b- Usually takes the form of an anterior intervertebral bar.
c- The spine gradually becomes kyphotic.
d- The risk of neurological compression is much less.
e- If the curve is progressive, a posterior fusion needed.
143- In lateral radiographs of the spine in adolescent kyphosis
a- The vertebral endplates of several adjacent vertebrae (usually T6–10) appear
irregular or fragmented.
b- The changes are more marked anteriorly and one or more vertebral bodies may
become wedge shaped.
c- There may also be small radiolucent defects in the subchondral bone)
Schmorl’s nodes).
d- The anterior part of the vertebral body fails to develop.
e- Wedging of more than 5 degrees in three adjacent vertebrae and an overall
kyphosis angle of more than 40 degrees are abnormal.
144- Senile osteoporosis
a- Affects both men and women.
b- Patients are usually over 75 years of age, often incapacitated by some other
illness, and lacking exercise.
c- They complain of back pain, and spinal deformity may be marked.
d- X-rays reveal multiple vertebral fractures.
e- It may result from metastatic disease or myelomatosis.
145- The feature of cauda equine syndrome
a- Bladder and bowel dysfunction.
b- Perineal numbness.
c- Unilateral sciatica.
d- Lower limb weakness.
e- Cross straight leg raising sign.
146- Persistent postoperative backache and sciatica:
a- May be due to residual disc material in the spinal canal.
b- May be due to prolapse at another level.
c- May be due to nerve root pressure by a Hypertrophic facet joint or a narrow
lateral recess (‘root canal stenosis).
d- May be due to inflammatory disorders.
e- May call for re-operation; but second procedures do not have a high success
rate.
147- Arachnoiditis
a- Cause diffuse back pain and vague lower limb symptoms such as ‘cramps’,
‘burning’ or ‘irritability’.
b- Sometimes appear after myelography, epidural injections
c- May follow disc operations.
d- There may also be sphincter dysfunction and male impotence.
e- Treatment is generally rewarding.
148- ‘Instability 'of lumbar spine is
a- Relieved by laying down.
b- Relieved by resting.
c- Relieved by sitting.
d- Relieved by recumbency.
e- Relieved by bending.
149- Dysplastic spondylolisthesis
a- Appears at an early age.
b- Progresses slowly and seldom exceeds 30 percent.
c- Carried a significant risk of neurological complication.
d- Associated anomalies (usually spina bifida occulta) are common.
e- The superior sacral facets are congenitally defective; leads to severe
displacement.
150- Operative decompression of spinal stenosis
a- Is almost always successful.
b- A large laminotomy with flavectomy, medial facetectomy and discectomy
performed.
c- If necessary extending over several levels and laterally to clear the nerve root
canals.
d- This relieves the back pain, but not the leg pain.
e- In patients under 60, the operation sometimes combined with spinal fusion.
151- Back strain
a- Is transient backache following muscular activity?
b- Characterized by neurological symptoms and signs.
c- Is common cause of backache in young adults.
d- Respond to a short period of rest followed by gradually increasing exercise.
e- People with thoracic kyphosis or fixed flexion of the hip, are particularly
prone to back strain.
152- Severe and constant pain localized to a particular site of spine
a- This suggests local bone pathology, such as a compression fracture.
b- This suggests local Paget’s disease, or a tumor.
c- This suggests local infection.
d- Spinal osteoporosis in middle-aged men is pathological and calls for a tests to
myelomatosis.
e- The patients are usually aged over 60 years.

153- Spinal fusion in facet joint instability


a- A ‘fusion 'does not always fuse (there is a 10–20 per cent failure rate).
b- The type of approach and the type of implant affect the long-term results.
c- A fusion at one level does not preclude further pathology developing at
another level.
d- In a long-term follow-up, 40 per cent had developed signs of instability
elsewhere.
e- The more extensive and more complex procedures carry a higher complication
rate.
154- Psychosomatic ailments of chronic backache
a- Pain and tenderness of bizarre degree or distribution.
b- Pain on performing impressive but non-stressful manoeuvres.
c- Variations in response to tests such as straight leg raising while distracting the
patient’s attention.
d- Sensory and/or motor abnormalities that do not fit the known anatomical and
physiological patterns.
e- Underdetermined behavior during physical examination, usually accompanied
by low groaning and exclamations of discomfort.
155- The Patients with chronic backache and psychosomatic ailments
a- Patients with these features are unlikely to respond to surgery.
b- They may require prolonged support.
c- Posterior intertransverse fusion in situ is almost always successful
d- They may require prolonged management in a special pain clinic.
e- All effort must be made to exclude organic pathology.
156- Pain arising in the hip joint
a- Felt in the groin.
b- Felt in the front of the thigh.
c- Felt in the knee.
d- Felt in groin and knee.
e- Felt in the back of the hip.
157- The causes of a positive Trendelenburg sign are
a- Lateral tilt of lumbar spine.
b- Pain on weight bearing.
c- Weakness of the hip abductors.
d- Shortening of the femoral neck.
e- Dislocation or subluxation of the hip.
158- Neonatal screening for DDH in newborn infants
a- Ideally, all neonates should be examined.
b- Led to a marked reduction in missed cases of DDH.
c- Risk factors taken into account in selecting newborn infants for special
examination and ultrasonography.
d- Screening carried out by ultrasonography and x-rays.
e- If the programme is to be effective, the examiner should receive special
training.
159- bilateral dislocation of the hip
a- The deformity is symmetrical and therefore not so noticeable.
b- The risk of operative intervention is also greater.
c- Most surgeons avoid operation above the age of 8 years
d- Reduction and stabilization become increasingly difficult with advancing age.
e- The untreated patient walks with a waddle but may be surprisingly
uncomplaining.
160- Avascular necrosis of the hip after DDH treatment
a- Is the most serious complication of the immature femoral head.
b- It may occur at any age and any stage of treatment.
c- Is probably due to vascular injury or obstruction resulting from forceful
reduction and hip splintage in abduction.
d- The effects vary considerably.
e- To prevent this complication, positions of extreme adduction must be avoided.
161- Protrusion acetabuli
a- The hip socket is too deep and bulges into the cavity of the pelvis.
b- Affect males more than female.
c- Cause limitation of hip movement.
d- Secondary OA may develop later in life.
e- May occur in late life secondary to bone softening disease.
162- Acquired dislocation of hip in children
a- Dislocation occurring after the first year of life.
b- May be due to pyogenic arthritis.
c- May be due to muscle imbalance.
d- May be due to rheumatoid arthritis.
e- May be due to trauma.
163- Dislocation of the hip due to muscle imbalance
a- Results from hip abductors being weaker than the adductors.
b- May result in partial or complete destruction of the largely cartilaginous
femoral head.
c- The fetal anteversion of the femoral neck persists.
d- The greater trochanter fails to develop properly.
e- The femoral neck becomes valgus.
164- In children between 3 and 10 years, the intoeing gait
a- The cause is usually femoral anteversion.
b- The gait may look clumsy but usually improves with growth.
c- Femoral neck anteversion can also be assessed by ultrasonography
d- Children often sit on the floor in the ‘television position’ with the knees facing
each other.
e- Shoe modifications and orthotics are necessary.
165- Acquired coxa vara in childhood
a- Is seen in rickets.
b- Is seen after Perthes' disease.
c- May follow epiphysiolysis.
d- May result from osteoprosis.
e- Lead to mechanical disadvantage during weight bearing.
166- Irritable hip is
a- Is common cause of hip pain in children.
b- Is a nonspecific, short-lived synovitis.
c- Girls affected twice as often as boy.
d- There is synovial effusion of the hip.
e- The groin pain last for 1-2 weeks and subside spontaneously.
167- In slipped capital femoral epiphysis
a- The age of patients between 10-16 years
b- The leg is externally rotated.
c- The leg is shorter.
d- There is limitation of flexion, abduction and internal rotation.
e- There is tendency of internal rotation as the hip flexed.
168- Pyogenic arthritis of the hip
a- Is usually seen in children over 2 years of age.
b- Adults, also, may develop pyogenic hip infection
c- The organism (usually a staphylococcus) reaches the joint either directly from
a distant focus or by local spread from osteomyelitis of the femur.
d- Unless the infection is rapidly aborted, the femoral head is liable to be
destroyed by the proteolytic enzymes of bacteria and pus.
e- The child is ill and in pain and all attempts at moving the hip are resisted.
169- Tuberculosis of the hip
a- Starts insidiously with aching in the groin and thigh, and a slight limp.
b- With early disease joint held flexed and adducted, and all movement are
restricted and painful.
c- Later, pain is more severe and may wake the patient from sleep.
d- Later, the muscle wasting becomes obvious, and all movements are grossly
limited by pain and spasm.
e- The earliest x-rays change is general rarefaction.
170- Total joint replacement of the hip in older patients with residual tuberculosis
a- Indicated if there is pain and deformity.
b- Indicated if the disease has clearly been inactive for a considerable time.
c- Is feasible and often successful.
d- May be necessary once all signs of activity have disappeared, but usually not
before the age of 14.
e- With anti-tuberculous drugs, the chances of recurrence are not great.
171- The pincer of femoro-acetabular impingement
a- The typical patients is adult female.
b- The level of deformity is proximal femur.
c- The pathological anatomy is deep socket.
d- The first radiological changes is postero- inferior joint space narrowing.
e- The progression is slow.
172- Open osteochondroplasty in femoro- acetabular impingement
a- Allow debridement of damage cartilage.
b- Allow resection of impediments that responsible for the malarticulation.
c- The morbidity of the procedure is low.
d- The short to mid-term results are good to excellent.
e- For cam type, the anterior rim of the acetabulum must be removed.
173- MR arthrography of the hip in femoro-acetabular impingement
a- Used routinely to visualize the labrum.
b- Used routinely to visualize articular cartilage.
c- Capable to detect abnormal sphericity.
d- Capable to detect impingement cysts.
e- Is sensitive and but not specific.
174- Indication for operative treatment in OA of the hip
a- Progressive increase in pain.
b- Severe restriction of activities.
c- Marked deformity.
d- Restriction of adduction.
e- X-ray sign of joint destruction.
175- The abnormal stress cause OA of the hip in
a- Subluxation.
b- Coxa magna.
c- Coxa vara.
d- Protrusion-acetabuli.
e- Paget's disease.
176- Non-traumatic osteonecrosis of the hip
a- The patients walk with limp.
b- The thigh is wasted and the limb is shorter.
c- Abduction and internal rotation is restricted.
d- There is tendency for hip to twist into internal rotation during passive flexion.
e- When the hip extended, internal rotation is almost full.
177- Grade II osteonecrosis of the hip
a- Are liable to progress.
b- If there is no distortion of femoral head, core decompression is indicated.
c- 30-50% of patient improved after decompression.
d- Treated by total hip replacement in patients over 45 years old.
e- Treated by realignment osteotomy in older age group.
178- The contraindication of hip arthrodesis are
a- Elderly patients.
b- Destructive condition of the hip articular surface.
c- Hip with good range of movement.
d- Lack of bone stock.
e- Abnormality in the compensating joints.
179- The cause of pain and tenderness over the greater trochanter in adults
a- Stress fracture.
b- Trochanteric bursitis.
c- Trochanteric tuberculosis.
d- Slipped proximal femoral epiphysis.
e- Referring pain from lumber spine.
180- The objective of intertrochanteric osteotomy.
a- To improve range of movement.
b- To reduce mechanical stress in the damaged segment.
c- To improve joint congruity.
d- To reduce intraosseous pressure.
e- To improve fibrocartilage repair of articular surface.
181- The prosthetic implants must be
a- Durable.
b- Brittle.
c- Permit extraordinary low friction movement at the articulation.
d- Firmly fixed to the skeleton.
e- Inert and not provoke any unwanted reaction in the tissues.
182- The ways to enhancing osseointegration are
a- Applied bone cement under pressure.
b- Use smooth, tapered and collarless femoral prosthesis.
c- To use the ceramic components, which have better frictional characteristics.
d- In uncemented, implants may be covered with a mesh or porous coating.
e- The implant may be coated with hydroxyapatite.
183- Factors that may contribute to the development of complications in total hip
replacement are
a- Age of patients.
b- Previous hip operations.
c- Severe deformity and inadequate bone stock.
d- Lack of preoperative planning.
e- Insufficiently sterile operating environment and lack of experience or expertise
on the part of the surgical team.
184- Heterotopic bone formation around the hip following arthroplasty
a- Is seen in about 20 per cent of patients 5 years after joint replacement.
b- There are a high risk in skeletal hyperostosis and ankylosing spondylitis.
c- In severe cases, this is associated with painless stiffness.
d- Ossification can be prevented in high-risk patients by giving either a course of
non-steroid anti-inflammatory drugs for 3–6 weeks postoperatively.
e- Ossification can be prevented in high-risk patients by giving a single dose of
irradiation to the hip.
185- Infection following total hip arthroplasty
a- Is the most serious postoperative complication.
b- With adequate prophylaxis, the risk should be less than 5 per cent.
c- The large bulk of foreign material restricts the access of the body’s normal
defence mechanism; consequently, even slight wound contamination may be
serious.
d- The results of revision arthroplasty using gentamicin-impregnated cement are
only moderately good.
e- Even many years later, haematogenous spread from a distant site may cause
late infection.
186- Hip arthroplasty implant selection
a- The selected implant will function satisfactorily for the individual patient.
b- The objectives are to obtain durable fixation of both components.
c- The objectives are to obtain good orientation and to avoid instability.
d- Care must be taken to restore the offset and ensure that the limb lengths end
up equal.
e- Health economics dictate that– the lowest-cost implant should be used.
187- The use of uncemented hip arthroplasty implants
a- Has become increasingly popular over the past two decades.
b- The surface of these implants often textured to enhance soft tissue formation at
bone- implant interface.
c- It is important to have initial press-fit stability to allow bone on- or ingrowth
into the textured surface.
d- More recently bioactive surface coatings – such as hydroxyapatite – have been
applied to accelerate bone ingrowth and improve the extent of the
osseointegration.
e- Well-fixed uncemented hips provide a durable biological fixation which is
cyclically renewed with time.
188- Arthroscopy of knee is useful
a- To establish or refine the accuracy of diagnosis.
b- To establish diagnosis of lateral collateral injuries.
c- To help in deciding whether to operate, or to plan the operative approach with
more precision.
d- To record the progress of a knee disorder.
e- To perform certain operative procedures.
189- Adolescents with anterior knee pain are usually found to have
a- Chondromalacia patellae.
b- Congenital knee disorders.
c- Patellar instability.
d- Osteochondritis.
e- Plica syndrome.
190- In genu valgus, observation should be advised
a- By the age of 6 years, the intermalleolar distance is less than 6 cm.
b- By the age of 6 years, the intermalleolar distance is less than 8 cm.
c- By the age of 10 years, the intermalleolar distance is less than 4 cm.
d- By the age of 10 years, the intermalleolar distance is less than 6 cm.
e- By the age of 10 years, the intermalleolar distance is more than 8 cm.
191- Genu recurvatum may be
a- Congenital or associated with congenital dislocation of patella.
b- If bony correction is undertaken, the knee left with some flexion to preserve
the stabilizing mechanism.
c- Result of generalized joint laxity.
d- Result of prolonged traction, especially on a frame, or holding the knee
hyperextended in plaster.
e- Associated with paralytic conditions such as poliomyelitis.
192- The medial meniscus injuries is more common due to
a- Is much more mobile.
b- Is larger.
c- It cannot as easily accommodate to abnormal stresses.
d- Degeneration of the menisci with age, so splits and tears are more likely in
later life.
e- Associated arthritis or chondrocalcinosis made splits and tears are more likely.
193- bucket-handle tear of meniscus
a- The separated fragment remains attached front and back.
b- The torn portion sometimes displaces towards the center of the joint and
becomes jammed between femur and tibia, causing a block to extension
(‘locking’).
c- If the tear emerges at the free edge of the meniscus, it leaves a tongue based
anteriorly (an anterior horn tear).
d- The patient is usually a young person who sustains a twisting injury to the
knee on the sports field.
e- Swelling appears some hours later or perhaps the following day.

194- Differential diagnosis of meniscus injuries


a- Loose bodies in the knee joint.
b- Traction apophysitis of tibial tuberosity,
c- Recurrent dislocation of the patella.
d- Fracture of the tibial spine follows an acute injury and may cause a block to
full extension.
e- A partial tear of the medial collateral ligament.
195- Arthroscopic meniscectomy
a- Has distinct advantages of shorter hospital stay.
b- Has distinct advantages lower costs
c- Has distinct advantages more rapid return to function.
d- It is free of complications.
e- Prophylactic non-steroidal anti-inflammatory drugs reduce postoperative pain
and stiffness.
196- Meniscal cyst
a- Originate between capsule and meniscus.
b- Is multilocular cyst.
c- Contain gelatinous fluid.
d- Result from degeneration of meniscus.
e- Surrounded by thick fibrous tissue.
197- The predisposing factors for recurrent dislocation of patella are
a- Generalized ligamentous laxity.
b- Under- development of the medial femoral condyle and flattening of the
intercondylar groove.
c- Maldevelopment of the patella, which may be too high or too small.
d- Valgus deformity of the knee.
e- External tibial torsion.
198- The realignments operative treatment for recurrent dislocation of patella
a- Direct medial patello-femoral ligament repair.
b- Suprapatellar realignment.
c- Infrapatellar soft-tissue realignment.
d- Infrapatellar bony realignment.
e- Patellectomy.
199- The common cause of pain over the anterior aspect of the knee are
a- Bursitis.
b- Osgood–Schlatter disease.
c- Patellar bone tumor.
d- Plica syndromes.
e- Patello-femoral arthritis.
200- Patello-femoral pain syndrome basic disorder is due to either
a- Malcongruence of the patello-femoral surfaces.
b- A single injury may damage the articular surfaces.
c- Malalignment of the lower extremity
d- Malalignment of the patella.
e- Muscular imbalance with decreased strength of the vastus medialis.
201- The avascular necrosis of the femoral condyle
a- Usually associated with corticosteroid therapy.
b- May associated with alcohol abuse.
c- Seen in an older age group.
d- On x-ray, the lesion is always on the dome of the femoral condyle.
e- If the knee flexed to 90 degrees, rotated medially and then gradually
straightened, pain felt.
202- The early X-rays changes in knee tuberculosis is
a- Show marked osteoporosis.
b- In children, enlargement of the bony epiphyses.
c- Joint space narrowing.
d- Soft tissue knee swelling.
e- Soft tissue wasting of thigh.
203- Realignment osteotomy for knee osteoarthritis
a- Is often successful in relieving symptoms.
b- Is indicated in a ‘young’ patient with a varus knee.
c- Is indicated in osteoarthritis confined to the medial compartment.
d- A high tibial valgus osteotomy will redistribute weight to the medial side of
the joint.
e- The degree and accuracy of angular correction are the most important
determinants of mid- and long-term clinical outcome.
204- Differential diagnosis of osteonecrosis of the knee
a- Patello-femoral joint chondromalacia.
b- Osteochondritis dissecans.
c- A fracture of an osteoarthritic osteophyte,
d- Disruption of a degenerative meniscus.
e- A stress fracture.
205- Radiographic feature of hemophilic arthropathy is
a- Soft tissue swelling.
b- Local osteoporosis.
c- Joint space narrowing.
d- Subarticular bone fragmentation.
e- Large cyst or erosion may appear in the subchondral bone.
206- Posttraumatic haemarthrosis
a- Swelling immediately after injury.
b- The knee is very painful and it feels warm, tense and tender.
c- Later there may be a ‘doughy’ feel.
d- Movements are restricted.
e- X-rays are essential to see if there is a tear of the anterior cruciate ligament.
207- Traumatic synovitis
a- Injury stimulates a reactive synovitis.
b- Typically, the swelling appears only after some hours.
c- There is stimulation of quadriceps action and the thigh wastes.
d- The knee splinted for several days but quadriceps exercise is essential.
e- If the amount of fluid is considerable, its aspiration hastens muscle recovery.
208- The commonest causes of chronic knee swelling are
a- Gout and pseudogout.
b- Osteoarthritis.
c- Rheumatoid arthritis.
d- Synovial chondromatosis.
e- Pigmented villonodular synovitis.
209- The popliteal cyst is
a- Bulging of the posterior capsule and synovial herniation in the popliteal fossa.
b- The lump, which is usually seen in young people, is in the midline
c- It fluctuates but is not tender.
d- Nowadays it is more likely to be caused by rheumatoid or osteoarthritis.
e- It is still often called a ‘Baker’s cyst’.
210- Popliteal aneurysm
a- Is the commonest limb aneurysm.
b- Is sometimes bilateral.
c- Pain and stiffness of the knee may precede the symptoms of peripheral arterial
disease.
d- Is essential to examine any lump behind the knee for pulsation.
e- A thrombosed popliteal aneurysm pulsate, and it feels almost cystic.
211- Complication of arthroscopy is
a- Intra-articular effusions.
b- Small haemarthrosis.
c- Formation of loose body.
d- Infection.
e- Reflex sympathetic dystrophy.
212- The main indications for arthrodesis of the knee were
a- Advance seronegative arthritis of knee.
b- Irremediable instability due to the late effects of poliomyelitis.
c- Painful loss of mobility due to tuberculosis.
d- Painful loss of mobility in chronic pyogenic infection.
e- Failed total knee replacement (either septic or aseptic).
213- The main indication for knee replacement is
a- Pain, especially when this combined with instability.
b- Pain and instability due to late effect of poliomyelitis.
c- Pain, especially when this combined with deformity.
d- Rheumatoid arthritis.
e- Osteoarthritis.
214- Minimally constrained total knee replacement
a- Used for prostheses where some of the stability after replacement is provided
by the prosthesis and some through preservation of the knee ligaments.
b- Most modern minimally constrained designs allow sacrifice of the anterior
cruciate ligament.
c- Some of minimally constrained designs even allow both cruciates to be
removed.
d- Rarely used because results are poor.
e- At operation, all the articular surfaces replaced –with metal on the femoral
side.
215- Constrained total knee replacement
a- Artificial joints with fixed hinges
b- Used when there is marked bone loss and severe instability.
c- Their main value nowadays is to provide a mobile joint following resection of
tumors at the bone ends.
d- The lack of rotation in these implants, made it less liable for aseptic loosening.
e- Moreover, a considerable amount of bone has to be removed, and this makes a
subsequent arthrodesis difficult.
216- Aseptic loosening of total knee replacement
a- Results from faulty prosthetic design.
b- A loose prosthesis cannot be re-cemented
c- Results from inaccurate bone shaping, incorrect placement of the implants or a
combination of these factors.
d- Revision surgery for loose prostheses must deal with the cause.
e- In revision, unless the cause is dealt with, loosening will recur.
217- Patellar problems in total knee replacement
a- Although relatively common, these can be not disabling.
b- They include recurrent patellar subluxation or dislocation, which may need
realignment.
c- Complications associated with patellar resurfacing, such as loosening of the
prosthetic component.
d- Fracture of the remaining bony patella.
e- Catching of soft tissues between the patella and the femur.
218- In idiopathic clubfoot there is
a- Heel in equines.
b- The entire hid foot is varus.
c- Midfoot and forefoot pronated.
d- Midfoot and forefoot supinated.
e- Midfoot and forefoot adducted.
219- Pathological anatomy of idiopathic clubfoot
a- The neck of the talus points downwards and deviates medially.
b- The body of the talus is rotated inwards in relation to both the calcaneum.
c- The posterior part of the calcaneum is held close to the fibula
d- The calcaneum is tilted into equinus and varus; it is also rotated medially
beneath the ankle.
e- The navicular and entire forefoot are shifted medially and rotated into
supination.
220- In severe idiopathic clubfoot
a- The heel is small.
b- The heel is high.
c- There is a deep skin creases appear posteriorly and medially.
d- The foot can be dorsiflexed and everted until the toes touch the front of the
leg.
e- In some cases, the calf is abnormally thin.
221- Congenital talipes equinovarus – treatment
a- First line treatment is non-operative, by manipulation and strapping or serial
casting.
b- If insufficient correction, a formal open release may be needed.
c- Persistent equinovarus in early childhood treated by triple arthrodesis of tarsal
bone.
d- Severe relapses need more radical forms of treatment such as the Ilizarov
fixator.
e- After successful correction of deformity, relapses may be prevented by using
Dennis Browne boots in infants or moulded ankle–foot orthoses in older
children.
222- Gradual correction of club foot by means of a circular external fixator (the
Ilizarov method)
a- Has gained popularity in treating difficult relapsed cases and severe
deformities.
b- The early results are not encouraging.
c- Full corrections can be achieved even in feet severely scarred from previous
surgery.
d- There is often an increase in the size of the foot, which is thought to be due to
an increase in the blood supply during distraction.
e- The procedure can be painful and long and for the time being it is best
reserved for these very difficult cases.
223- Metatarsus adductus
a- Varies from a slightly curved forefoot to something resembling a mild club-
foot.
b- The majority (90 per cent) either improve spontaneously or can be managed
non-operatively using serial corrective casts followed by straight-last shoes.
c- The more severe examples need operation.
d- Extensive capsulectomies of the tarso-metatarsal joints followed by prolonged
splintage is effective treatment.
e- Variations of the Dilwyn Evans procedure, often in combination with basal
metatarsal osteotomies, are suitable for the small percentage of children who
require surgical treatment.
224- Talipes calcaneovalgus
a- Is an uncommon deformity that presents in the newborn as an acutely
dorsiflexed foot.
b- There is a deep crease on the front of the ankle, and the calcaneum juts out
posteriorly.
c- Unlike congenital vertical talus, this deformity is flexible.
d- Calcaneovalgus is usually bilateral.
e- There is an association with hip dysplasia, especially if it presents on one side
only.
225- X-ray features of congenital vertical talus.
a- The calcaneum is in equinus.
b- The talus points into the sole of the foot.
c- The navicular dislocated medially onto the neck of the talus.
d- It is important to repeat the lateral x-ray with the foot maximally
plantarflexed; in congenital vertical talus the appearance will be unchanged.
e- In flexible flatfoot the dorsally subluxated navicular returns to the normal
position in the lateral x-ray with the foot maximally plantarflexed.
226- Flat-foot
a- Is an uncommon complaint among children.
b- Flexible flatfoot appears in toddlers as a normal stage in development, it
usually disappears after a few years.
c- Occasionally, it persists into adult life.
d- Many of these children have ligamentous laxity.
e- There may be a family history of both flat feet and joint hypermobility.
227- Stiff (or ‘rigid’) flat-foot
a- A deformity that cannot corrected passively.
b- Congenital vertical talus is one cause.
c- Tarsal coalition is common cause.
d- Inflammatory joint disorder is not a cause of rigid flatfoot.
e- A neurological disorder is other cause.
228- Conservative treatment of painful flatfoot in adult
a- Rest for some time by modified shoes and by crutches.
b- Temporary insole and splint
c- Temporary by NSAIDs.
d- Local steroid injection of tibialis posterior tendon.
e- Modification of daily activities.
229- The cord lesion causes of pes cavus are
a- Poliomyelitis.
b- Friedreich's ataxia.
c- Syringomylia.
d- Diastomatomyelia.
e- Tethered cord.
230- Calcaneo-cavus deformity treated by
a- Tibialis anterior transfer to the calcaneum.
b- Crescent calcaneal osteotomy.
c- Triple arthrodesis may be used.
d- Tenodesis of the ankle, using tendon Achilles.
e- Excising a wedge of bone from the lateral side of calcaneum.
231- The commonest complain of hallux valgus
a- Pain over the bunion.
b- Awareness about cosmoses.
c- Stiffness of MTP joint of big toe.
d- Difficulty in fitting shoes.
e- Deformities of lessor toes.
232- The stable type of hallux valgus
a- The MTP joint is normally centered.
b- The MTP joint is congruent.
c- The MTP joint are tilted towards valgus
d- The articular surface are subluxated.
e- Deformity is likely to progress very slowly.
233- Hallux rigidus
a- Occur in old age.
b- May follow gout and pseudogout.
c- May follow repeated trauma.
d- May follow osteoarthritis.
e- Is very often bilateral.
234- Claw toe
a- Patient complain of pain and callosity under metatarsal head.
b- Is usually unilateral.
c- Walking may be severely restricted.
d- Painful corns may develop on the dorsum of the toes.
e- In the most severe cases, the skin ulcerates at the pressure sites.
235- Tailor's bunion
a- An irritating or painful bunionette may form over an abnormally prominent
fifth metatarsal head.
b- The MTP joint is dislocated and the little toe sits on the dorsum of the
metatarsal head.
c- If the shoe cannot adjusted to fit the bump, the bony prominence can trimmed.
d- If the metatarsal shaft bowed laterally, it can straightened by performing a
distal osteotomy.
e- It can corrected by a varus correction at the base of the metatarsal.
236- The diabetes mellitus affect the foot by
a- Peripheral vascular disease.
b- Peripheral neuropathy.
c- Reduction of local and systemic resistance.
d- Muscle imbalance.
e- Osteoporosis.
237- Sever's disease
a- Usually occur in girls about 6 years.
b- It is a mild traction injury.
c- Pain and tenderness localized to the tendo Achilles insertion.
d- The x-ray report usually refers to increased density and fragmentation of the
apophysis.
e- The heel of the shoe should raise a little and strenuous activities restricted for
a few weeks.
238- Achilles tendinitis treated by
a- Rest.
b- Local ice application.
c- The use of NSAIDs (local or oral).
d- Corticosteroid injection.
e- Compression and elevation.
239- Achilles tendon rupture
a- May be preceded by local steroid injection.
b- May be preceded by tendinitis.
c- The typical site is at tendon insertion.
d- The typical site is 4 cm above tendon insertion.
e- May be a sport injury.
240- Calcaneal bursitis
a- Older girls and young women often complain of painful bumps on the backs
of their heels.
b- The posteromedial portion of the calcaneum is prominent and shoe friction
causes retrocalcaneal bursitis.
c- Symptoms are worse in cold weather.
d- Symptoms are worse when wearing high-heeled shoes.
e- Treatment should be conservative – attention to footwear and padding of the
heel.
241- Plantar fasciitis
a- There is pain and tenderness in the sole of the foot, mostly under the heel.
b- Sometimes there is a history of sudden increase in sporting activity, or a
change of footwear.
c- The pain is often worse when first getting up in the morning.
d- X-ray often shows what looks like a spur on the undersurface of the
calcaneum.
e- Spur is responsible for the pain and local tenderness.
242- In Morton’s metatarsalgia
a- The pain is typically reproduced by laterally compressing the forefoot
b- Lateral compression of forefoot with compressing the affected interspace –
this produces the pathognomonic Mulder’s click.
c- Neuralgia affecting a single distal metatarsal interspace, usually the fourth.
d- This is essentially an entrapment or compression syndrome affecting one of
the digital nerves.
e- Secondary thickening of the nerve creates the impression of a ‘neuroma’.
243- Surgical treatment of Morton's metatarsalgia
a- Is often successful.
b- The nerve should release by dividing the tight transverse intermetatarsal
ligament.
c- This can be done through either a dorsal longitudinal or a plantar incision.
d- Most surgeons will also excise the heads of the metatarsal.
e- This is successful in about 90 per cent of patients.
244- Tarsal tunnel syndrome
a- Pain and sensory disturbance in the lateral part of the forefoot, unrelated to
weight bearing.
b- May be due to compression of the posterior tibial nerve behind and below the
medial malleolus.
c- Sometimes this is due to a space-occupying lesion, e.g. a ganglion,
hemangioma or varicosity.
d- The pain is often worse at night and the patient may seek relief by walking
around or stamping the foot.
e- The conduction studies may show slowing of motor or sensory conduction.
245- Ingrown toe-nails
a- The nail burrows into the nail groove; this ulcerates and its wall grows over
the nail.
b- The patient taught to cut the nail rounded.
c- The patient taught to insert pledgets of wool under the ingrowing edges and to
keep the feet clean and dry at all times.
d- If these measures fail, the portion of germinal matrix should ablated.
e- Rarely is it necessary to remove the entire nail or completely ablate the nail
bed.
246- Immediately life threatening chest injuries
a- Tension pneumothorax.
b- Open pneumothorax (sucking chest wound).
c- Massive haemothorax
d- Fracture ribs.
e- Flail chest.
247- Hypovolemic shock results from a loss of volume within the circulation; it
may be due to
a- Whole blood loss from hemorrhage.
b- The peripheral vasculature dilatation.
c- Loss of plasma.
d- Fluid loss from burns.
e- Loss of fluid from severe medical conditions.
248- Neurogenic shock is
a- Caused by an isolated head injury.
b- Is different from ‘spinal shock.
c- Temporary flaccidity following spinal damage.
d- Always related to traumatic spinal cord damage.
e- Likely to co-exist with a degree of hypovolemia from associated trauma.
249- Intraosseous cannulation
a- Reserved only for young children up to the age of about 5 years.
b- Used where intravenous cannulation is not possible.
c- The bone cortex is thin and relatively soft in children, and the marrow
plentiful and vascular.
d- Response time to drug administration is close to IV administration.
e- The entire resuscitations performed through intraosseous cannula, including
all anaesthetic drugs and fluids.
250- Pelvic fractures
a- Can result in life threatening hemorrhage.
b- Should be recognized and managed as part of the circulation assessment
during the primary survey.
c- Pelvic compression devices should used to minimize bleeding.
d- A rapid, surgical referral made for definitive management.
e- A CT scan should obtained during the primary survey.
251- Spinal injuries
a- Should identified during the secondary survey.
b- Should managed first.
c- Immobilization is crucial throughout.
d- Ventilatory and circulatory failure must be recognized and managed.
e- Injuries should excluded clinically, or with CT and MRI, as soon as possible.
252- Limb injuries
a- Are immediately life threatening in the absence of catastrophic hemorrhage.
b- They should be recognized and initially managed in the secondary survey.
c- Splinting and immobilization instituted before prompt surgical consultation.
d- Traumatic amputations, de-gloving injuries and blast injuries can initially
managed with specialist blast dressings.
e- May need operative intervention whilst life-saving abdominal or neurological
surgery is taking place.
253- Management of shock
a- Initially attention should focused on rapid assessment with airway, breathing
and circulation (ABC) addressed in the first instance.
b- High flow oxygen should administered via a patent airway, and intravenous
access obtained.
c- Definitive treatment of the underlying cause of shock should commenced
alongside resuscitative measures.
d- This may be rapidly successful.
e- Mortality determined only by etiology of circulatory shock.
254- Tetanus
a- Is caused by Clostridium tetani flourishes only in dead tissue.
b- The endotoxin released passes to the central nervous system via the blood and
the perineural lymphatics from the infected region.
c- The toxin is fixed in the anterior horn cells and therefore cannot be neutralized
by antitoxin.
d- Established tetanus is characterized by tonic, and later clonic contractions.
e- Ultimately, the diaphragm and intercostal muscles may be ‘locked’ by spasm
resulting in asphyxia.
255- Fat embolism
a- Is an uncommon phenomenon following limb fractures.
b- Circulating fat globules larger than 10μm in diameter occur in most adults after
closed fractures of long bones.
c- A small percentage of these patients develop clinical features similar to those
of ARDS.
d- The source of the fat emboli is probably the bone marrow, and the condition is
more common in patients with multiple fractures.
e- Pathognomonic signs are petechiae on the trunk, axillae and in the conjunctival
folds and retinae.
256- Most fractures are splinted to
a- Ensure union.
b- Alleviate pain.
c- Ensure that union takes place in good position.
d- Permit early movement of the limb.
e- Permit early return of function.
257- Remodeling
a- A cuff of solid bone has bridged the fracture.
b- Over a period of months, or even years, a continuous process of alternating
bone resorption and formation reshapes this crude ‘weld’.
c- The thick cellular mass, with its islands of immature bone and cartilage, forms
splint on the periosteal and endosteal surfaces.
d- Thicker lamellae are laid down where the stresses are high, unwanted
buttresses are carved away and the medullary cavity is reformed.
e- Eventually, and especially in children, the bone reassumes something like its
normal shape.
258- Union
a- The ensheathing callus is calcified.
b- Clinically the fracture site is still a little tender and, though the bone moves in
one piece.
c- Attempted angulation is painful.
d- Is complete repair.
e- X-Rays show the fracture line still clearly visible, with fluffy callus around it.
259- Closed tibial fracture grade III Tscherne classification:
a- A fracture with superficial abrasion or bruising of the skin and subcutaneous
tissue.
b- A severe injury with marked soft-tissue damage.
c- There is a risk of threatened compartment syndrome.
d- They are more likely to require some form of mechanical fixation.
e- Good skeletal stability aids soft-tissue recovery.
260- Reduction of fractures is unnecessary.
a- When there is no displacement.
b- When there is little displacement.
c- When displacement does not matter initially.
d- When reduction is unlikely to succeed.
e- When the fracture involving an articular surface.
261- The objectives of exercise are
a- To restore function of the injured parts only.
b- To reduce edema.
c- To preserve joint movement,
d- To restore muscle power.
e- To guide the patient back to normal activity.
262- Stabilizing the open fracture
a- Is important in reducing the likelihood of infection.
b- Assisting recovery of the soft tissues.
c- Immediate stabilization and wound cover is ideal at the time of initial surgery.
d- The method of fixation depends on the degree of contamination, length of time
from injury to operation and amount of soft-tissue damage.
e- Internal or external fixation may be appropriate depending on the individual
characteristics of the fracture and wound.
263- When infected open fracture communicated with joint, the principle of
treatment is
a- Debridement.
b- Drainage.
c- Antibiotic and splintage.
d- Early joint mobilization.
e- Bone stabilization.
264- Emergency treatment of gunshot injuries is
a- Arrest external bleeding by suturing.
b- General resuscitation.
c- Wound covered with sterile dressing.
d- Antibiotics.
e- Examine for arteries and nerve injuries.
265- Closed-range shotgun injury
a- Is low velocity technically.
b- The mass of shot transfer large quantity of energy to the tissues.
c- Treated as low velocity injury.
d- Is heavily contaminated.
e- There is marked tissue destruction over a wide area.
266- The diagnosis of compartmental syndrome.
a- The early diagnosis is essential.
b- The difference between systolic pressure and compartment pressure more than
30 mmHg is diagnostic.
c- The difference between diastolic pressure and compartment pressure less than
30 mmHg is diagnostic.
d- Ischemic muscles is highly sensitive to stretch.
e- Presence of pulse did not exclude compartment syndrome
267- If facilities for measuring compartmental pressures are not available,
a- The decision to operate will have to be made on clinical grounds.
b- If three or more signs are present, the diagnosis is almost certain.
c- If the clinical signs are ‘soft’, the limb should be examined at 30-minute
intervals and if there is no improvement within 2 hours of splitting the
dressings, fasciotomy should be performed.
d- In the case of the leg, ‘fasciotomy’ means opening all four compartments
through anterior incision.
e- Muscle will be dead after 6 hours of total ischaemia.
268- To prevent gas gangrene in lacerated open femoral fracture
a- The wound should be explored.
b- Antitoxin for gas gangrene is essential.
c- All dead and doubtful tissue should be excised.
d- The wound should be left open.
e- Prophylactic penicillin and Flagel should be given.
269- The early lifesaving treatment of gas gangrene is
a- General management of patient.
b- Hyperbaric oxygen.
c- Intravenous antibiotics.
d- Prompt decompression of wound and removal of all dead tissues.
e- Early amputation.
270- Biological causes of delayed union is
a- Inadequate blood supply.
b- Infection.
c- Imperfect splintage.
d- Sever soft tissue damage.
e- Periosteal stripping.
271- A stress fracture
a- Is one occurring in the normal bone of a healthy patient.
b- Due to small repetitive stresses
c- Of two main types: bending and compression.
d- This process affects old adults undertaking strenuous physical routines.
e- A combination of compression and shearing stresses may account for the
osteochondral fractures.
272- The clinical features of stress fracture
a- There may be a history of unaccustomed and repetitive activity.
b- There may be a history of a strenuous physical exercise programme.
c- A common sequence of events is pain after exercise – pain during exercise –
pain without exercise.
d- Occasionally the patient presents only after the fracture has healed and may
then complain of a lump (the callus).
e- The patient is usually ill and in pain.
273- Physeal injuries type V and VI
a- A longitudinal compression injury of the physis.
b- There is no visible fracture but the growth plate is crushed and this may result
in growth arrest.
c- There is perichondrial ring injury, which carries a significant risk of growth
disturbance.
d- Diagnosis is made usually in retrospect after development of deformity.
e- Treated by splinting the part in a cast or a close-fitting plaster slab for 2–4
weeks
274- Type 1 physeal injuries
a- A transverse fracture through the hypertrophic or calcified zone of the plate.
b- Even if the fracture displaced, the growing zone of the physis is usually not
injured.
c- Growth disturbance is common.
d- Usually occurs in infants but is also seen at puberty as a slipped femoral
epiphysis.
e- Displaced fractures should be reduced as soon as possible.
275- Type 4 physeal injuries
a- The fracture splits the epiphysis, but it extends into the metaphysis.
b- Demand perfect anatomical reduction by gentle manipulation under general
anesthesia; if this is successful, the limb is held in a cast for 2–3 weeks.
c- These fractures are liable to displacement.
d- May resulting in asymmetrical growth.
e- If cannot be reduced accurately by closed manipulation, immediate open
reduction and internal fixation with smooth K-wires is essential. The limb is
then splinted for 4–6 weeks.
276- Pathological compression fractures of the spine
a- Cause mild pain and no spinal instability.
b- Treatment should include operative stabilization.
c- If there are actual or threatened spinal cord or cauda equina compression, the
segment should also be decompressed.
d- Postoperative irradiation is given as usual.
e- With all types of metastatic lesion, the primary tumor should be investigated
and treated as well.
277- Ligament strain
a- Is only some of the fibers in the ligament are torn and the joint remains stable.
b- Is one in which the joint is momentarily twisted or bent into an abnormal
position.
c- Is complete tear of the ligament and the joint is unstable.
d- The joint is painful and swollen and the tissues may be bruised.
e- Tenderness is localized to the injured ligament and tensing the tissues on that
side causes a sharp increase in pain.
278- Complete ligament rupture
a- Pain is severe.
b- There may be considerable bleeding under the skin.
c- The joint is not swollen; there is no haemarthrosis.
d- Under general anesthesia, the instability can be demonstrated.
e- X-ray may show a detached flake of bone where the ligament is inserted.
279- Solitary medial collateral ligament ruptures of the knee
a- Early operative treatment is the best method.
b- Usually treated non-operatively.
c- The joint splinted, after 1–2 weeks; the splint exchanged for a functional
brace.
d- Physiotherapy applied to maintain muscle strength and later proprioceptive
exercises added.
e- This nonoperative approach has shown better results not only in the strength
of the healed ligament but also in the nature of healing – there is less fibrosis.
280- The dislocation
a- The joint surfaces are completely displaced and are no longer in contact.
b- Reduced as soon as possible; usually a general anaesthetic is required.
c- The joint then mobilized until soft-tissue swelling reduces – usually after 2
weeks.
d- Controlled movements then begin in a functional brace; progress with
physiotherapy monitored.
e- Occasionally surgical reconstruction for residual instability is called for.
281- Late complications of dislocation
a- nerve injury
b- Avascular necrosis of bone.
c- Heterotopic ossification.
d- joint stiffness
e- Secondary osteoarthritis.
282- Displaced fracture of the middle third of the clavicle
a- Is the most common injury.
b- Non-operative management consists of applying a simple sling for comfort. It
is discarded once the pain subsides.
c- Shortening of more than 2 cm cause no symptoms.
d- The fracture usually unites in this position, leaving a barely noticable ‘bump’.
e- The severe displaced fracture treated by internal fixation.
283- Risk factors for nonunion of clavicle is
a- Increasing age.
b- Increasing displacement.
c- Increasing comminution.
d- Female sex.
e- Type of fixation.
284- The body of the scapula fracture
a- Caused by a crushing force.
b- There may be severe bruising over the scapula or the chest wall.
c- Usually associated with fractures ribs.
d- Usually associated with humeral shaft.
e- Careful neurological and vascular examinations are essential.
285- Fracture of the glenoid fossa
a- Usually suggests a medially directed force (impaction of the joint).
b- Often associated with severe injuries to the chest, and brachial plexus
c- May occur with dislocation of the shoulder.
d- The inner fragment pulled downwards by the weight of the arm.
e- May result in instability of the shoulder.
286- Fractures of the acromion
a- Is due to direct force.
b- Undisplaced fractures treated non-operatively.
c- Inferior displacement in type III reduced subacromial space
d- Only Type III acromial fractures require operative intervention.
e- Type I is unstable.
287- Floating shoulder
a- Glenoid fracture is associated with fracture of the clavicle or disruption of the
acromioclavicular ligament.
b- Is unstable injury.
c- Diagnosis is easy and there is no need for advanced imaging.
d- At least one of the injuries (and sometimes both) will need operative fixation
before the fragments are stabilized.
e- Associated injuries is common.
288- Scapulothoracic dissociation
a- Is a high-energy injury.
b- The scapula and arm are wrenched away from the chest.
c- Rupturing the subclavian vessels and brachial plexus.
d- The limb is flail and ischemic.
e- The diagnosis is usually made on the finding of CT scan.
289- Acute injury of the acromioclavicular joint is
a- Common and usually follows indirect trauma.
b- Chronic sprains often associated with degenerative changes.
c- If the force is severe enough, the coracoclavicular ligaments will also be torn.
d- (Type V) if there is very markedly upwards displacement of clavicle.
e- (Type VI) if the clavicle is inferiorly beneath the coracoid process.
290- Operative repair of the acromioclavicular joint should be considered only
a- For patients with extreme prominence of the clavicle.
b- For those with anterior dislocation.
c- For those with posterior dislocation.
d- For those with inferior dislocation of the clavicle.
e- Those who aim to resume strenuous overarm or overhead activities.
291- Unreduced dislocation acromioclavicular joint
a- Followed by supraspinatus tendinitis.
b- Is ugly and sometimes affects function.
c- Simple excision of the distal clavicle will only make matters worse.
d- An attempt should be made to reconstruct the coracoclavicular ligament.
e- The Weaver–Dunn procedure may be suitable.
292- Anterior sternoclavicular dislocation
a- Is usually caused by lateral compression of the shoulders.
b- Easily diagnosed; the dislocated medial end of the clavicle forms a prominent
bump over the sternoclavicular joint.
c- The condition is painful and there may cardiothoracic complications.
d- Plain x-rays are difficult to interpret.
e- Internal fixation is unnecessary and very dangerous.
293- Fracture-dislocation of the shoulder
a- Open reduction and internal fixation may be necessary.
b- The greater tuberosity may be sheared off during dislocation.
c- The greater tuberosity usually falls into place during reduction.
d- If greater tuberosity remains displaced, surgical reattachment is recommended.
e- Prolonged immobilization is indicated for treatment.
294- Unreduced dislocation of anterior dislocation of the shoulder
a- Surprisingly, a dislocation of the shoulder sometimes remains undiagnosed.
b- Is more likely if the patient is either unconscious or very old.
c- Closed reduction is worth attempting up to 6 weeks after injury.
d- Operative reduction is indicated after 6 weeks in all age.
e- In the elderly, the dislocation is disregarded and gentle active movements are
encouraged. Moderately good function is often regained.
295- Recurrent dislocation of shoulder
a- Detachment of the labrum occurs particularly in young patients and predispose
for recurrence.
b- If at injury a bony defect has been gouged out of the posterolateral aspect of
the humeral head, recurrence is even more likely.
c- In older patients, especially if there is a rotator cuff tear or greater tuberosity
fracture, recurrent dislocation is more common.
d- The period of post-operative immobilization makes no difference.
e- The history is diagnostic and apprehension test is pathognomonic.
296- Posterior dislocation of the shoulder
a- Is uncommon, accounting for less than 8 per cent of all dislocations around the
shoulder.
b- Indirect force producing marked internal rotation and adduction needs be very
severe to cause a dislocation.
c- This happens most commonly during a fit or convulsion, or with an electric
shock .
d- The diagnosis is frequently missed.
e- The arm is held in internal rotation and is locked in that position.
297- Inferior dislocation of the shoulder
a- Is rare but it demands early recognition .
b- A severe hyper-abduction force causes the injury
c- The arm remains fixed in abduction.
d- Soft-tissue injury may be severe and includes avulsion of the capsule and
damage to the brachial plexus and axillary artery .
e- If the humeral head is stuck in the soft tissues arthroscopic examination is
needed.
298- The Neer's classification of proximal neck fracture
a- Neer drew attention to the four major segments involved in these injuries: the
head of the humerus, the lesser tuberosity, the greater tuberosity and the shaft.
b- Neer’s classification distinguishes between the number of displaced
fragments,
c- Fragment displacement defined as greater than 45 degrees of angulation or 1
cm of separation.
d- Thus, however many fracture lines there are, if the fragments are undisplaced
it is regarded as a two-part fracture;
e- If one segment is separated from the others, it is a two-part fracture;
299- Three-part fracture-dislocations of proximal humerus
a- The surgical neck is also broken.
b- Usually causes little disability.
c- Usually require open reduction and fixation.
d- Prosthetic replacement not recommended in very active patients.
e- The brachial plexus is at particular risk during this operation.
300- Fractures of proximal humerus in children
a- Considerable displacement and angulation can be accepted.
b- Malunion readily compensated for during the remaining growth period.
c- Pathological fractures are common in children.
d- Fracture through a simple cyst usually unites and the cyst often heals
spontaneously.
e- All that needed is to rest the arm in a sling for 4–6 weeks.
301- Fracture shaft of humerus
a- Fracture of the shaft in an elderly patient may be due to a metastasis.
b- With fractures above the deltoid insertion, the proximal fragment adducted by
pectoralis major.
c- With fractures lower down the deltoid insertion, the proximal fragment
abducted by the deltoid.
d- They heal readily, they require neither perfect reduction nor immobilization
e- Injury to the radial nerve is uncommon, fortunately, recovery is usual.
302- Supracondylar fracture of humerus in adult
a- These extra-articular fractures are not rare in adults.
b- They are usually displaced and unstable.
c- There may be comminution of the distal humerus.
d- Closed reduction is unlikely to be stable and K-wire fixation is not strong
enough to permit early mobilization.
e- Open reduction and internal fixation is the treatment of choice.
303- Displace intercondylar fracture of lower humerus
a- Open reduction and internal fixation is the treatment of choice.
b- The conservative treatment had strong tendency to stiffness.
c- Good exposure of the joint needed, if necessary by performing an intra-
articular olecranon osteotomy.
d- First, the articular block reconstructed with a transverse screw; bone graft is
sometimes needed.
e- The distal block then fixed to the humeral shaft with K- wire.
304- Displaced fractures of capitulum
a- Closed reduction is feasible, but prolonged immobilization may result in a stiff
elbow.
b- Operative treatment is therefore preferred.
c- The fragment is always larger than expected.
d- Headless bone screws ideally passed from back to front.
e- If internal fixation difficult, the fragment is best excised.
305- Treatment of comminuted fracture of radial head
a- Always assess for an associated soft tissue injury
b- If there is interosseous ligament injury, excision of the radial head is indicated.
c- The head must be meticulously reconstructed or replaced with a metal spacer.
d- Radial head excision usually gives a good long-term result.
e- Joint stiffness is common and may involve both the elbow and the radioulnar
joints.
306- Treatment of olecranon fracture
a- A comminuted fracture with the triceps intact; should be treated by cast for 6
weeks.
b- Many of these patients are old and osteoporotic, and immobilizing the elbow
will lead to stiffness.
c- An undisplaced transverse fracture that does not separate when the elbow is x-
rayed in flexion can be treated by splintage.
d- Displaced transverse fractures can be held by tension band wiring.
e- Oblique fractures may need a lag screw, neutralized by a tension band system
or plate.
307- Dislocation of the elbow
a- Is fairly common- more so in adults than in children.
b- In 40% of cases is displaced posteriorly or posterolaterally.
c- Provided there is no associated fracture, reduction will usually be stable.
d- The combination of ligamentous disruption and fracture will render the joint
more unstable.
e- Soft tissue disruption is often considerable and surrounding nerves and vessels
may be damaged.
308- Heterotopic ossification (myositis ossificans) of elbow
a- May occur in the damaged soft tissues in front of the joint.
b- It is due to muscle bruising or hematoma formation.
c- Usually associated with forceful reduction and passive movement of the
elbow.
d- If the condition is suspected, exercises are stopped and the elbow is splinted in
comfortable flexion until pain subsides
e- Anti-inflammatory drugs of no value to reduce the risk of heterotopic bone
formation.
309- The elbow
a- Is a complex hinge.
b- Had little stability to support the necessary gripping, pushing, pulling and
carrying activities of daily life.
c- Stability is due largely to the shape and fit of the bones.
d- Is liable to be compromised by any break in the articulating structures.
e- Surrounding soft-tissue structures also are important, especially the capsular
and collateral ligaments.
310- Baumann’s angle
a- Measurement of Baumann’s angle is helpful.
b- An anteroposterior view in flexed elbow after reduction of the supracondylar
fracture.
c- This is the angle subtended by the longitudinal axis of the humeral shaft and a
line through the coronal axis of the capitellar physis.
d- Normally this angle is more than 80 degrees.
e- If the distal fragment is tilted in varus or shifted, the increased angle is readily
detected.
311- Open reduction of supracondylar fracture indicated for
a- Severely displaced fracture.
b- Unreducible fractures.
c- Unstable fracture after reduction.
d- An open fracture;
e- A fracture associated with vascular damage.
312- The nerve injuries in supracondylar fractures is
a- The radial nerve
b- The posterior interosseous.
c- The median nerve.
d- The anterior interosseous.
e- The ulnar nerve may be injured.
313- Malunion following supracondylar fracture
a- Is common.
b- Backward or sideways shifts are gradually smoothed out by modelling.
c- Forward or backward tilt may limit flexion or extension caused marked
disability.
d- Uncorrected sideways tilt (angulation) and rotation lead to varus (or rarely
valgus) deformity of the elbow; this is permanent and will not improve with
growth.
e- If deformity is marked, it will need correction by supracondylar osteotomy
usually once the child approaches skeletal maturity.
314- The lateral condylar fracture
a- The child falls on the hand with the elbow extended and forced into valgus.
b- A large fragment, which includes the lateral condyle, breaks off and pulled
upon by the attached wrist extensors.
c- The fracture line usually runs along the physis and into the trochlea.
d- It crosses the growth plate and so is a Salter Harris Type IV injury.
e- The lateral condyle is ‘capsized’ by muscle pull.
315- The lateral condyle fracture
a- If the condyle is left capsized, non-union is inevitable.
b- With growth, the elbow becomes increasingly valgus and ulnar nerve palsy is
then likely to develop.
c- Even minor displacements sometimes lead to nonunion,
d- Even slight malunion may lead to ulnar palsy in later life.
e- The fracture is a Salter Harris Type V injury and so imperfect reduction can
result in growth arrest.
316- The medial epicondyle fractures
a- May be avulsed by a severe muscle or ligament strain.
b- The child falls on the outstretched hand with the wrist and elbow extended; the
elbow is wrenched into valgus.
c- The elbow may subluxates (even momentarily), the epicondyle may be
dragged into the joint.
d- Ulnar nerve damage is very common.
e- If the epicondyle is trapped in the joint, it must be freed.
317- The medial condyle fracture
a- Is an intra-articular fracture, resulting in considerable pain and swelling.
b- In older children, the metaphyseal component usually easily visualized on x-
ray.
c- In doubtful cases, an arthrogram may be helpful.
d- Displaced fractures treated by closed reduction and splintage for 6 weeks.
e- Stiffness of the elbow is common and extension often limited for months.
318- Fractures the neck of the radius
a- A fall on the outstretched hand forces the elbow into varus and pushes the
radial head against the capitulum.
b- There may be localized tenderness over the radial head and pain on rotating
the forearm.
c- The proximal fragment tilted distally, forwards and outwards.
d- Sometimes the upper end of the ulna also fractured or there may be a posterior
dislocation of the elbow.
e- Displacement of more than 30 degrees requires reduction.
319- Monteggia fracture dislocation of the ulna
a- Any fracture of the ulna associated with dislocation of the radio-capitellar
joint.
b- Not including trans-olecranon fractures in wither associated with radial head
dislocation or not.
c- If the ulnar shaft fracture angulated with the apex anterior then the radial head
displaced anteriorly.
d- In children, the ulnar injury may be an incomplete fracture (greenstick or
plastic deformation of the shaft).
e- With isolated fractures of the ulna, it is essential to obtain a true
anteroposterior and true lateral view of the elbow.
320- Treatment of Galeazzi fracture in adult with irreducible distal radio-ulnar joint
a- Is open operation and compression plating of the radius.
b- An x-ray taken to ensure that the distal radio-ulnar joint reduced.
c- If distal radioulnar joint irreducible, open reduction needed to remove the
interposed soft tissues.
d- The triangular fibrocartilage complex and dorsal capsule then carefully
repaired.
e- The forearm immobilized in the position of stability (usually pronation,
supported by a wire) for 6 weeks.
321- The typical Colles‘ fracture , the distal fragment is
a- Displaced toward dorsum.
b- Angulated towards the dorsum.
c- Displaced towards the radial side.
d- Angulated towards the ulnar side of the wrist.
e- After successful reduction, the radial articular surface faces correctly both
distally and slightly volarwards.
322- Colles' fracture with sever impaction or comminution in osteoporotic bone
treated by
a- Manipulation and plaster immobilization alone may be sufficient.
b- Reduction and holding with percutaneous wires.
c- An external fixator to neutralize the compressive force of muscles, and bone
graft placed into the gap.
d- Plate fixation increasingly used for some Colles’ fractures.
e- Volar locking plate applied to the front of the radius is new alternative.
323- Smith’s fracture
a- The patient presents with a wrist injury, there is a ‘garden spade’ deformity.
b- Lateral view shows that the distal fragment displaced and tilted anteriorly.
c- Reduced by traction, supination and extension of the wrist, and the forearm
immobilized in a cast for 3 weeks.
d- X-rays taken every 7–10 days to ensure the fracture has not slipped.
e- Unstable fractures fixed with percutaneous wires or a plate.
324- Fracture of radial styloid ‘chauffeur’s fracture‘
a- Caused by forced radial deviation of the wrist and may occur after a fall, or
when a starting handle ‘kicks back’.
b- The fracture line is transverse, extending laterally from the articular surface of
the radius.
c- The radial styloid can also fractured as part of trans-scaphoid perilunate
fracture dislocation.
d- The fracture manipulated in the same way as a Colles’ fracture immobilized in
cast for 3 weeks.
e- Imperfect reduction may lead to osteoarthritis; therefore, if closed reduction is
imperfect, the fragment should screwed back, or held with K-wires.
325- The Barton’s volar fracture subluxation
a- Is a volar fracture of the distal radius associated with volar subluxation of the
carpus.
b- It is sometimes mistaken for a Smith’s fracture,
c- The distal fragment displaced posteriorly, carrying the carpus with it.
d- The fracture is inherently unstable.
e- Internal fixation, using a small anterior buttress plate, recommended.
326- Non-union or avascular necrosis may lead to secondary osteoarthritis of the
wrist
a- If the arthritis localized to the distal pole, excising the radial styloid may help.
b- As the arthritis progresses, changes appear in the scapho-capitate joint then the
capitate-lunate joint.
c- The lunate-radius joint affected later.
d- The proximal row carpectomy can used as salvage procedures.
e- The four-corner fusion is other alternative.
327- Lunate dislocation
a- Most dislocations are perilunate.
b- If the lunate is dislocated, it has a characteristic quadrilateral appearance.
c- One or more of the carpal bones may be fractured (usually the scaphoid) with
the dislocation
d- In the lateral view, it is easy to distinguish a lunate from a perilunate
dislocation.
e- With a perilunate dislocation, the lunate tilted only slightly and not displaced
forwards.
328- Scapho-lunate dissociation
a- A wrist sprain may followed by persistent pain and tenderness over the
dorsum just distal to Lister’s tubercle.
b- X-rays show an excessively large gap between the scaphoid and the lunate.
c- The scaphoid may appear foreshortened, with a typical cortical ring sign.
d- In the lateral view, the lunate is tilted anteriorly and the scaphoid posteriorly
(DISI pattern).
e- Should repaired directly with interosseous sutures, protected by K-wires for 6
weeks and a cast for 8–12 weeks.
329- Fractures of the metacarpal neck
a- A blow may fracture the metacarpal neck, usually of the fifth finger (the
‘boxer’s fracture’).
b- X-rays show an impacted transverse fracture with volar angulation of the distal
fragment.
c- The flexion deformity of up to 40 degrees can accepted.
d- The hand immobilized with the MCP joint flexed and the interphalangeal (IP)
joints straight for 6 weeks.
e- The patient warned that the knuckle profile may permanently lost.
330- Bennett’s fracture-dislocation
a- This fracture occurs at the base of the first metacarpal bone.
b- Is commonly due to punching; however, the fracture is oblique, extends into
the CMC joint and is unstable.
c- The thumb looks short and the carpo-metacarpal region swollen.
d- X-rays show that a small triangular fragment has remained in contact with the
medial edge of the trapezoid.
e- The thumb has subluxated proximally, pulled upon by the abductor pollicis
longus tendon.
331- Rolando's fracture
a- This is an intra-articular comminuted fracture of the base of the first
metacarpal with a T or Y configuration.
b- Closed reduction and K-wiring can used.
c- Angular deformities usually remodelled.
d- Open reduction and plate fixation can be used.
e- With more severe comminution, external fixation needed.
332- Comminuted fracture of the phalanges,
a- Usually due to a crush injury
b- Often associated with significant tendon damage
c- Often associated with skin loss.
d- Stiffness is common complication.
e- Commonly seen in osteopaenic bone.
333- Displaced phalangeal fractures
a- Treated, depending on the ‘personality’ of the fracture.
b- Treated, depending on the experience of the surgeon and equipment available.
c- Can treated with neighbour strapping.
d- Treated by prolonged cast splintage.
e- Can treated with plate fixation, percutaneous screw fixation or percutaneous
wires.
334- Fracture of the tuft
a- Caused by a hammer blow.
b- Treated by a protective dressing.
c- The subungual hematoma should evacuated.
d- Best treated with a splint for 6 weeks.
e- If the nail bed is shattered and cosmesis is important, it should meticulously
repaired under magnification.
335- Complications of mallet finer is
a- Nonunion.
b- Stiffness
c- Boutonniere deformity.
d- Swan neck deformity.
e- Persistent droop.
336- The partial rupture of ulnar collateral ligament of thumb
a- Is very common.
b- Ulnar collateral inserts partly into the palmar plate.
c- Only the ligament proper is torn.
d- The thumb is unstable in flexion.
e- Thumb still more or less stable in full extension because the palmar plate is
intact.
337- Preoperative care for hand injury
a- The patient may need treatment for pain and shock.
b- If the wound is contaminated, it should rinsed with sterile crystalloid.
c- Antibiotics should be given.
d- Prophylaxis against tetanus may also needed.
e- The hand wound covered with an alcohol-soaked dressing.
338- Relative contraindication to finger replantation is
a- Single digits.
b- The thumb amputation.
c- Severely crushed digits.
d- Severely avulsed parts.
e- A long ischemic time.
339- Full thickness burns of hand
a- Devitalized tissue should be excised; the wound is cleaned and dressed and 2–
5 days later skin-grafted.
b- Full thickness circumferential burns may need early escharotomy to preserve
the distal circulation.
c- Can usually allowed to heal spontaneously.
d- Skin flaps sometimes needed.
e- The hand should splinted in the position of safety.
340- Electric burns of hand
a- May cause extensive damage.
b- May cause thrombosis, which become apparent only after several days.
c- The patient may of course need resuscitation (treating cardiac anomalies and
myoglobinuria).
d- Should irrigated copiously for 20 or 30 minutes, usually with water or saline.
e- The arm needs to monitor and fasciotomy with debridement of dead tissue
often needed.
341- Injection injuries of hand
a- Many substances can be injected under pressure are damaging because of
tension, toxicity or both.
b- The thumb or index finger is usually involved.
c- Substances can gain entry even through intact skin, air or lead paint may show
on x-ray.
d- Immediate wide decompression and removal of the foreign substance offers
the best hope.
e- The outcome is often good, rarely amputation being necessary.
342- The posterior Osseo-ligamentous complex (or posterior column) of spine
consisting of
a- The pedicles.
b- Facet joints.
c- The posterior half of the vertebral body.
d- Posterior bony arch.
e- Interspinous and supraspinatus ligaments.
343- The anterior column composed of
a- The anterior half of the vertebral body.
b- The anterior part of the annulus fibrosis.
c- The anterior part of the intervertebral disc.
d- The anterior longitudinal ligament.
e- The posterior longitudinal ligament.
344- An unstable injury of spine
a- Is one in which there is a significant risk of displacement and consequent
damage to the neural tissues.
b- All fractures involving the middle column and at least one other column
regarded as unstable.
c- If the neural elements are undamaged, there is little risk of them becoming
damaged.
d- Only 10 per cent of spinal fractures are unstable.
e- Less than 5 per cent are associated with cord damage.
345- The objectives of treatment of spine injuries are:
a- To preserve neurological function.
b- To minimize a perceived threat of neurological compression.
c- To prevent the deformity.
d- To stabilize the spine.
e- To rehabilitate the patient.
346- The indications for urgent surgical stabilization of spine are
a- To preserve the neurological function.
b- MRI signs of likely farther neurological deterioration.
c- An unstable fracture with progressive neurological.
d- Controversially an unstable fracture in a patient with multiple injuries.
e- An unstable fracture with progressive urinary sphincteric dysfunction.
347- Unstable injuries of spine treated
a- Spine held secure until the tissues heal and the spine becomes stable.
b- By the halo attached to a body cast.
c- By internal fixation.
d- Rest in bed until pain and muscle spasm subside.
e- Attempts to reduce dislocations and subluxations should made whether by
adjusting the posture, by traction or by open operation.
348- Four-poster braces are
a- More stable.
b- Applying pressure to the mandible and occiput.
c- Applying pressure to sternum and upper thoracic spine.
d- Adequate for very unstable injury patterns.
e- They can be uncomfortable.
349- Tongs are
a- A pin inserted into the outer table on each side of the skull.
b- These are mounted on a pair of tongs and traction.
c- Restricted to minor sprains for the first few days after injury.
d- Applied to reduce the fracture or dislocation.
e- Used to maintain the reduced position.
350- Plain x-rays of cervical spine
a- Must be of high quality.
b- An open-mouth view is necessary to show C1 and C2.
c- Should be inspected methodically.
d- In the lateral view, the lateral outlines should be intact.
e- In anteroposterior view, the spinous processes and tracheal shadow in the
midline.
351- Occipital condyle fracture
a- Is usually a high-energy fracture.
b- Associated skull or cervical spine injuries must be sought.
c- The diagnosis depends on plain x-ray examination.
d- Impacted and undisplaced fractures can be treated by brace immobilization for
8–12 weeks.
e- Displaced fractures are best managed by using a halo-vest or by operative
fixation.
352- C1 ring fracture (Jefferson’s fracture)
a- Is fracture of the ring of the atlas.
b- Result from severe load on the top of the head.
c- The fracture seen on the open-mouth view and the lateral view.
d- A CT scan is particularly helpful in defining the fracture.
e- Usually, there is neurological damage.
353- Hangman’s fracture
a- Is bilateral fractures of the pars interarticularis of C2 and the C2/3 disc is torn.
b- The mechanism is flexion with distraction.
c- This is one cause of death in motor vehicle accidents when the forehead strikes
the dashboard.
d- Neurological damage is unusual.
e- The fracture is potentially unstable.
354- Hyperextension strains of soft-tissue structures of cervical spine
a- Are common.
b- May caused by comparatively mild de-acceleration forces.
c- Bone and joint disruptions, however are rare.
d- The posterior bone elements are compressed.
e- The anterior structures fail in tension, with tearing of the anterior longitudinal
ligament or an avulsion fracture.
355- Avulsion injury of the spinous process of cervical spine
a- Fracture of the C7 spinous process may occur with severe voluntary
contraction of the muscles at the back of the neck.
b- Known as the clay-shoveller’s fracture.
c- Is a high-energy trauma, the injury may affect more than one level.
d- The injury is painful but harmless.
e- No treatment is required; as soon as symptoms permit, neck exercises are
encouraged.
356- Sprained neck (whiplash injury) differential diagnosis
a- Largely a process of exclusion reaches the diagnosis of sprained neck.
b- X-rays carefully examined to avoid missing a vertebral fracture or a mid-
cervical subluxation.
c- An acute disc lesion should be excluded.
d- Traction injuries of the suprascapular nerve and the brachial plexus may give
similar picture.
e- Pain and stiffness of the neck usually appear immediately after injury.
357- A stress fracture of the pars interarticularis
a- Should be suspected, if an athlete complains the sudden onset of back pain
during their course of strenuous activity.
b- The injury is often ascribed to a disc prolapse.
c- Is best seen in the oblique x-rays.
d- An isotope bone scan may show a ‘cold’ spot.
e- Bilateral fractures occasionally lead to spondylolisthesis.
358- Fractures of the transverse processes
a- Can be avulsed with sudden muscular activity.
b- Is by far the most common vertebral fracture
c- Isolated injuries need no more than symptomatic treatment.
d- It is stable injury.
e- A fracture of the transverse process of L5; this should alert one to the
possibility of a vertical shear injury of the pelvis.
359- Flexion–compression injury of thoracolumbar spine
a- Is the most common vertebral fracture.
b- Is due to severe spinal flexion, though in osteoporotic individuals fracture may
occur with minimal trauma.
c- The posterior ligaments usually remain intact.
d- Pain may be quite severe but the fracture is usually stable.
e- Neurological injury is not rare.
360- Patients with a wedge compression fracture and neurological impairment
a- If nerve loss is incomplete treated by thoracolumbar brace.
b- There is no potential for further recovery.
c- Any increase in kyphotic deformity is an indication for operative
decompression and stabilization.
d- MRI signs of impending cord neurological compression would be an
indication for conservative treatment.
e- If there is complete paraplegia with no improvement after 48 hours, operative
management is indicated
361- Axial compression ( burst injury) of lumbar spine
a- Causing failure of both the anterior and the middle columns.
b- The posterior column is usually, but not always, undamaged.
c- The posterior part of the vertebral body is shattered and fragments of bone
may be displaced into the spinal canal.
d- The injury is usually stable.
e- Posterior displacement of bone into the spinal canal is difficult to see on the
plain lateral radiograph; a CT is essential.
362- Jack-knife injury
a- Caused by combined flexion and posterior distraction.
b- This is seen most typically in lap seat-belt injuries.
c- Are unstable in flexion.
d- The tear passes transversely through the bones or the ligament structures, or
both.
e- Neurological damage is common.
363- The first priority in pelvic fracture,
a- Maintain patent airway.
b- Is to assess the patient’s general condition.
c- Look for signs of blood loss.
d- The abdomen should be carefully palpated.
e- It may be necessary to start resuscitation before the examination is completed.
364- Direct fractures of pelvis
a- Result from a direct blow to the pelvis.
b- Result from usually after a fall from a height.
c- The ischium or the iliac blade is the common site.
d- Internal fixation indicated in some cases.
e- Bed rest until pain subsides is usually all that is needed.
365- Stress fractures of the pelvis
a- The pubic rami are uncommon site.
b- More difficult to diagnose around the sacroiliac joints.
c- Often quite painless.
d- Occurs in severely osteomalacic patients.
e- Obscure stress fractures are best demonstrated by radioisotope scans.
f-
366- Anteroposterior compression pelvic fracture
a- Usually caused by a frontal collision.
b- The pubic rami are fractured or the innominate bones are sprung apart and
externally rotated.
c- The posterior sacroiliac ligaments are strained and may be torn.
d- With disruption of the symphysis called ‘open book’ injury.
e- There may be a fracture of the posterior part of the ilium.
367- Lateral compression pelvic fracture
a- The cause is side-to-side compression.
b- This is usually due to a frontal collision in a road accident or a fall from a
height.
c- Anteriorly the pubic rami on one or both sides are fractured.
d- Posteriorly there is a severe sacroiliac strain or a fracture of the sacrum or
ilium.
e- If the sacroiliac injury is much displaced, the pelvis is unstable.
368- Vertical type of fracture of the pelvis treated by
a- By a skeletal traction combined with bed rest for at least 10 weeks.
b- By an external fixator combined with bed rest for at least10 weeks.
c- By Internal fixation with attaching a plate across the symphysis.
d- By an anterior external fixation and posterior stabilization using screws across
the sacroiliac joint.
e- By a plating anteriorly and iliosacral screw fixation posteriorly.
369- Open-book fractures of the pelvis treated
a- By Internal fixation with attaching a plate across the symphysis.
b- By skeletal traction.
c- By a plating anteriorly.
d- By external fixation.
e- By ilio-sacral screw fixation posteriorly.
370- Fractures of the acetabulum
a- Occur when the head of the femur is driven into the pelvis.
b- Caused by a blow in the side in sever trauma.
c- Is stable and safe injury.
d- Associated fractures are not uncommon.
e- With joint disruption, secondary osteoarthritis is common.
371- Complex acetabular fractures patterns share the following features:
a- The injury is severe.
b- The joint surface is disrupted.
c- They usually need operative reduction and internal fixation.
d- It is usually associated with a posterior dislocation of the hip and may injure
the sciatic nerve.
e- The end result is likely to be less than perfect, unless surgical restoration has
been exact.
372- Conservative treatment of acetabular fractures is preferable in
a- Acetabular fractures with minimal displacement.
b- Patients with isolated posterior wall fractures and dislocation.
c- Displaced fractures that do not involve the superomedial weight bearing
segment (roof) of the acetabulum.
d- Fractures in elderly patients, where closed reduction seems feasible.
e- Patients with ‘medical’ contraindications to operative treatment (including
local sepsis).
373- Posterior dislocation of the hip
a- Usually occurring in a road accident.
b- Often a piece of bone at the back of the acetabulum is sheared off, making it a
fracture-dislocation.
c- Seat-belt restraints can increase the number of posterior hip dislocations.
d- The golden rule is to x-ray the pelvis in every case of severe injury and, with
femoral fractures.
e- The leg is short and lies adducted, internally rotated and slightly flexed.
374- Associated fractured femoral shaft with posterior dislocation of the hip
a- When this occurs at the same time as the hip dislocation, the dislocation is
often missed.
b- It should be a rule that with every femoral shaft fracture, the buttock and
trochanter are palpated, and the hip clearly seen on x-ray.
c- A dislocation should be suspected, if the proximal fragment of a transverse
shaft fracture is seen to be abducted.
d- Closed reduction of the dislocation will be much more difficult.
e- A prompt open reduction of the hip followed by internal fixation of the shaft
fracture should be undertaken.
375- Avascular necrosis of the femoral head following posterior dislocation of the
hip
a- Reported in about 10 per cent of traumatic hip dislocations.
b- If reduction delayed by more than 12 hours, the figure rises to over 40 per
cent.
c- Changes seen first on MRI or isotope bone scans.
d- Result from ischaemia due to interruption of femoral head blood supply when
the hip is immobilized.
e- If the femoral head develops signs of fragmentation, an operation may be
needed.
376- The femoral neck fractures
a- Is the commonest fractures in the elderly.
b- Usually results from a car accident or fall from height.
c- The vast majority of patients are women in the seventh and eighth decades.
d- Is much less common among people whose bone mass is above that of the
population average.
e- Used as a measure of age-related osteoporosis in population studies.
377- Risk factors for femoral neck fractures are
a- Osteoporosis.
b- Osteomalacia.
c- Osteoarthritis.
d- Diabetes.
e- Stroke and alcoholism.
378- Intrascapular transcervical fractures have a poor capacity for healing because
of
a- Tearing the retinacular vessels.
b- The bone is intra-articular has only a flimsy periosteum.
c- The bone had no contact with soft tissues, which could promote callus
formation.
d- Synovial fluid prevents clotting of the fracture hematoma.
e- The bone is mainly cancellous.
379- The situations in which a femoral neck fracture may be missed are
a- Stress fractures.
b- Undisplaced impacted fractures.
c- Painless fractures.
d- Fractures in young adults.
e- Multiple fractures.
380- The elderly patients with fracture neck femur are prone
a- Shock.
b- Deep vein thrombosis.
c- Pulmonary embolism.
d- Pneumonia.
e- Bed sores.
381- In younger patients with traumatic osteonecrosis after femoral neck fracture.
a- The choice of treatment is controversial.
b- Core decompression used in the management of traumatic osteonecrosis.
c- Realignment osteotomy is suitable for those with a relatively small necrotic
segment.
d- Rotational osteotomy is suitable for those with a relatively small necrotic
segment.
e- Joint replacement may be justifiable even in this group.
382- Non-union of femoral neck fractures
a- Occur in more than 30 per cent of all femoral neck fractures.
b- The risk is particularly high in those that severely displaced.
c- The poor blood supply and inadequate fixation are important causes.
d- The patient complains of pain, shortening of the limb and difficulty with
walking.
e- The CT scan shows the sorry outcome.
383- The treatment of femoral neck nonunion
a- Depends on the cause of the non-union and the age of the patient.
b- In the relatively young, subtrochanteric osteotomy may be used.
c- If the patient is old and infirm and pain not unbearable total joint replacement
performed.
d- If the reduction or fixation was faulty, redoing and bone graft advised.
e- If the head is avascular, prosthetic replacement may be suitable.
384- Combined fractures of the neck and femoral shaft
a- Both fractures must be fixed.
b- The femoral neck fracture takes priority.
c- The femoral neck fracture is reduced using closed or, if necessary, open
methods.
d- The femoral shaft fracture managed with an ante-grade locked intramedullary
nail.
e- The femoral neck fracture fixed using multiple screws.
385- Intertrochanteric fractures
a- Are extracapsular.
b- Are common in elderly, osteoporotic people.
c- Most of the patients are women in the 6th decade.
d- Intertrochanteric fractures unite quite easily.
e- Seldom cause avascular necrosis.
386- Unstable intertrochanteric fractures are
a- There is poor contact between the fracture fragments as in four-part
intertrochanteric types.
b- Displaced fracture with extension to lesser trochanter fracture.
c- If the posteromedial cortex is comminuted.
d- Those with a reverse oblique pattern or with a subtrochanteric extension.
e- Those with severe osteoporosis leading to poor quality grip by the fixation
implants.
387- Intertrochanteric fractures are
a- Usually treated conservatively.
b- They unite quite readily.
c- Treated surgically to obtain the best possible position.
d- Treated operatively to get the patient up and walking as soon as possible and
thereby reduce the complications.
e- Non-operative treatment may be appropriate for a small group who are too ill
to undergo anesthesia.
388- The reversed oblique intertrochanteric fracture
a- The fracture line runs downwards obliquely from medial to lateral.
b- There is a tendency for the distal fragment to shift laterally.
c- A 95-degree screw-plate device or an intramedullary device with a hip screw
gives more stable fixation.
d- If closed reduction fails to achieve a satisfactory position, open reduction and
manipulation of the fragments will be necessary.
e- The addition of bone grafts may hasten union of the medial cortex.
389- Failed intertrochanteric fixation
a- Screws may cut out of the osteoporotic bone if reduction is good.
b- If the fixation device is incorrectly positioned.
c- If union is delayed.
d- The implant itself may break.
e- Reduction and fixation may have to be re-done.
390- Undisplaced fractures of femoral neck in children
a- May be treated by immobilization in a plaster spica for 6–8 weeks.
b- Fracture position is not always maintained.
c- There is a considerable risk of late displacement and malunion.
d- There is a considerable risk of non-union.
e- Early aspiration of the intracapsular hematoma must be done.
391- Avascular necrosis of the femoral head following femoral neck in children
a- This is the most common complication; it occurs in about 30 per cent of all
cases.
b- Important risk factors are displacement, an age of more than 10 and a high
velocity injury.
c- X-ray changes usually appear within one month of injury.
d- Non-weight bearing, or ‘containment splintage’ in abduction and internal
rotation, is sometimes advocated .
e- The outcome depends largely on the size of the necrotic area.
392- Coxa vara following femoral neck in children
a- May result from malunion.
b- May result from avascular necrosis or premature physeal closure.
c- If the deformity is mild, remodelling may take care of it.
d- Arthrodesis may be advisable, as a late salvage procedure.
e- If the neck-shaft angle is less than 110 degrees, subtrochanteric valgus
osteotomy will needed.
393- Subtrochanteric fracture in the elderly results from
a- Osteoporosis.
b- Osteomalacia.
c- Paget’s disease
d- Sever trauma.
e- Secondary deposit.
394- Subtrochanteric fractures have several features which make them interesting
a- Blood loss is greater than with femoral neck or trochanteric fractures.
b- There may be subtle extensions of the fracture into the intertrochanteric
region.
c- The proximal part is abducted and externally rotated and flexed.
d- The shaft of the femur has to be brought into a position to match the proximal
part or else a malunion is created by internal fixation.
e- The leg lies in internal rotation and looks short; the thigh is markedly swollen.
395- The main indications for traction in femoral shaft fractures are
a- Fractures in children.
b- In the upper third of the femur.
c- Contraindications to anesthesia.
d- Lack of suitable skill for internal fixation.
e- Lack of suitable facilities for internal fixation.
396- Plate and screw fixation in femoral shaft fractures
a- Is the method of choice for most femoral shaft fractures.
b- Is a comparatively easy way of obtaining accurate reduction.
c- Had a high complication rate.
d- Achieved perfect anatomical reduction of all bone pieces.
e- May damage the healing potential and led to delayed union.
397- The main indications for plates in femoral shaft fractures are
a- Fractures with extensions into the supracondylar region.
b- Fractures with extensions into the pertrochanteric areas.
c- A shaft fracture in a growing child.
d- A comminuted shaft fractures.
e- A fracture with a vascular injury, which requires repair.
398- Retrograde insertion of intramedullary nails in femur indicated in
a- Obese patients.
b- Segmental fractures.
c- When there are bilateral femoral shaft fractures.
d- When there is a tibial shaft fracture on the same side.
e- If there is a femoral neck fracture more proximally.
399- The main indications for external fixation in femur are
a- Comminuted large segment of mid-shaft.
b- Severe open injuries.
c- Management of patients with multiple injuries.
d- The need to deal with severe bone loss.
e- Treating femoral fractures in adolescents.
400- Open femoral shaft fractures
a- Should carefully assessed for skin loss and wound contamination.
b- The patient started on intravenous antibiotics.
c- The wound need debridement.
d- Stabilization best achieved with plate and screws.
e- If there is heavy contamination, or bone loss external fixation is preferable.
401- Fat embolism in femoral shaft fracture
a- Occur in those with multiple injuries .
b- Occur in those with severe shock.
c- Occur in patients with associated chest injuries .
d- It results in progressive respiratory distress and multi-organ failure.
e- Treatment is supportive, with the emphasis on preventing hypercapnia.
402- Infection in femoral shaft fractures
a- In open injuries, there is always a risk of infection.
b- Prophylactic antibiotics and careful attention to the principles of fracture
surgery should keep the incidence below 5 per cent.
c- The patient should be treated as for an acute osteomyelitis.
d- Antibiotic treatment may suppress the infection until the fracture unites, at
which time the femoral nail can be removed.
e- If there is pus or a sequestrum, a wound debridement done and the nail
removed and an external fixator applied.
403- Delayed union and non-union after femoral shaft fractures
a- If there is failure to progress by 6 months, as judged by serial x-rays, then
intervention may be needed.
b- A common practice is to remove locking screws from the intramedullary nail
to enable the fracture to ‘collapse’.
c- A better course is to remove the nail, ream the medullary canal and introduce a
larger diameter nail – exchange nailing.
d- Bone graft must applied in all cases.
e- Bone grafts added to the fracture site if there are gaps.
404- Malunion of femoral shaft fracture
a- No more than 15 degrees of angulation should be accepted.
b- Malunion is much less likely in those treated with static interlocked nails.
c- Shift less than third of diameter cannot be accepted.
d- Shortening is seldom a major problem unless there was bone loss.
e- Malrotation need immediate correction.
405- Joint stiffness after femoral shaft fracture
a- The knee is often affected .
b- The joint may be injured at the same time.
c- Stiffens due to soft-tissue adhesions during treatment.
d- Early physiotherapy is important.
e- Usually there is fibrous band and adhesion in side joint.
406- Femoral shaft fractures in teenagers
a- Require (4–6 weeks) in balanced traction.
b- Those aged over 15 may need skeletal traction.
c- Once the fracture feels firm, traction exchanged for either a spica cast or a
cast-brace, which retained for a further 6 weeks.
d- The limit of acceptable angulation is 25 degrees in the anteroposterior x-ray.
e- The limit of acceptable angulation is 15 degrees in the lateral x-ray.
407- Operative treatment for femoral shaft fracture in children
a- This is growing in popularity.
b- A shorter in-patient stay (a quick return home).
c- A lower incidence of malunion.
d- The tendency had low risk of complication.
e- Surgical options include fixation with flexible intramedullary nails and plates
inserted by the MIPO technique.
408- Shortening following treatment for femoral shaft fracture in children
a- Overlapping and comminution of the bone fragments may shorten the femur.
b- Anything up to 5 cm is quite acceptable in young children.
c- Some surgeons regard this as an advantage because there is a tendency for the
fractured bone to grow faster for up to 2 years after the injury.
d- Stimulation of the physes derived from the increased blood flow that
accompanies fracture healing.
e- Unfortunately, the effect on growth is unpredictable.
409- Malunion of femoral shaft fractures in children
a- Angulation can usually be tolerated within 10 degrees.
b- Excellent bone modelling in children is no excuse for casual management.
c- Shortening less than 2 cm can be accepted.
d- Certainly rotational malunion less than 15 degrees corrected by growth or
remodelling.
e- It is probably wise to observe a malunited fracture for 2 years before offering
corrective osteotomy.
410- Supracondylar fractures of the femur
a- Are encountered in young adults, usually as a result of high energy trauma.
b- In elderly, result from osteoporosis.
c- The fracture line is just above the condyles, but may extend between them.
d- In the worst cases, the fracture severely comminuted.
e- Gastrocnemius, arising from the posterior surface of the distal femur, will tend
to pull the distal segment into flexion.
411- The supracondylar fracture of femur x-ray assessment
a- Is a fracture into the joint?
b- Is it comminuted?
c- The size of the distal segment.
d- Is the bone osteoporotic?
e- These factors not influence the type of internal fixation required.
412- Valgus stresses are resisted by
a- The superficial and deep layers of the medial collateral ligament.
b- Semimembranosus tendon and its expansions.
c- Popliteo-fibular ligament.
d- The tough posterior oblique ligament.
e- The cruciate ligaments.
413- Extracapsular restraints on varus stresses
a- Lateral collateral ligament.
b- The cruciate ligaments.
c- Popliteus tendon.
d- Popliteofibular ligament.
e- The capsule.
414- If the fibular styloid is avulsed
a- The injury involves part of the posterolateral capsule and arcuate complex.
b- There is posterolateral instability.
c- Arthroscopy should be performed.
d- These injuries may benefit from repair.
e- A long cast-brace worn for 6 weeks.
415- Isolated tears of the ACL
a- Treated by early knee brace.
b- Operation indicated for non-professionals if the tibial spine avulsed.
c- ACL replaced and fixed under arthroscopic control and the knee braced for 6
weeks.
d- In all other cases movement and muscle-strengthening exercises are
encouraged.
e- About half of these patients regain sufficiently good function.
416- Arthroscopy of knee is indicated in
a- The diagnosis of the doubtful ligament injury.
b- Assessment of extent of the ligament injury.
c- Assessment of meniscal tears or cartilage damage.
d- Assessment of acute collateral injury.
e- Partial meniscectomy and removal of loose cartilage tags.
417- The indications for operation in knee chronic instability are
a- Recurrent locking.
b- Chronic instability in elderly.
c- Intolerable symptoms of giving way.
d- Suboptimal function in a sportsperson.
e- Ligament injuries in adolescents.
418- Partial tears of the anterior cruciate ligament
a- There is still much controversy about the need for surgery in these cases.
b- Arthroscopic assisted reconstruction of the ligaments is advisable in all young
patients.
c- The decision of treatment based on an assessment of the patient’s symptoms
and functional capacity.
d- Young adults show diminished activity and run the risk of developing
secondary problems such as meniscal lesions, cartilage damage, increasing
instability and secondary osteoarthritis.
e- With careful follow-up and reassessment, those most at risk usually advised to
undergo reconstructive surgery.
419- Isolated ACL insufficiency
a- Is uncommon.
b- Can usually managed by physiotherapy.
c- Splints or braces may be used to speed the return to weight bearing
d- All the torn ACL repaired with an autologous graft, usually a strip of patellar
tendon.
e- Patients seeking to resume competitive sport may need reconstructive surgery.
420- Combined anteromedial rotatory instability
a- Are common reasons for reconstructive surgery.
b- Early arthroscopy in acute injury should be done.
c- Reconstruction of the ACL alone often suffices.
d- The torn ACL replaced by an autograft. :[Comment [h5
e- The ideal synthetic graft has yet to be developed.
421- Fractured tibial spine
a- Twisting injuries may fracture the tibial spine.
b- This is the adolescent variant of a cruciate ligament tear.
c- The detached bone fragment may remain almost undisplaced.
d- It may be completely detached and displaced.
e- The image seen on x-ray is larger than the actual fragment.
422- Treatment of the tibial spine fracture
a- If there is a block to full extension, surgical treatment indicated.
b- If the bone fragment remains displaced, operative reduction is essential.
c- The fragment anchored by small screws.
d- A long plaster cylinder applied with the knee almost straight for 3 weeks and
then movements are encouraged.
e- The outcome is usually good.
423- Dislocation of knee
a- The knee dislocated only by considerable violence.
b- The cruciate ligaments and one or both lateral ligaments are torn.
c- Leading to huge haemarthrosis.
d- Repeated examination is necessary.
e- Common peroneal nerve injury occurs in 20 per cent of cases.
424- Rupture of quadriceps tendon
a- The patient is usually elderly.
b- May have a history of diabetes or rheumatoid disease, or may have been
treated with corticosteroids.
c- There is bruising and local tenderness; sometimes a gap can be felt proximal
to the patella.
d- Active knee extension is impossible.
e- CT scan confirm the diagnosis.
425- Rupture of patellar ligament
a- This is an uncommon injury.
b- It is usually seen in young athletes
c- There may be a previous history of ‘tendinitis’ and local injection of
corticosteroid.
d- X-rays may show a low-riding patella.
e- MRI will help to distinguish a partial from a complete tear.
426- Patellar ligament complete tears treated by
a- Need operative repair or reattachment to bone.
b- Tension on the suture line can be lessened by inserting a temporary pull-out
wire
c- Immobilization in full extension for 6 weeks is essential.
d- It may be better to support the knee in a hinged brace with limits to the amount
of flexion permitted.
e- Early repair of acute ruptures gives excellent results.
427- Displaced transverse fracture of patella treated
a- Operation is essential through a longitudinal incision .
b- The patella repaired by the tension-band principle.
c- The tears in the extensor expansions are repaired.
d- A plaster backslab or hinged brace worn until active extension of the knee
regained.
e- Patellectomy is advocated, whatever the degree of displacement.
428- Lateral deviation of the patella during knee extension is prevented
a- By the patella seated in the intercondylar groove.
b- By a high lateral condylar ‘embankment’.
c- By the knee is normally angled in slight valgus.
d- By the force of extensor muscle contraction pulls it firmly into the groove.
e- By the medial patellofemoral ligament.
429- Predisposing factors for dislocation of patella are
a- Genu valgum and tibial torsion.
b- Genu varum and internal tibial torsion.
c- High-riding patella (patella alta).
d- A shallow intercondylar groove.
e- Patellar hypermobility due to generalized ligamentous laxity or localized
muscle weakness.
430- Treatment of patellar dislocation
a- In most cases, the patella can pushed back into place without much difficulty.
b- The cast applied for 2 or 3 weeks.
c- The patient then undergoes a long period (2–3 months) of quadriceps
strengthening exercises.
d- If there is much bruising, swelling and tenderness medially, immediate
operative repair indicated.
e- Patients treated non-operatively for a first-time dislocation have a 30–40 per
cent chance of suffering further dislocations.
431- Osteochondral fractures of the knee
a- The joint is swollen and aspiration yields blood-stained fluid mixed with fat
globules.
b- If the diagnosis suspected, ultrasonography needed.
c- MRI or arthroscopy will be more helpful in diagnosis.
d- Small fragments removed as they may cause symptoms.
e- Larger fragments, reattached with headless screws.
432- Osteochondritis dissecans of the knee
a- Affect teenagers and young adults.
b- Usually occur on the lateral aspect of the medial femoral condyle.
c- Medial patellar surface is the common site.
d- This is probably a traumatic lesion, caused by repetitive stress.
e- In x-ray seen as a small segment of osteochondral necrosis.
433- Fractures of the tibial plateau
a- Are caused by a varus or valgus force combined with axial loading.
b- Sometimes the result of a car striking a pedestrian.
c- Rarely, it is due to a fall from a height in which the knee is forced into valgus
or varus.
d- The tibial condyle crushed or split by the opposing femoral condyle, which
remains intact.
e- The fracture pattern and degree of displacement depend on the type and
direction of force as well as the quality of the bone.
434- Type 4 – fracture of the medial tibial condyle
a- May be a depressed, crush fracture of osteoporotic bone in an elderly person.
b- May be a high-energy fracture resulting in a condylar split that runs obliquely
from the intercondylar eminence to the medial cortex.
c- The momentary varus angulation may cause a rupture of the lateral collateral
ligament and a traction injury of the peroneal nerve.
d- The joint is usually stable and may tolerate early movement.
e- The severity of these injuries should not be underestimated.
435- The clinical features of tibial plateau fracture
a- The knee is swollen and may be deformed.
b- Bruising is usually extensive and the tissues feel ‘doughy’ because of
haemarthrosis.
c- The leg and foot carefully examined for signs of vascular or neurological
injury.
d- Examining the knee may suggest medial or lateral instability
e- Traction injury of the peroneal or tibial nerves is common.
436- Type 2 fractures require
a- A direct reduction
b- Elevation of depressed areas.
c- Circular external fixator.
d- Bone grafting.
e- Buttress plate fixation.
437- Types 5 and 6 tibial plateau fractures
a- Are severe injuries that carry risk of a compartment syndrome.
b- Surgery is without significant risk.
c- The wide exposure necessary to gain access to both condyles
d- New strategies involve spanning the knee joint with an external fixator.
e- Buttress plates placed in a submuscular fashion are used.
438- Ligamentotaxis in complex tibial plateau fracture
a- Traction used to achieve reduction.
b- Many of the fragments that have soft-tissue attachments will reduce by direct
open reduction.
c- Done by applying bone distractors across the knee joint or by traction on a
traction table.
d- High-quality imaging needed to define the fracture pattern accurately.
e- The difficulty of fixing plateau fractures should not be underestimated.
439- Complex tibial plateau fractures
a- Soft tissue trauma usually makes it unsafe to undertake extensive open surgery
early.
b- Provisional stabilization by a spanning external fixator allows the swelling to
reduce and the patient to rest comfortably.
c- When conditions improve, and this may take as long as 12 weeks, open
surgery can be undertaken.
d- Two buttress plates may be used to shore up the lateral and posteromedial
aspects of the tibial plateau.
e- A single large fragment may be re-positioned and held with lag screws and
washers.
440- Fracture-separation of proximal tibial epiphysis
a- This common injury, usually caused by a severe hyperextension and valgus
strain.
b- The epiphysis displaces forwards and laterally, often taking a small fragment
of the metaphysis with it (a Salter–Harris type 2injury).
c- There is a risk of popliteal artery damage.
d- The knee is tensely swollen and extremely tender.
e- The swelling may extend into the calf and a careful watch for compartment
syndrome.
441- Associated lesions with fracture of proximal fibula are
a- The ankle injury.
b- Peroneal nerve injury.
c- Lateral collateral ligament injury of knee.
d- Complete growth arrest at the proximal tibia
e- Peroneal nerve entrapment – an occasional late complication.
442- Fracture of tibia and fibula
a- Because of its subcutaneous position, tibia commonly fractured.
b- More often sustains an open fracture than any other long bone.
c- An angulatory force produces spiral fractures
d- Indirect injury is usually low energy.
e- Direct injury crushes or splits the skin over the fracture.
443- The behavior of tibial fractures depends on the following factors
a- The state of the soft tissues.
b- The incidence of tissue breakdown.
c- The severity of the bone injury.
d- Stability of the fracture.
e- Degree of contamination.
444- High-energy tibial fractures
a- Usually caused by direct trauma.
b- Tend to be open (Gustilo III A–C).
c- Tend to be spiral.
d- Tend to be transverse.
e- Tend to be comminuted.
445- Signs of soft-tissue damage in fracture tibia
a- Bruising and severe swelling.
b- Crushing or tenting of the skin.
c- A spiral fracture of both leg bones.
d- Weak or absent pulses.
e- Diminution or loss of sensation and inability to move the toes.
446- A high-energy injury of tibia and fibula.
a- Transverse displaced fractures.
b- Short oblique displaced fractures.
c- Comminuted displaced fractures
d- Long spiral displaced fractures.
e- Segmental displaced fractures.
447- The main objectives of tibial fracture treatment
a- To limit soft-tissue damage and preserve skin cover.
b- To prevent or at least recognize a compartment syndrome.
c- To obtain and hold fracture alignment.
d- To start early weight-bearing (loading promotes healing).
e- To delay joint movements until bone united. .
448- Displaced fracture of tibia and fibula.
a- Reduced under general anesthesia with x-ray control.
b- Alignment must be near perfect and rotation perfect.
c- A below knee cast is applied as for undisplaced fractures.
d- If there is excessive swelling, the cast split.
e- The position checked repeatedly by x-ray.
449- Functional bracing for fracture tibia and fibula
a- Full-length cast applied for 4–6 days first.
b- A functional below-knee brace that is carefully moulded.
c- Brace bear weight upon the upper tibia and patellar tendon.
d- This liberates the knee and allows full weight bearing.
e- A snug fit is important and the fastening straps will need to be tightened.
450- Plate fixation of tibia
a- Is best for metaphyseal fractures that are unsuitable for nailing.
b- Used for unstable tibial shaft fractures in children.
c- Decrease the risk of introducing infection and delaying union.
d- Submuscular plating preserves the soft tissues around the fracture site better
than conventional open plating.
e- Full weight bearing will need to defer until some callus formation is evident.
451- External fixation of tibia
a- This is an alternative to closed nailing.
b- It avoids exposure of the fracture site and allows further adjustments.
c- Partial weight bearing permitted from the start.
d- Active movements delayed until callus formed.
e- The external fixator can replaced by a functional brace once there are signs of
union.
452- Fracture tibia and fibula require early surgical stabilization
a- Comminuted fractures.
b- Segmental fractures.
c- Long oblique fractures.
d- Those associated with bone loss.
e- Any high-energy fracture that is inherently unstable.
453- Open fractures of tibia, principle of treatment is:
a- Antibiotics.
b- Debridement.
c- External fixation.
d- Prompt soft-tissue cover.
e- Passive exercises.
454- Antibiotics in open tibial fractures
a- Are started immediately.
b- A second-generation cephalosporin is suitable for Gustilo grades I–IIIA
wounds.
c- Gustilo grades III B and C used second-generation cephalosporin and
aminoglycoside.
d- With an adequate debridement, the antibiotics are continued for 24 day in a
grade II fracture.
e- Most infections from delayed closure of open tibial wounds tend to be by
nosocomial hospital- acquired bacteria.
455- The risk factors of compartment syndrome in the leg.
a- The proximal tibial fractures.
b- The distal tibial fractures.
c- Severe crush injury.
d- A long delay to treatment.
e- Hemorrhagic shock.
456- The use of continuous compartment pressure monitoring for
a- Patients who are unconscious.
b- Patients who are uncooperative,
c- Those with multiple injuries.
d- Displaced spiral fracture of tibia in children.
e- All comminuted tibial fractures.
457- The difference between diastolic pressure and compartment pressure regarded
as an indication for compartment decompression
a- Is of less than 10 mmHg.
b- Is of less than 15 mmHg.
c- Is of less than 20 mmHg.
d- Is of less than 30 mmHg.
e- Is of more than 30 mmHg.
458- The leg compartments are the
a- Anterior.
b- Lateral.
c- Tibialis anterior.
d- Superficial posterior.
e- Deep posterior.
459- Fasciotomy and decompression of leg
a- Should carried out with the minimum delay.
b- Of all four compartments at the first operation.
c- Accomplished through anterolateral and posteromedial.
d- The incisions closed by light suture.
e- The incisions left open.
460- Delayed decompression of leg compartment syndrome carries the risk of
permanent dysfunction
a- Which cause venous insufficiency.
b- Which varies from mild sensory to sever nerve damage.
c- Which varies from mild motor loss to severe muscle damage.
d- Which cause joint contractures.
e- Which cause atrophic changes in the foot.
461- Malunion of tibial fractures
a- Angulation should prevented at all stages.
b- Anything more than 15 degrees of rotation is unacceptable.
c- Angulation in the sagittal plane, may result in either refracture or non-union
d- Varus or valgus angulation will cause increased stress in some part of the joint.
e- Rotational alignment should be near-perfect.
462- Prophylactic bone grafting in tibial fractures is recommended in
a- High-energy fractures.
b- If there is insufficient contact at the fracture site.
c- Fractures with bone loss
d- Fractures with marked comminution.
e- Hypertrophic non-union.
463- The talus is prevented from slipping out of the mortise by
a- The medial collateral ligament.
b- The lateral collateral ligaments.
c- The inferior tibiofibular ligament.
d- The ankle joint capsule.
e- The peroneal tendons.
464- The distal tibiofibular joint is held by
a- Anterior ligaments.
b- Posterior ligaments.
c- Inferior transverse ligaments.
d- Superior transverse ligaments.
e- The interosseous ‘ligament’, which is really a thickened part of the
interosseous membrane.
465- X-ray examination of ankle in sprain is indicated if there is
a- Pain around the malleolus.
b- A history of a twisting injury followed by swelling.
c- Inability to take weight on the ankle immediately after the injury.
d- Inability to take four steps in the Emergency Department.
e- Bone tenderness at the posterior edge or tip of the medial or lateral malleolus.
466- Initial treatment of ankle sprain consists of
a- Rest.
b- Ice.
c- Cast.
d- Compression.
e- Elevation.
467- Recurrent ankle sprains are potentially associated with
a- Missed fracture of medial malleolus.
b- Added cartilage damage.
c- Warrant careful investigation by MRI.
d- Warrant careful arthroscopy.
e- Warrant careful examination under anesthesia.
468- Talar tilt test
a- The range of movement estimated clinically and compared with that of the
normal ankle.
b- The exact degree of talar tilt measured by x-rays.
c- In a positive test, the talus can felt sliding forwards and backwards.
d- 15 degrees of talar tilt (or 5 degrees more than in the normal ankle) regarded
as abnormal.
e- Inversion laxity suggests injury to both the calcaneofibular and anterior
talofibular ligaments.
469- Anterior drawer test of ankle
a- In a positive test, the talus sliding forwards and backwards.
b- The position of the talus verified by lateral x-rays.
c- Anterior displacement of 10 mm (or 5 mm more than on the normal side)
indicates abnormal laxity of the anterior talofibular ligament.
d- With an isolated tear of the anterior talofibular ligament, the anterior drawer
test may be positive with abnormal talar tilt.
e- A positive anterior drawer test sometimes obtained in normal.
470- Recurrent ‘giving way’ of ankle can sometimes be prevented
a- By modifying shoe-wear.
b- By below knee cast bracing.
c- By raising the outer side of the heel and extending it laterally.
d- By strengthening the peronei, by specific physiotherapy regimes.
e- By a light brace worn during stressful activities.
471- Most patients with functional instability of ankle can be improved and
returned to sport by
a- Arthroscopic debridement of the impinging tissue within the ankle joint,
followed by physiotherapy.
b- Operations that aim to repair or tighten the ligaments.
c- Operation that are designed to construct a ‘check-rein’ against the unstable
movement.
d- The non-anatomic reconstructions that they sacrifice the secondary stabilizers,
the peroneal tendons.
e- Arthrodesis of the ankle joint.
472- The Rupture of the deltoid ligament
a- Is usually associated with either a fracture of the distal end of the fibula or
tearing of the distal tibiofibular ligaments (or both).
b- The effect is to destabilize the talus and allow it to move into inversion and
internal rotation.
c- Diagnosis is made by x-ray.
d- Without apparent lateral disruption at the ankle, it is important to look for the
highly unstable Maissoneuve injury.
e- The fibular fracture or diastasis must be accurately reduced, if necessary by
open operation and internal fixation.
473- The objective for assessment the accuracy of reduction of ankle fractures
a- The fibula restored to its full length.
b- The talus must sit squarely in the mortise, with the talar and tibial articular
surfaces parallel.
c- The medial joint space restored to its normal width.
d- There is a step in the articular surface
e- Oblique x-rays must show that there is no tibiofibular diastasis.
474- Displaced Weber type B ankle fractures
a- The causal mechanism is external rotation of the ankle when the foot caught
in a supinated position.
b- Accompanied by disruption the tibiofibular syndesmosis and interosseous
membrane.
c- If closed reduction succeeds, a cast is applied, similar to undisplaced
fractures.
d- These injuries are unstable and often poorly controlled in a cast.
e- Failure of closed reduction or late redisplacement calls for operative
treatment.
475- Displaced Weber type C ankle fractures
a- The fibular fracture is well above the syndesmosis.
b- There are associated medial and posterior malleolar fragments.
c- There is major ligament damage to the syndesmosis and medial side of the
joint.
d- Sometime are unstable and will need open reduction and internal fixation.
e- The syndesmosis stabilized by inserting a transverse screw across from the
fibula into the tibia.
476- Fracture subluxations of ankle more than 2 weeks old
a- May prove difficult to reduce.
b- Below knee cast brace used for safety.
c- Granulation tissue should be removed from the syndesmosis.
d- Treated by open reduction and internal fixation.
e- A transverse tibiofibular fixation for syndesmosis secured.
477- Postoperative management of ankle fractures
a- Elevation of foot and observation of skin is important.
b- After open reduction and fixation of ankle fractures, movements should be
regained before applying a below-knee plaster cast.
c- The patient allowed full weight bearing in cast with crutches.
d- Some advocate removal of the diastasis-screw when the syndesmosis has
healed.
e- The diastasis-screw may break after weight bearing (especially if four cortices
are engaged).
478- Open fractures of the ankle
a- Usually result from severe injury.
b- Reduction and stabilization at an early stage is very essential.
c- Direct plaster of Paris applied after wound debridement and reduction.
d- Unstable injuries should be treated by internal fixation even in the presence of
an open wound.
e- An external fixator can be used if there is heavy contamination, often as a
temporary spanning option.
479- Pilon fracture
a- Occur when the twisting injuries forced the talus upwards.
b- Occurs when a large force drives the talus upwards against the tibial plafond.
c- There is considerable damage to the articular cartilage
d- There is considerable damage to the subchondral bone.
e- The comminution extends some way up the shaft of the tibia.
480- Treatment of Pilon fractures
a- Staged treatment has reduced the complication rate.
b- Control of soft tissue swelling by elevation is a priority.
c- Applying a spanning external fixator across the ankle joint.
d- It may take 2–3 days of elevation before the soft tissues improve.
e- Surgery can be planned, based on the CT scan.
481- Severely displaced Pilon fracture recently, have been successfully treated by
a- Using a combination of indirect reduction methods and small screws to hold
the articular fragments.
b- Using axially stable locking plates.
c- Circular frame fixation not successful.
d- After fixation, elevation and early movement help to reduce the edema.
e- Arterio-venous impulse devices applied to the sole of the foot are also helpful.
482- Epiphyseal injuries of ankle
a- Salter–Harris types one and 2 injuries are treated closed.
b- If it is displaced, the fracture is gently reduced; the limb is immobilized for 6
weeks.
c- Displaced fractures can sometimes be reduced closed by reversing the forces
that produced the injury.
d- Unless reduction is near-perfect, the fracture should be reduced open and
fixed.
e- Postoperatively the leg is immobilized in a below-knee cast for 12 weeks.
483- Swelling of the foot
a- Is a common problem.
b- Make clinical examination difficult.
c- Delay the union of the fractures.
d- Delay definitive treatment.
e- Reduction of fractures and dislocations are more difficult.
484- To reduce foot edema
a- Realign and splint the foot.
b- Keep it elevated.
c- Apply Cryo-Cuff.
d- Apply warm packs.
e- Used intermittent pneumatic compression foot pumps.
485- Prolonged immobilization of the foot predisposes to
a- Stiffness.
b- Nonunion.
c- Impaired function.
d- Localized osteoporosis.
e- Complex regional pain syndrome.
486- Blood vessels enter the talus from
a- The anterior tibial artery.
b- The posterior tibial artery.
c- The dorsalis pedis artery.
d- The peroneal arteries.
e- The anastomotic vessels from the surrounding capsule and ligaments.
487- The fractures of the lateral and posterior process of talus
a- May associated with ankle sprain.
b- Fracture of posterior process should be distinguish from os trigonum.
c- Tiny fragment fractures are treated by cast for 4 weeks.
d- Displaced large fragment treated by open reduction and fixation by small
screw.
e- Lateral process fracture called snowboarder fracture.
488- The imaging used for talar fractures
a- Anteroposterior x-ray view.
b- Lateral x-ray view.
c- Oblique x-ray view.
d- CT scanning helps to identify the type.
e- MRI for osteochondral injuries.
489- Type III fracture–dislocations of the neck of the talus
a- Need urgent open reduction and internal fixation.
b- The approach will depend on the fracture pattern and position of displaced
fragments.
c- Osteotomy of the medial malleolus might help.
d- Small exposure is essential to permit removal of small fragments and perfect
reduction.
e- Postoperatively the foot splinted and elevated until the swelling subsides.
490- Open fractures of the talus
a- Are often associated with burst skin wounds.
b- There is a high risk of infection in these wounds and prophylactic antibiotics
are advisable.
c- Treated as an emergency.
d- Under general anesthesia, the wound cleaned and debrided and all necrotic
tissue removed.
e- The wound closed by primary suture.
491- Intra-articular fractures of calcaneum
a- Are much more complex and unpredictable in their outcome.
b- If the heel is in valgus (abducted), the fracture is in the medial part of the
facet.
c- Flattening of the angle subtended by the posterior articular surface and the
upper surface of the body posterior to the joint (Böhler’s angle).
d- The advent of CT, and the operative reduction and fixation of displaced
calcaneal fractures, improve the outcome.
e- Especially with bilateral fractures or in the unconscious patient – it is essential
to assess the knees, spine and pelvis as well.
492- Fractures of the calcaneal tuberosity
a- Usually due to avulsion by the tendo Achilles'.
b- Clinical signs are similar to those of a torn Achilles tendon.
c- If the fragment displaced, it reduced and fixed with cancellous screws.
d- Bone grafts added to fill in defects.
e- Postoperatively the foot immobilized in slight equinus to relieve tension on the
tendo Achillis.
493- The outcome for displaced intra-articular fractures of calcaeum
a- Is much less predictable.
b- The results of operative treatment are heavily dependent on the severity of the
fracture and the experience of the surgeon.
c- Closed treatment, though it may be the only alternative, has a bad reputation.
d- Closed treatment, only five out of 10 patients had a ‘good’ result.
e- Those with comminuted fractures treated by closed method: all of them
assessed as having a poor result.
494- Malunion of calcaneum
a- Result from closed treatment of displaced fractures.
b- Result from injudicious weight bearing after open reduction.
c- The heel is broad and squat, and the patient has a problem fitting shoes.
d- Result from improper reduction of displaced fractures.
e- Usually the foot is in varus and walking may be impaired.
495- Fracture–dislocation of midtarsal bone
a- These are severe injuries.
b- Reduced by closed manipulation with percutaneous K-wires across the joints
to fix them in position.
c- The foot immobilized in a below-knee cast for 6–8 weeks.
d- Open reduction and screw fixation may be necessary.
e- Early arthrodesis, with restoration of the longitudinal arch, is advisable.
496- The tarso-metatarsal (TMT) injuries
a- Dislocation is common.
b- The term Lisfranc injury, often used for these disruptions.
c- Classifying these by direction of forefoot dislocation.
d- Often high-energy injuries with extensive damage to the whole region of the
foot.
e- CT scan is a more efficient way of showing the extent of injury.
497- Compartment syndrome following tarsometatarsal injuries
a- A tensely swollen foot may hide a serious compartment syndrome.
b- If this suspected, intracompartmental pressures measured.
c- Treatment should be prompt and effective.
d- The medial compartments decompressed;
e- The wound left open until swelling subsides and the skin closed without
tension.
498- Metatarsal fractures
a- Are relatively uncommon.
b- May be crush fractures due to a direct blow;
c- May be a spiral fracture of the shaft due to a twisting injury.
d- Avulsion fractures due to ligament strains.
e- Insufficiency fractures due to repetitive stress.
499- Metatarsal stress injury ( march fracture)
a- Occur in a young adult (often a military recruit or a nurse).
b- The foot may become painful and slightly swollen after overuse.
c- A tender lump is palpable just distal to the mid-shaft of a metatarsal bone.
d- Usually the fifth metatarsal affected.
e- A hairline crack may be visible and 4 weeks later, a mass of callus seen.
500- Forefoot pain in elderly people
a- May be due to stress fractures.
b- X-ray diagnosis is more difficult because callus is minimal.
c- If osteoporosis not already diagnosed, bone densitometry should be
considered.
d- Metatarsal pain after forefoot surgery may also be due to stress fractures of the
adjacent metatarsals.
e- Stress fractures treated by fixation by percutaneous Kirschner wires and cast.
Key of part one
Number Answer Number Answer Number Answer Number Answer
1 C 37 D 73 A 109 A
2 D 38 E 74 B 110 C
3 B 39 C 75 D 111 D
4 B 40 A 76 D 112 E
5 C 41 D 77 E 113 B
6 E 42 C 78 D 114 D
7 D 43 D 79 C 115 A
8 D 44 C 80 B 116 C
9 E 45 E 81 D 117 E
10 E 46 B 82 E 118 B
11 D 47 E 83 D 119 D
12 E 48 C 84 A 120 C
13 B 49 D 85 E 121 D
14 B 50 A 86 C 122 B
15 D 51 E 87 C 123 B
16 A 52 B 88 B 124 D
17 B 53 A 89 D 125 E
18 A 54 B 90 C 126 C
19 E 55 C 91 C 127 B
20 C 56 A 92 A 128 C
21 E 57 B 93 D 129 B
22 A 58 D 94 E 130 D
23 E 59 B 95 B 131 E
24 D 60 C 96 B 132 D
25 E 61 E 97 C 133 E
26 A 62 E 98 E 134 C
27 B 63 A 99 C 135 B
28 E 64 B 100 B 136 D
29 B 65 E 101 C 137 B
30 C 66 D 102 B 138 B
31 B 67 D 103 C 139 B
32 D 68 A 104 A 140 A
33 E 69 D 105 D 141 A
34 D 70 C 106 C 142 D
35 C 71 E 107 B 143 D
36 D 72 B 108 A 144 B
Number Answer Number Answer Number Answer Number Answer
145 C 185 E 225 D 265 D
146 E 186 D 226 C 266 A
147 E 187 C 227 A 267 C
148 D 188 D 228 D 268 C
149 D 189 D 229 A 269 E
150 E 190 E 230 B 270 C
151 C 191 A 231 C 271 A
152 B 192 D 232 E 272 B
153 D 193 E 233 D 273 C
154 A 194 E 234 B 274 D
155 D 195 B 235 A 275 C
156 A 196 C 236 B 276 E
157 B 197 A 237 D 277 D
158 E 198 C 238 B 278 C
159 B 199 E 239 D 279 D
160 A 200 B 240 B 280 B
161 E 201 E 241 A 281 C
162 D 202 B 242 B 282 B
163 D 203 D 243 D 283 D
164 D 204 B 244 D 284 B
165 E 205 A 245 B 285 D
166 C 206 B 246 C 286 E
167 A 207 E 247 D 287 B
168 C 208 D 248 A 288 D
169 D 209 B 249 B 289 C
170 D 210 E 250 B 290 C
171 C 211 D 251 A 291 E
172 B 212 D 252 C 292 C
173 D 213 A 253 D 293 E
174 C 214 B 254 D 294 C
175 E 215 E 255 A 295 D
176 D 216 C 256 E 296 E
177 B 217 A 257 D 297 B
178 A 218 B 258 A 298 E
179 D 219 D 259 A 299 A
180 A 220 E 260 B 300 A
181 C 221 A 261 C 301 D
182 E 222 C 262 C 302 B
183 A 223 E 263 B 303 C
184 B 224 C 264 E 304 D
Number Answer Number Answer Number Answer Number Answer
305 A 345 B 385 E 425 D
306 B 346 C 386 E 426 D
307 E 347 D 387 A 427 E
308 C 348 E 388 C 428 D
309 D 349 C 389 E 429 B
310 A 350 C 390 C 430 D
311 B 351 D 391 A 431 D
312 C 352 E 392 B 432 E
313 D 353 D 393 D 433 A
314 A 354 B 394 C 434 D
315 E 355 A 395 E 435 C
316 B 356 E 396 C 436 D
317 E 357 B 397 D 437 D
318 C 358 D 398 A 438 A
319 A 359 C 399 A 439 C
320 D 360 A 400 E 440 C
321 C 361 E 401 D 441 A
322 D 362 B 402 E 442 D
323 E 363 C 403 C 443 C
324 E 364 C 404 B 444 B
325 C 365 E 405 C 445 D
326 A 366 B 406 C 446 E
327 E 367 D 407 B 447 D
328 E 368 C 408 D 448 C
329 A 369 D 409 A 449 E
330 B 370 A 410 B 450 A
331 C 371 D 411 D 451 A
332 D 372 D 412 C 452 B
333 A 373 B 413 A 453 E
334 B 374 D 414 C 454 A
335 E 375 C 415 B 455 D
336 A 376 D 416 D 456 C
337 D 377 C 417 A 457 D
338 C 378 C 418 B 458 B
339 B 379 B 419 D 459 A
340 C 380 B 420 B 460 C
341 A 381 B 421 E 461 C
342 E 382 D 422 C 462 B
343 C 383 C 423 D 463 E
344 A 384 D 424 C 464 A
Number Answer Number Answer Number Answer Number Answer
465 D 475 A 485 A 495 E
466 E 476 B 486 D 496 C
467 C 477 C 487 C 497 B
468 B 478 B 488 E 498 B
469 B 479 B 489 B 499 B
470 A 480 D 490 A 500 A
471 C 481 D 491 D 501 B
472 C 482 E 492 D
473 D 483 D 493 C
474 D 484 B 494 D
Key of part two

Number Answer Number Answer Number Answer Number Answer


1 B 37 E 73 E 109 C
2 E 38 D 74 A 110 B
3 D 39 C 75 C 111 E
4 C 40 B 76 B 112 E
5 D 41 E 77 D 113 D
6 E 42 E 78 E 114 E
7 E 43 B 79 A 115 C
8 C 44 D 80 E 116 D
9 A 45 D 81 A 117 B
10 C 46 E 82 B 118 E
11 D 47 C 83 C 119 B
12 E 48 B 84 C 120 D
13 E 49 E 85 B 121 B
14 A 50 D 86 C 122 B
15 A 51 C 87 A 123 C
16 E 52 D 88 E 124 E
17 B 53 C 89 B 125 D
18 D 54 D 90 A 126 E
19 A 55 D 91 E 127 D
20 E 56 E 92 B 128 E
21 B 57 B 93 E 129 C
22 C 58 D 94 C 130 C
23 B 59 A 95 A 131 E
24 D 60 C 96 C 132 E
25 E 61 E 97 D 133 A
26 D 62 B 98 E 134 E
27 A 63 E 99 D 135 C
28 E 64 C 100 B 136 E
29 D 65 B 101 C 137 A
30 B 66 C 102 A 138 D
31 E 67 E 103 D 139 E
32 B 68 B 104 D 140 D
33 A 69 E 105 E 141 B
34 E 70 D 106 C 142 A
35 B 71 D 107 C 143 D
36 A 72 B 108 D 144 E
Number Answer Number Answer Number Answer Number Answer
145 C 185 B 225 C 265 C
146 D 186 E 226 A 266 B
147 E 187 B 227 D 267 D
148 E 188 B 228 D 268 B
149 B 189 C 229 B 269 E
150 D 190 E 230 E 270 C
151 B 191 B 231 C 271 D
152 E 192 A 232 D 272 E
153 B 193 C 233 A 273 E
154 E 194 B 234 B 274 C
155 C 195 D 235 B 275 B
156 E 196 D 236 D 276 A
157 A 197 B 237 A 277 C
158 D 198 E 238 D 278 C
159 C 199 C 239 C 279 A
160 E 200 B 240 B 280 C
161 B 201 E 241 E 281 A
162 D 202 C 242 C 282 C
163 B 203 D 243 D 283 E
164 E 204 A 244 A 284 D
165 D 205 D 245 B 285 D
166 C 206 E 256 E 286 E
167 E 207 C 247 B 287 C
168 A 208 A 248 C 288 E
169 E 209 B 249 A 289 A
170 D 210 E 250 E 290 B
171 B 211 C 251 B 291 A
172 E 212 A 252 A 292 C
173 E 213 B 253 E 293 E
174 D 214 D 254 B 294 D
175 E 215 D 255 A 295 C
176 D 216 B 256 A 296 A
177 E 217 A 257 C 297 E
178 C 218 C 258 D 298 D
179 D 219 B 259 A 299 B
180 A 220 D 260 E 300 C
181 B 221 C 261 A 301 E
182 C 222 B 262 C 302 A
183 A 223 D 263 D 303 E
184 C 224 A 264 A 304 D

Number Answer Number Answer Number Answer Number Answer


305 B 345 C 385 C 425 D
306 A 346 A 386 B 426 C
307 B 347 D 387 A 427 E
308 E 348 D 388 B 428 C
309 B 349 C 389 A 429 B
310 D 350 D 390 E 430 E
311 A 351 C 391 C 431 B
312 B 352 E 392 D 432 C
313 C 353 B 393 D 433 C
314 A 354 B 394 E 434 D
315 E 355 C 395 B 435 E
316 D 356 E 396 A 436 C
317 D 357 D 397 D 437 B
318 A 358 B 398 B 438 B
319 B 359 E 399 A 439 C
320 E 360 E 400 D 440 A
321 D 361 D 401 E 441 D
322 A 362 E 402 B 442 C
323 C 363 D 403 D 443 B
324 D 364 D 404 C 444 C
325 C 365 A 405 E 445 C
326 C 366 C 406 D 446 D
327 B 367 B 407 D 447 E
328 D 368 C 408 B 448 C
329 D 369 B 409 D 449 A
330 D 370 C 410 E 450 C
331 C 371 C 411 E 451 D
332 E 372 B 412 C 452 C
333 D 373 C 413 B 453 E
334 D 374 C 414 C 454 D
335 C 375 D 415 A 455 B
336 A 376 B 416 D 456 D
337 E 377 C 417 B 457 E
338 B 378 E 418 B 458 C
339 C 379 D 419 D 459 D
340 D 380 A 420 B 460 A
341 E 381 B 421 E 461 B
342 C 382 E 422 D 462 E
343 E 383 C 423 C 463 E
344 C 384 D 424 E 464 D

Number Answer Number Answer Number Answer Number Answer


465 B 474 B 483 C 492 D
466 C 475 D 484 D 493 D
467 A 476 B 485 B 494 E
468 C 477 C 486 C 495 E
469 B 478 C 487 C 496 A
470 B 479 A 488 D 497 D
471 E 480 D 489 D 498 A
472 B 481 C 490 E 499 D
473 D 482 E 491 B 500 E
View publication stats

You might also like